SlideShare a Scribd company logo
1 of 322
Download to read offline
F
E
D
E
RA
L D
E
MO
C
RA
T
I
CRE
P
UB
L
I
CO
FE
T
HI
O
P
I
A
MI
NI
S
T
R
Y O
FE
D
UC
A
T
I
O
N
G
R
A
D
E9
7DNH*RRGDUHRI
7DNH*RRGDUHRI
7DNH*RRGDUHRI
7DNH*RRGDUHRI
7KLV7H[WERRN
7KLV7H[WERRN
7KLV7H[WERRN
7KLV7H[WERRN
 dŚŝƐƚĞdžƚŬŝƐƚŚĞƉƌŽƉĞƌƚLJŽĨLJŽƵƌƐĐŚŽŽů͘
dĂŬĞŐŽŽĚĐĂƌĞŶŽƚƚŽĚĂŵĂŐĞŽƌůŽƐĞŝƚ͘
,ĞƌĞĂƌĞϭϬŝĚĞĂƐƚŽŚĞůƉƚĂŬĞĐĂƌĞŽĨƚŚĞŬ͗
ϭ͘ ŽǀĞƌƚŚĞŬǁŝƚŚƉƌŽƚĞĐƚŝǀĞŵĂƚĞƌŝĂů͕ƐƵĐŚĂƐƉůĂƐƚŝĐ͕ŽůĚ
ŶĞǁƐƉĂƉĞƌƐŽƌŵĂŐĂnjŝŶĞƐ͘
Ϯ͘ ůǁĂLJƐŬĞĞƉƚŚĞŬŝŶĂĐůĞĂŶĚƌLJƉůĂĐĞ͘
ϯ͘ ĞƐƵƌĞLJŽƵƌŚĂŶĚƐĂƌĞĐůĞĂŶǁŚĞŶLJŽƵƵƐĞƚŚĞŬ͘
ϰ͘ ŽŶŽƚǁƌŝƚĞŽŶƚŚĞĐŽǀĞƌŽƌŝŶƐŝĚĞƉĂŐĞƐ͘
ϱ͘ hƐĞĂƉŝĞĐĞŽĨƉĂƉĞƌŽƌĐĂƌĚďŽĂƌĚĂƐĂŬŵĂƌŬ͘
ϲ͘ EĞǀĞƌƚĞĂƌŽƌĐƵƚŽƵƚĂŶLJƉŝĐƚƵƌĞƐŽƌƉĂŐĞƐ͘
ϳ͘ ZĞƉĂŝƌĂŶLJƚŽƌŶƉĂŐĞƐǁŝƚŚƉĂƐƚĞŽƌƚĂƉĞ͘
ϴ͘ WĂĐŬƚŚĞŬĐĂƌĞĨƵůůLJǁŚĞŶLJŽƵƉůĂĐĞŝƚŝŶLJŽƵƌƐĐŚŽŽůďĂŐ͘
ϵ͘ ,ĂŶĚůĞƚŚĞŬǁŝƚŚĐĂƌĞǁŚĞŶƉĂƐƐŝŶŐŝƚƚŽĂŶŽƚŚĞƌƉĞƌƐŽŶ͘
ϭϬ͘ tŚĞŶƵƐŝŶŐĂŶĞǁŬĨŽƌƚŚĞĨŝƌƐƚƚŝŵĞ͕ůĂLJŝƚŽŶŝƚƐďĂĐŬ͘KƉĞŶ
ŽŶůLJĂĨĞǁƉĂŐĞƐĂƚĂƚŝŵĞ͘WƌĞƐƐůŝŐŚƚůLJĂůŽŶŐƚŚĞďŽƵŶĚĞĚŐĞĂƐ
LJŽƵƚƵƌŶƚŚĞƉĂŐĞƐ͘dŚŝƐǁŝůůŬĞĞƉƚŚĞĐŽǀĞƌŝŶŐŽŽĚĐŽŶĚŝƚŝŽŶ͘
MATHEMATICS
STUDENT TEXTBOOK
Authors, Editors and Reviewers:
C.K. Bansal (B.Sc., M.A.)
Rachel Mary Z
Dawod Adem
Kassa Michael
Berhanu Guta
Evaluators:
Tesfaye Ayele
Dagnachew Yalew
Tekeste Woldetensai
FEDERAL DEMOCRATIC REPUBLIC OF ETHIOPIA
MATHEMATICS
STUDENT TEXTBOOK
GRADE
GRADE
GRADE
GRADE 9
9
9
9
Authors, Editors and Reviewers:
(B.Sc., M.A.)
Rachel Mary Z. (B.Sc.)
(M.Sc.)
Kassa Michael (M.Sc.)
Berhanu Guta (Ph.D.)
Tesfaye Ayele
Dagnachew Yalew
Tekeste Woldetensai
FEDERAL DEMOCRATIC REPUBLIC OF ETHIOPIA
MINISTRY OF EDUCATION
MATHEMATICS
STUDENT TEXTBOOK
9
9
9
9
Published E.C. 2002 by the Federal Democratic Republic of Ethiopia,
Ministry of Education, under the General Education Quality
Improvement Project (GEQIP) supported by IDA Credit No. 4535-ET,
the Fast Track Initiative Catalytic Fund and the Governments of Finland,
Italy, Netherlands and the United Kingdom.
© 2010 by the Federal Democratic Republic of Ethiopia, Ministry of Education.
All rights reserved. No part of this book may be reproduced, stored in a retrieval
system or transmitted in any form or by any means including electronic,
mechanical, magnetic or other, without prior written permission of the Ministry
of Education or licensing in accordance with the Federal Democratic Republic of
Ethiopia, Federal Negarit Gazeta, Proclamation No. 410/2004 – Copyright and
Neighbouring Rights Protection.
The Ministry of Education wishes to thank the many individuals, groups and
other bodies involved – directly and indirectly – in publishing this textbook and
the accompanying teacher guide.
Copyrighted materials used by permission of their owners. If you are the owner
of copyrighted material not cited or improperly cited, please contact with the
Ministry of Education, Head Office, Arat Kilo, (PO Box 1367), Addis Ababa,
Ethiopia.
PHOTO CREDIT: p.106 and 292-Encarta Encyclopedia, 2009 edition.
p.126-http:// en.wikipedia.org
p.277-http://en.wikipedia.org
Developed and Printed by
STAR EDUCATIONAL BOOKS DISTRIBUTORS Pvt. Ltd.
24/4800, Bharat Ram Road, Daryaganj,
New Delhi – 110002, INDIA
and
ASTER NEGA PUBLISHING ENTERPRISE
P.O. Box 21073
ADDIS ABABA, ETHIOPIA
under GEQIP Contract No. ET-MoE/GEQIP/IDA/ICB/G01/09.
ISBN 978-99944-2-042-1
U
Numerical Value
Arabic
Numeral
1 2 3 5 10 20 21 100
Babylonian        
Egyptian
Hieroglyphic
I II III IIIII Λ ΛΛ IΛΛ ϑ
Greek
Herodianic
Ι ΙΙ ΙΙΙ Γ ∆ ∆∆ ∆∆Ι Η
Roman I II III V X XX XXI C
Ethiopian
Geez
1 2 3 5 0 ! !1 )
Contents
Unit 1
1 The Number System........................ 1
1.1 Revision on the set of rational numbers...........2
1.2 The real number system....................................13
Key Terms ............................................. 59
Summary .............................................. 59
Review Exercises on Unit 1..................... 61
Unit 2
2 Solutions of Equations .................63
2.1 Equations involving exponents and
radicals...............................................................64
2.2 Systems of linear equations in two
variables.............................................................66
2.3 Equations involving absolute value .................82
2.4 Quadratic equations.........................................86
Key Terms ............................................. 101
Summary .............................................. 102
Review Exercises on Unit 2..................... 103
Unit 3
3 Further on Sets 105
3.1 Ways to describe sets........................................106
3.2 The notation of sets............................................110
3.3 Operations on sets.............................................119
Key Terms ............................................. 135
Summary .............................................. 135
Review Exercises on Unit 3..................... 136
Unit 4
4 Relations and Functions .......... 139
4.1 Relations.............................................................140
4.2 Functions ............................................................149
4.3 Graphs of functions ...........................................157
Key Terms ............................................. 169
Summary .............................................. 170
Review Exercises on Unit 4..................... 171
Contents
Unit 5
5 Geometry and Measurement ... 175
5.1 Regular polygons ..............................................176
5.2 Further on congruency and similarity..............190
5.3 Further on trigonometry.....................................210
5.4 Circles.................................................................221
5.5 Measurement.....................................................232
Key Terms ............................................. 243
Summary .............................................. 243
Review Exercises on Unit 5..................... 247
Unit 6
6 Statistics and Probability.......... 249
6.1 Statistical data ...................................................250
6.2 Probability ..........................................................277
Key Terms ............................................. 286
Summary .............................................. 286
Review Exercises on Unit 6..................... 288
Unit 7
7 Vectors in Two Dimensions...... 291
7.1 Introduction to Vectors and scalars.................292
7.2 Representation of a vector ...............................294
7.3 Addition and subtraction of vectors and
multiplication of a vector by a scalar.............. 299
7.4 Position vector of a point ..................................307
Key Terms ............................................. 311
Summary .............................................. 311
Review Exercises on Unit 7..................... 312
Table of Trigonometric Functions...............................313
Numerical Value
Arabic
Numeral
1 2 3 5 10 20 21 100
Babylonian        
Egyptian
Hieroglyphic
I II III IIIII Λ ΛΛ IΛΛ ϑ
Greek
Herodianic
Ι ΙΙ ΙΙΙ Γ ∆ ∆∆ ∆∆Ι Η
Roman I II III V X XX XXI C
Ethiopian
Geez
1 2 3 5 0 ! !1 )
Unit Outcomes:
After completing this unit, you should be able to:
know basic concepts and important facts about real numbers.
justify methods and procedures in computation with real numbers.
solve mathematical problems involving real numbers.
Main Contents
1.1 Revision on the set of rational numbers
1.2 The real number system
Key Terms
Summary
Review Exercises
U
Un
ni
it
t
T
T
T
T
T
T
T
TH
H
H
H
H
H
H
HE
E
E
E
E
E
E
E N
N
N
N
N
N
N
NU
U
U
U
U
U
U
UM
M
M
M
M
M
M
MB
B
B
B
B
B
B
BE
E
E
E
E
E
E
ER
R
R
R
R
R
R
R S
S
S
S
S
S
S
SY
Y
Y
Y
Y
Y
Y
YS
S
S
S
S
S
S
ST
T
T
T
T
T
T
TE
E
E
E
E
E
E
EM
M
M
M
M
M
M
M
Mathematics Grade 9
2
INTRODUCTION
In earlier grades, you have learnt about rational numbers, their properties, and basic
mathematical operations upon them. After a review of your knowledge about rational
numbers, you will continue studying the number systems in the present unit. Here, you
will learn about irrational numbers and real numbers, their properties and basic
operations upon them. Also, you will discuss some related concepts such as
approximation, accuracy, and scientific notation.
1.1 REVISION ON THE SET OF RATIONAL
NUMBERS
ACTIVITY 1.1
The diagram below shows the relationships between the sets of
Natural numbers, Whole numbers, Integers and Rational numbers.
Use this diagram to answer Questions 1 and 2 given below. Justify
your answers.
1 To which set(s) of numbers does each of the following numbers belong?
a 27 b –17 c –7
2
3
d 0.625 e 0.615
2 i Define the set of:
a Natural numbers
b Whole numbers
c Integers
d Rational numbers
ii What relations do these sets have?
Figure 1.1
1.1.1 Natural Numbers, Integers, Prime
Numbers and Composite Numbers
In this subsection, you will revise important facts about the sets of natural numbers, prime
numbers, composite numbers and integers. You have learnt several facts about these sets
in previous grades, in Grade 7 in particular. Working through Activity 1.2 below will
refresh your memory!
1, 2, 3, . . .
0
–11
–3
0 7
.
3
4
1
2
−
Unit 1 The Number System
3
ACTIVITY 1.2
1 For each of the following statements write 'true' if the statement is
correct or 'false' otherwise. If your answer is 'false', justify by
giving a counter example or reason.
a The set {1, 2, 3, . . .} describes the set of natural numbers.
b The set {1, 2, 3, . . .}⋃{. . . −3, −2, −1} describes the set of integers.
c 57 is a composite number.
d {1} ∩ {Prime numbers} = ∅.
e {Prime numbers}⋃{Composite number} = {1, 2, 3, . . .}.
f {Odd numbers} ∩{Composite numbers} ≠ ∅.
g 48 is a multiple of 12.
h 5 is a factor of 72.
i 621 is divisible by 3.
j {Factors of 24} ∩ {Factors of 87} = {1, 2, 3}.
k {Multiples of 6} ∩ {Multiples of 4} = {12, 24}.
l 2 2
2 3 5
× × is the prime factorization of 180.
2 Given two natural numbers a and b, what is meant by:
a a is a factor of b b a is divisible by b c a is a multiple of b
From your lower grade mathematics, recall that;
 The set of natural numbers, denoted by N, is described by N = {1, 2, 3,…}
 The set of whole numbers, denoted by W, is described by W = {0, 1, 2, 3,…}
 The set of integers, denoted by Z, is described by Z = {...,–3, –2, –1, 0, 1, 2, 3,...}
 Given two natural numbers m and p, m is called a multiple of p if there is a
natural number q such that
m = p × q.
In this case, p is called a factor or divisor of m. We also say m is divisible by p.
Similarly, q is also a factor or divisor of m, and m is divisible by q.
Mathematics Grade 9
4
For example, 621 is a multiple of 3 because 621 = 3 × 207.
Note: 1 is neither prime nor composite.
G r o u p W o r k 1 . 1
G r o u p W o r k 1 . 1
G r o u p W o r k 1 . 1
G r o u p W o r k 1 . 1
1 List all factors of 24. How many factors did you find?
2 The area of a rectangle is 432 sq. units. The measurements
of the length and width of the rectangle are expressed by
natural numbers.
Find all the possible dimensions (length and width) of the rectangle.
3 Find the prime factorization of 360.
The following rules can help you to determine whether a number is divisible by 2, 3, 4,
5, 6, 8, 9 or 10.
Divisibility test
A number is divisible by:
 2, if its unit’s digit is divisible by 2.
 3, if the sum of its digits is divisible by 3.
 4, if the number formed by its last two digits is divisible by 4.
 5, if its unit’s digit is either 0 or 5.
 6, if it is divisible by 2 and 3.
 8, if the number formed by its last three digits is divisible by 8.
 9, if the sum of its digits is divisible by 9.
 10, if its unit’s digit is 0.
Observe that divisibility test for 7 is not stated here as it is beyond the scope of your
present level.
Definition 1.1 Prime numbers and composite numbers
♦
♦
♦ A natural number that has exactly two distinct factors, namely 1
and itself, is called a prime number.
♦
♦
♦ A natural number that has more than two factors is called a
composite number.
Unit 1 The Number System
5
Example 1 Use the divisibility test to determine whether 2,416 is divisible by 2, 3, 4,
5, 6, 8, 9 and 10.
Solution: ♦ 2, 416 is divisible by 2 because the unit’s digit 6 is divisible by 2.
♦ 2,416 is divisible by 4 because 16 (the number formed by the last two digits)
is divisible by 4.
♦ 2,416 is divisible by 8 because the number formed by the last three digits
(416) is divisible by 8.
♦ 2,416 is not divisible by 5 because the unit’s digit is not 0 or 5.
♦ Similarly you can check that 2,416 is not divisible by 3, 6, 9, and 10.
Therefore, 2,416 is divisible by 2, 4 and 8 but not by 3, 5, 6, 9 and 10.
A factor of a composite number is called a prime factor, if it is a prime number. For
instance, 2 and 5 are both prime factors of 20.
Every composite number can be written as a product of prime numbers. To find the prime
factors of any composite number, begin by expressing the number as a product of two
factors where at least one of the factors is prime. Then, continue to factorize each resulting
composite factor until all the factors are prime numbers.
When a number is expressed as a product of its prime factors, the expression is called the
prime factorization of the number.
For example, the prime factorization of 60 is
60 = 2 × 2 × 3 × 5 = 22
× 3 × 5.
The prime factorization of 60 is also found by
using a factoring tree.
Note that the set {2, 3, 5} is a set of prime factors of 60. Is this set unique? This
property leads us to state the Fundamental Theorem of Arithmetic.
You can use the divisibility tests to check whether or not a prime number divides a
given number.
Theorem 1.1 Fundamental theorem of arithmetic
Every composite number can be expressed (factorized) as a product of
primes. This factorization is unique, apart from the order in which the
prime factors occur.
2
60
2 30
3 5
15
Mathematics Grade 9
6
Example 2 Find the prime factorization of 1,530.
Solution: Start dividing 1,530 by its smallest prime factor. If the quotient is a
composite number, find a prime factor of the quotient in the same way.
Repeat the procedure until the quotient is a prime number as shown below.
1,530 765
765 255
255 85
85
Prime factors
17
2
3
; and 17 is a prime
3
5 number.
÷ =
÷ =
÷ =
÷ =
↓
Therefore, 1,530 = 2 × 32
× 5 × 17.
1.1.2 Common Factors and Common Multiples
In this subsection, you will revise the concepts of common factors and common
multiples of two or more natural numbers. Related to this, you will also revise the
greatest common factor and the least common multiple of two or more natural numbers.
A Common factors and the greatest common factor
ACTIVITY 1.3
1 Given the numbers 30 and 45,
a find the common factors of the two numbers.
b find the greatest common factor of the two numbers.
2 Given the numbers 36, 42 and 48,
a find the common factors of the three numbers.
b find the greatest common factor of the three numbers.
Given two or more natural numbers, a number which is a factor of all of them is called a
common factor. Numbers may have more than one common factor. The greatest of the
common factors is called the greatest common factor (GCF) or the highest common
factor (HCF) of the numbers.
 The greatest common factor of two numbers a and b is denoted by GCF (a, b).
Example 1 Find the greatest common factor of:
a 36 and 60. b 32 and 27.
Unit 1 The Number System
7
Solution:
a First, make lists of the factors of 36 and 60, using sets.
Let F36 and F60 be the sets of factors of 36
and 60, respectively. Then,
F36 = {1, 2, 3, 4, 6, 9, 12, 18, 36}
F60 = {1, 2, 3, 4, 5, 6, 10, 12, 15, 20, 30, 60}
You can use the diagram to summarize the
information. Notice that the common factors
are shaded in green. They are 1, 2, 3, 4, 6 and
12 and the greatest is 12.
i.e., F36 ∩ F60 = {1, 2, 3, 4, 6, 12}
Therefore, GCF (36, 60) = 12.
b Similarly,
F32 = {1, 2, 4, 8, 16, 32} and
F27 = {1, 3, 9, 27}
Therefore, F32 ∩ F27 = {1}
Thus, GCF (32, 27) = 1
 Two or more natural numbers that have a GCF of 1 are called relatively prime.
G r o u p W o r k 1 .
G r o u p W o r k 1 .
G r o u p W o r k 1 .
G r o u p W o r k 1 . 2
2
2
2
Let a = 1800 and b = 756
1 Write:
a the prime factorization of a and b
b the prime factors that are common to both a and b.
Now look at these common prime factors; the lowest powers of them (in the two
prime factorizations) should be 22
and 32
.
c What is the product of these lowest powers?
d Write down the highest powers of the common prime factors.
e What is the product of these highest powers?
Definition 1.2
The greatest common factor (GCF) of two or more natural numbers is the
greatest natural number that is a factor of all of the given numbers.
Figure 1.3
F32 F27
2
4
8
16
32
3
9
27
1
Figure 1.2
F36 F60
9
18
36
5
10 15
20 30
60
1
2 3 4
6 12
Mathematics Grade 9
8
2 a Compare the result of 1c with the GCF of the given numbers.
Are they the same?
b Compare the result of 1e with the GCF of the given numbers.
Are they the same?
The above Group Work leads you to another alternative method to find the GCF of
numbers. This method (which is a quicker way to find the GCF) is called the prime
factorization method. In this method, the GCF of a given set of numbers is the
product of their common prime factors, each power to the smallest number of times it
appears in the prime factorization of any of the numbers.
Example 2 Use the prime factorization method to find GCF (180, 216, 540).
Solution:
Step 1 Express the numbers 180, 216 and 540 in their prime factorization.
180 = 22
× 32
× 5; 216 = 23
× 33
; 540 = 22
× 33
× 5
Step 2 As you see from the prime factorizations of 180, 216 and 540, the
numbers 2 and 3 are common prime factors.
So, GCF (180, 216, 540) is the product of these common prime factors with the
smallest respective exponents in any of the numbers.
∴ GCF (180, 216, 540) = 22
× 32
= 36.
B Common multiples and the least common multiple
G r o u p W o r k 1 .
G r o u p W o r k 1 .
G r o u p W o r k 1 .
G r o u p W o r k 1 . 3
3
3
3
For this group work, you need 2 coloured pencils.
Work with a partner
Try this:
∗
∗
∗
∗ List the natural numbers from 1 to 100 on a sheet of paper.
∗
∗
∗
∗ Cross out all the multiples of 10.
∗
∗
∗
∗ Using a different colour, cross out all the multiples of 8.
Discuss:
1 Which numbers were crossed out by both colours?
2 How would you describe these numbers?
3 What is the least number crossed out by both colours? What do you call this number?
Unit 1 The Number System
9
Example 3 Find LCM (8, 9).
Solution: Let M8 and M9 be the sets of multiples of 8 and 9 respectively.
M8 ={ }
8,16, 24, 32, 40, 48, 56, 64, , 80, 88
72 ,...
M9 = { }
9,18, 27, 36, 45, 54, 63, , 81, 90
72 ,...
Therefore LCM (8, 9) = 72
Prime factorization can also be used to find the LCM of a set of two or more than two
numbers. A common multiple contains all the prime factors of each number in the set.
The LCM is the product of each of these prime factors to the greatest number of times it
appears in the prime factorization of the numbers.
Example 4 Use the prime factorization method to find LCM (9, 21, 24).
Solution:
9 = 3 × 3 = 32
21 = 3 × 7
24 = 2 × 2 × 2 ×3 = 23
× 3
Therefore, LCM (9, 21, 24) = 23
× 32
× 7 = 504.
ACTIVITY 1.4
1 Find:
a The GCF and LCM of 36 and 48
b GCF (36, 48) × LCM ( 36, 48)
c 36 × 48
2 Discuss and generalize your results.
 For any natural numbers a and b, GCF (a, b) × LCM (a, b) = a × b.
Definition 1.3
For any two natural numbers a and b, the least common multiple of a
and b denoted by LCM (a, b), is the smallest multiple of both a and b.
The prime factors that appear in these
factorizations are 2, 3 and 7.
Considering the greatest number of times
each prime factor appears, we can get 23
,
32
and 7, respectively.
Mathematics Grade 9
10
1.1.3 Rational Numbers
H
H
H I
I
I S
S
S T
T
T O
O
O R
R
R I
I
I C
C
C A
A
A L
L
L N
N
N O
O
O T
T
T E
E
E :
::
About 5,000 years ago, Egyptians used
hieroglyphics to represent numbers.
The Egyptian concept of fractions was
mostly limited to fractions with numerator
1. The hieroglyphic was placed under the
symbol to indicate the number as a
denominator. Study the examples of
Egyptian fractions.
Recall that the set of integers is given by
Z = {…, – 3, – 2, – 1, 0, 1, 2, 3,…}
Using the set of integers, we define the set of rational numbers as follows:
Through the following diagram, you can
show how sets within rational numbers are
related to each other. Note that natural
numbers, whole numbers and integers are
included in the set of rational numbers. This
is because integers such as 4 and −7 can be
written as
4 7
and
1 1
−
.
The set of rational numbers also includes
terminating and repeating decimal numbers
because terminating and repeating decimals
can be written as fractions.
Definition 1.4 Rational number
Any number that can be expressed in the form
a
b
, where a and b are
integers and b ≠ 0, is called a rational number. The set of rational
numbers, denoted by Q, is the set described by
Q = : and are integers and 0
a
a b b
b
 
≠
 
 
.
1,000

1 10 100
 1 1 1 1 1
2 3 10 20 100
4
6
0
2
2
3
3
3
−8
−7
1
−9
3
1
2
−
2
5
Figure 1.4
Unit 1 The Number System
11
For example, −1.3 can be written as 13 29
and 0.29 as
10 100
− −
− .
Mixed numbers are also included in the set of rational numbers because any mixed number
30
45
2 3 5 2
= = .
3 3 5 3
× ×
× ×
can be written as an improper fraction.
For example, 2
2
3
can be written as
8
.
3
When a rational number is expressed as a fraction, it is often expressed in simplest form
(lowest terms). A fraction
a
b
is in simplest form when GCF (a, b) = 1.
Example 1 Write
30
45
in simplest form.
Solution:
30
45
2 3 5 2
= = .
3 3 5 3
× ×
× ×
(by factorization and cancellation)
Hence
30
45
when expressed in lowest terms (simplest form) is
2
.
3
Exercise 1.1
1 Determine whether each of the following numbers is prime or composite:
a 45 b 23 c 91 d 153
2 Prime numbers that differ by two are called twin primes.
i Which of the following pairs are twin primes?
a 3 and 5 b 13 and 17 c 5 and 7
ii List all pairs of twin primes that are less than 30.
3 Determine whether each of the following numbers is divisible by 2, 3, 4, 5, 6, 8, 9
or 10:
a 48 b 153 c 2,470
d 144 e 12,357
4 a Is 3 a factor of 777? b Is 989 divisible by 9?
c Is 2,348 divisible by 4?
5 Find three different ways to write 84 as a product of two natural numbers.
6 Find the prime factorization of:
a 25 b 36 c 117 d 3,825
Mathematics Grade 9
12
7 Is the value of 2a + 3b prime or composite when a = 11 and b = 7?
8 Write all the common factors of 30 and 42.
9 Find:
a GCF (24, 36) b GCF (35, 49, 84)
10 Find the GCF of 2 × 33
× 52
and 23
× 3 × 52
.
11 Write three numbers that have a GCF of 7.
12 List the first six multiples of each of the following numbers:
a 7 b 5 c 14 d 25 e 150
13 Find:
a LCM (12, 16) b LCM (10, 12, 14)
c LCM (15, 18) d LCM (7, 10)
14 When will the LCM of two numbers be the product of the numbers?
15 Write each of the following fractions in simplest form:
a
3
9
− b
24
120
c
48
72
d
72
98
16 How many factors does each of the following numbers have?
a 12 b 18 c 24 d 72
17 Find the value of an odd natural number x if LCM (x, 40) = 1400.
18 There are between 50 and 60 eggs in a basket. When Mohammed counts by 3’s,
there are 2 eggs left over. When he counts by 5’s there are 4 left over. How many
eggs are there in the basket?
19 The GCF of two numbers is 3 and the LCM is 180. If one of the numbers is 45,
what is the other number?
20 i Let a, b, c, d be non-zero integers. Show that each of the following is a
rational number:
a
a c
b d
+ b
a c
b d
− c
a c
b d
× d
a c
b d
÷
What do you conclude from these results?
ii Find two rational numbers between
1
3
and
3
4
.
Unit 1 The Number System
13
1.2 THE REAL NUMBER SYSTEM
1.2.1 Representation of Rational Numbers by
Decimals
In this subsection, you will learn how to express rational numbers in the form of
fractions and decimals.
ACTIVITY 1.5
1 a What do we mean by a ‘decimal number’?
b Give some examples of decimal numbers.
2 How do you represent
3 1
and
4 3
as decimals?
3 Can you write 0.4 and 1.34 as the ratio (or quotient) of two integers?
Remember that a fraction is another way of writing division of one quantity by another.
Any fraction of natural numbers can be expressed as a decimal by dividing the
numerator by the denominator.
Example 1 Show that
3
8
and
7
12
can each be expressed as a decimal.
Solution:
3
8
means 3 ÷ 8
7
12
means 7 ÷ 12
0.375 0.5833 . . .
8 3.000 12 7.0000
24 60
60 100
56 96
40 40
40 36
0 40
36
4
∴
3
= 0.375
8
∴ 7
12
= 0.5833 . . .
Mathematics Grade 9
14
The fraction (rational number)
3
8
can be expressed as the decimal 0.375. A decimal like
0.375 is called a terminating decimal because the division ends or terminates, when
the remainder is zero.
The fraction
7
12
can be expressed as the decimal 0.58333… (Here, the digit 3 repeats
and the division does not terminate.) A decimal like 0.58333... is called a repeating
decimal. To show a repeating digit or a block of repeating digits in a repeating decimal
number, we put a bar above the repeating digit (or block of digits). For example
0.58333… can be written as 0.583 , and 0.0818181… can be written as 0.081. This
method of writing a repeating decimal is known as bar notation.
The portion of a decimal that repeats is called the repetend. For example,
In 0.583333… =0.583 , the repetend is 3.
In 1.777… = 1.7 , the repetend is 7.
In 0.00454545… = 0.0045 , the repetend is 45.
To generalize:
Any rational number
a
b
can be expressed as a decimal by dividing the numerator
a by the denominator b.
When you divide a by b, one of the following two cases will occur.
Case 1 The division process ends or terminates when a remainder of zero is
obtained. In this case, the decimal is called a terminating decimal.
Case 2 The division process does not terminate as the remainder never
becomes zero. Such a decimal is called a repeating decimal.
Expressing terminating and repeating decimals as
fractions
 Every terminating decimal can be expressed as a fraction (a ratio of two integers)
with a denominator of 10, 100, 1000 and so on.
Example 2 Express each of the following decimals as a fraction in its simplest form
(lowest terms):
a 0.85 b 1.3456
Unit 1 The Number System
15
Solution:
a
100 85 17
0.85 0.85
100 100 20
= × = = (Why?)
b
4
4
10000 10 13456 841
1.3456 1.3456 1.3456
10000 10 10000 625
= × = × = =
 If d is a terminating decimal number that has n digits after a decimal point, then
we rewrite d as
10
10
n
n
d
d
×
=
The right side of the equation gives the fractional form of d.
For example, if d = 2.128, then n = 3.
∴ 2.128 =
3
3
10 2.128 2128 266
= =
10 1000 125
×
 Repeating decimals can also be expressed as fractions (ratios of two integers).
Example 3 Express each of the following decimals as a fraction (ratio of two
integers):
a 0.7 b 0.25
Solution: a Let 0.7 0.777
d = = … then,
10d = 7.777… (multiplying d by 10 because 1 digit repeats)
Subtract d = 0.777… (to eliminate the repeating part 0.777…)
10d = 7.777… 1
1d = 0.777… 2 (d = 1d)
9d = 7 (subtracting expression 2 from expression 1)
∴ d =
7
9
(dividing both sides by 9)
Hence 0.
7
7 =
9
b Let d = 0.25 = 0.252525…
Then, 100d = 25.2525… (multiplying d by 100 because 2 digits repeat)
Mathematics Grade 9
16
100d = 25.252525…
1d = 0.252525…
99d = 25
∴ d =
25
99
So,
25
0.25
99
=
In Example 3a, one digit repeats. So, you multiplied d by 10. In Example 3b, two digits
repeat. So you multiplied d by 100.
The algebra used in the above example can be generalized as follows:
 In general, if d is a repeating decimal with k non-repeating and p repeating digits
after the decimal point, then the formula
( )
10 10
10 10
k p k
k p k
d
d
+
+
−
=
−
is used to change the decimal to the fractional form of d.
Example 4 Express the decimal 0.375 as a fraction.
Solution: Let 0.375
d = , then,
k = 1 (number of non-repeating digits)
p = 2 (number of repeating digits) and
k + p = 1 + 2 = 3.
( )
( )
( )
( )
3 1 3
3
3 1
10 10 10 10 10 10
10 10
10 10 10 10
k p k
k p k
d d d d
d
+
+
− − −
⇒ = = =
−
− −
3
10 0.375 10 0.375
990
× − ×
=
375.75 3.75 372
990 990
−
= =
From Examples 1, 2, 3 and 4, you conclude the following:
i Every rational number can be expressed as either a terminating decimal or a
repeating decimal.
ii Every terminating or repeating decimal represents a rational number.
(subtracting 1d from 100d eliminates the
repeating part 0.2525…)
Unit 1 The Number System
17
Exercise 1.2
1 Express each of the following rational numbers as a decimal:
a
4
9
b
3
25
c
11
7
d
2
5
3
− e
3706
100
f
22
7
2 Write each of the following as a decimal and then as a fraction in its lowest term:
a three tenths b four thousandths
c twelve hundredths d three hundred and sixty nine thousandths.
3 Write each of the following in metres as a fraction and then as a decimal:
a 4 mm b 6 cm and 4 mm c 56 cm and 4 mm
Hint: Recall that 1 metre(m) = 100 centimetres(cm) = 1000 millimetres(mm).
4 From each of the following fractions, identify those that can be expressed as
terminating decimals:
a
5
13
b
7
10
c
69
64
d
11
60
e
11
80
f
17
125
g
5
12
h
4
11
Generalize your observation.
5 Express each of the following decimals as a fraction or mixed number in simplest
form:
a 0.88 b 0.77 c 0.83 d 7.08 e 0.5252 f 1.003
−
6 Express each of the following decimals using bar notation:
a 0.454545… b 0.1345345…
7 Express each of the following decimals without bar notation. (In each case use at
least ten digits after the decimal point)
a 0.13 b 0.305
− c 0.381
8 Verify each of the following computations by converting the decimals to fractions:
a 0.275 0.714 0.989
+ = b 0.6 1.142857 0.476190
− = −
Mathematics Grade 9
18
1.2.2 Irrational Numbers
Remember that terminating or repeating decimals are rational numbers, since they can
be expressed as fractions. The square roots of perfect squares are also rational numbers.
For example, 4 is a rational number since
2
4 = 2 =
1
. Similarly, 0.09 is a rational
number because, 0.09 0.3
= is a rational number.
If x2
= 4, then what do you think is the value of x?
4 2.
x = ± = ± Therefore x is a rational number. What if x2
= 3?
In Figure 1.4 of Section 1.1.3, where do numbers like 2 and 5 fit? Notice what
happens when you find 2 and 5 with your calculator:
If you first press the button 2 and then the square-root
button, you will find 2 on the display.
i.e., 2 : 2 √ = 1.414213562…
5 : 5 √ = 2.236067977…
Note that many scientific calculators, such as Casio ones,
work the same as the written order, i.e., instead of pressing
2 and then the √ button, you press the √ button and then 2.
Before using any calculator, it is always advisable to read
the user’s manual.
Note that the decimal numbers for 2 and 5 do not terminate, nor do they have a
pattern of repeating digits. Therefore, these numbers are not rational numbers. Such
numbers are called irrational numbers. In general, if a is a natural number that is not
a perfect square, then a is an irrational number.
Example 1 Determine whether each of the following numbers is rational or
irrational.
a 0.16666 . . . b 0.16116111611116111116 . . . c π
Solution: a In 0.16666 . . . the decimal has a repeating pattern. It is a
rational number and can be expressed as
1
6
.
b This decimal has a pattern that neither repeats nor terminates. It is an
irrational number.
Study Hint
Most calculators round
answers but some
truncate answers. i.e.,
they cut off at a certain
point, ignoring
subsequent digits.
Unit 1 The Number System
19
c π = 3.1415926… This decimal does not repeat or terminate. It is an
irrational number. (The fraction
22
7
is an approximation to the value of π.
It is not the exact value!).
In Example 1, b and c lead us to the following fact:
 A decimal number that is neither terminating nor repeating is an irrational
number.
1 Locating irrational numbers on the number line
G r o u p W o r k 1 .
G r o u p W o r k 1 .
G r o u p W o r k 1 .
G r o u p W o r k 1 . 4
4
4
4
You will need a compass and straight edge to perform the
following:
1 To locate 2 on the number line:
Draw a number line. At the point corresponding to 1
on the number line, construct a perpendicular line
segment 1 unit long.
Draw a line segment from the point corresponding to
0 to the top of the 1 unit segment and label it as c.
Use the Pythagorean Theorem to show that c is 2 unit long.
Open the compass to the length of c. With the tip of the compass at the point
corresponding to 0, draw an arc that intersects the number line at B. The distance
from the point corresponding to 0 to B is 2 units.
2 To locate 5 on the number line:
Find two numbers whose squares have a sum of 5. One pair that works is 1 and 2,
since 12
+ 22
= 5.
Draw a number line. At the point corresponding
to 2, on the number line, construct a
perpendicular line segment 1 unit long.
Draw the line segment shown
from the point corresponding to 0
to the top of the 1 unit segment.
Label it as c.
0 1 2 3
Figure 1.6
c 1
2
0 1 2
c 1
1
B
Figure 1.5
Mathematics Grade 9
20
The Pythagorean theorem can be used to show that c is 5 units long.
c2
= 12
+ 22
= 5
c = 5
Open the compass to the length of
c. With the tip of the compass at
the point corresponding to 0, draw
an arc intersecting the number line
at B. The distance from the point
corresponding to 0 to B is 5
units.
ACTIVITY 1.6
1 Locate each of the following on the number line, by using
geometrical construction:
a 3 b – 2 c 6
2 Explain how 2 can be used to locate:
a 3 b 6
3 Locate each of the following on the number line:
a 1 + 2 b –2 + 2 c 3 – 2
Example 2 Show that 3 2
+ is an irrational number.
Solution: To show that 3 2
+ is not a rational number, let us begin by assuming
that 3 2
+ is rational. i.e., 3 2
a
b
+ = where a and b are integers, b ≠ 0.
Then
3
2 3 .
a a b
b b
−
= − =
Since a – 3b and b are integers (Why?),
3
a b
b
−
is a rational number, meaning
that
3
2
a b
b
−
 
=  
 
is rational, which is false. As the assumption that 3 2
+ is
rational has led to a false conclusion, the assumption must be false.
Therefore, 3 2
+ is an irrational number.
Definition 1.5 Irrational number
An irrational number is a number that cannot be expressed as
a
b
, such
that a and b are integers and b ≠ 0.
0 1 2 3
5
c 1
B
Figure 1.7
Unit 1 The Number System
21
ACTIVITY 1.7
Evaluate the following:
1 0.3030030003 . . . + 0.1414414441 . . .
2 0.5757757775 . . . – 0.242442444 . . .
3 ( ) ( )
3 + 2 3 2
× −
4 12 3
÷
From Example 2 and Activity 1.7, you can generalize the following facts:
i The sum of any rational number and an irrational number is an irrational number.
ii The set of irrational numbers is not closed with respect to addition, subtraction,
multiplication and division.
iii If p is a positive integer that is not a perfect square, then a + b p is irrational
where a and b are integers and b ≠ 0. For example, 3 2
+ and 2 2 3
− are
irrational numbers.
Exercise 1.3
1 Identify each of the following numbers as rational or irrational:
a
5
6
b 2.34 c 0.1213141516...
−
d 0.81 e 0.121121112... f 5 2
−
g 3
72 h 1 + 3
2 Give two examples of irrational numbers, one in the form of a radical and the
other in the form of a non-terminating decimal.
3 For each of the following, decide whether the statement is 'true' or 'false'. If your
answer is 'false', give a counter example to justify.
a The sum of any two irrational numbers is an irrational number.
b The sum of any two rational numbers is a rational number.
c The sum of any two terminating decimals is a terminating decimal.
d The product of a rational number and an irrational number is irrational.
Mathematics Grade 9
22
1.2.3 Real Numbers
In Section 1.2.1, you observed that every rational number is either a terminating
decimal or a repeating decimal. Conversely, any terminating or repeating decimal is a
rational number. Moreover, in Section 1.2.2 you learned that decimals which are
neither terminating nor repeating exist. For example, 0.1313313331… Such decimals
are defined to be irrational numbers. So a decimal number can be a rational or an
irrational number.
It can be shown that every decimal number, be it rational or irrational, can be associated
with a unique point on the number line and conversely that every point on the number
line can be associated with a unique decimal number, either rational or irrational. This is
usually expressed by saying that there exists a one-to-one correspondence between the
sets C and D where these sets are defined as follows.
C = {P : P is a point on the number line}
D = {d : d is a decimal number }
The above discussion leads us to the following definition.
The set of real numbers and its subsets
are shown in the adjacent diagram.
From the preceding discussion, you can
see that there exists a one-to-one
correspondence between the set R and
the set C = {P:P is a point on the number
line}.
Definition 1.6 Real numbers
A number is called a real number, if and only if it is either a rational
number or an irrational number.
The set of real numbers, denoted by R, can be described as the union of
the sets of rational and irrational numbers.
R = { x : x is a rational number or an irrational number.}
–4 –1
Whole
Numbers
0 1 2
Integers
-2
Rational Numbers
.
.
0 7
0 4
3
5
19
4
−
. . . .
2
1 0100100010
5
π
−
Irrational Numbers
Real Numbers
Figure 1.8
Unit 1 The Number System
23
It is good to understand and appreciate the existence of a one-to-one correspondence
between any two of the following sets.
1 D = {x : x is a decimal number}
2 P = {x : x is a point on the number line}
3 R = {x : x is a real number}
Since all real numbers can be located on the number line, the number line can be used to
compare and order all real numbers. For example, using the number line you can tell
that
3 0, 2 2.
−  
Example 1 Arrange the following numbers in ascending order:
5 3
, 0.8, .
6 2
Solution: Use a calculator to convert
5
6
and
3
2
to decimals
5 ÷ 6 = 0.83333... and
3 2 = 0.866025...
÷
Since 0.8 0.83 0.866025...,
  the numbers when arranged in ascending order are
5 3
0.8, , .
6 2
However, there are algebraic methods of comparing and ordering real numbers.
Here are two important properties of order.
1 Trichotomy property
For any two real numbers a and b, one and only one of the following is true
a  b or a = b or a  b.
2 Transitive property of order
For any three real numbers a, b and c, if a  b and b  c, then, a  c.
Mathematics Grade 9
24
A third property, stated below, can be derived from the Trichotomy Property and the
Transitive Property of Order.
 For any two non-negative real numbers a and b, if a2
 b2
, then a  b.
You can use this property to compare two numbers without using a calculator.
For example, let us compare
5
6
and
3
.
2
2
2
5 25 3 3 27
= , = =
6 36 2 4 36
 
 
 
   
   
Since
2
2
5 3
,
6 2
 
 
  
   
   
it follows that
5 3
 .
6 2
Exercise 1.4
1 Compare the numbers a and b using the symbol  or .
a
6
, 0.6
4
a b
= =
b a = 0.432, b = 0.437
c a = – 0.128, b = – 0.123
2 State whether each set (a – e given below) is closed under each of the following
operations:
i addition ii subtraction iii multiplication iv division
a N the set of natural numbers. b Z the set of integers.
c Q the set of rational numbers. d The set of irrational numbers.
e R the set of real numbers.
1.2.4 Exponents and Radicals
A Roots and radicals
In this subsection, you will define the roots and radicals of numbers and discuss their
properties. Computations of expressions involving radicals and fractional exponents are
also considered.
Unit 1 The Number System
25
Roots
H
H
H I
I
I S
S
S T
T
T O
O
O R
R
R I
I
I C
C
C A
A
A L
L
L N
N
N O
O
O T
T
T E
E
E :
::
The Pythagorean School of ancient Greece focused on the study of philosophy,
mathematics and natural science. The students, called Pythagoreans, made
many advances in these fields. One of their studies was to symbolize numbers.
By drawing pictures of various numbers, patterns can be discovered. For
example, some whole numbers can be represented by drawing dots arranged
in squares.
•
1
1 × 1
• •
• •
4
2 × 2
• • •
• • •
• • •
9
3 × 3
• • • •
• • • •
• • • •
• • • •
16
4 × 4
• • • • • • • •
• • • • • • • •
• • • • • • • •
• • • • • • • •
• • • • • • • •
• • • • • • • •
• • • • • • • •
• • • • • • • •
64
8 × 8
Numbers that can be pictured in squares of dots are called perfect squares or square
numbers. The number of dots in each row or column in the square is a square root of
the perfect square. The perfect square 9 has a square root of 3, because there are 3 rows
and 3 columns. You say 8 is a square root of 64, because 64 = 8 × 8 or 82
.
Perfect squares also include decimals and fractions like 0.09 and
4
.
9
Since ( )
2
0.3 0.09
=
and
2
2 4
=
3 9
 
 
 
, it is also true that (–8)2
= 64 and (–12)2
= 144.
So, you may say that –8 is also a square root of 64 and –12 is a square root of 144.
The positive square root of a number is called the principal square root.
The symbol , called a radical sign, is used to indicate the principal square root.
Definition 1.7 Square root
For any two real numbers a and b, if a2 = b, then a is a square root of b.
Mathematics Grade 9
26
The symbol 25 is read as the principal square root of 25 or just the square root
of 25 and 25
− is read as the negative square root of 25. If b is a positive real
number, b is a positive real number. Negative real numbers do not have square roots in
the set of real numbers since a2
≥ 0 for any number a. The square root of zero is zero.
Similarly, since 43
= 64, you say that 64 is the cube of 4 and 4 is the cube root of 64.
That is written as 3
4 64
= .
The symbol 3
4 64
= is read as the principal cube root of 64 or just the cube root of 64.
 Each real number has exactly one cube root.
(– 3)3
= – 27 so, 3
27 =
− −3 03
= 0 so, 3
0 0.
=
You may now generalize as follows:
Example 1
a – 3 is a cube root of – 27 because (– 3)3
= – 27
b 4 is a cube root of 64 because 43
= 64
i If b  0 and n is even, there is no real nth
root of b, because an even power of any
real number is a non-negative number.
ii The symbol n
is called a radical sign, the expression n
b is called a radical, n
is called the index and b is called the radicand. When no index is written, the
radical sign indicates square root.
Definition 1.9 Principal nth
root
If b is any real number and n is a positive integer greater than 1, then,
the principal nth root of b, denoted by n
b is defined as
the positive root of , if 0.
the negative root of , if 0 and is odd.
0, if 0.
th
th
n
n b b
b n b b n
b
 

= 

 =

Definition 1.8 The nth
root
For any two real numbers a and b, and positive integer n, if an = b, then
a is called an nth
root of b.
Unit 1 The Number System
27
Example 2
a 4
16 = 2 because 24
= 16
b 0.04 = 0.2 because (0. 2)2
= 0.04
c 3
1000 = 10
− − because (– 10)3
= – 1000
Numbers such as 3 3
23, 35 and 10 are irrational numbers and cannot be written as
terminating or repeating decimals. However, it is possible to approximate irrational
numbers as closely as desired using decimals. These rational approximations can be
found through successive trials, using a scientific calculator. The method of successive
trials uses the following property:
 For any three positive real numbers a, b and c and a positive integer n
if an
 b  cn
, then n
a b c
  .
Example 3 Find a rational approximation of 43 to the nearest hundredth.
Solution: Use the above property and divide-and-average on a calculator.
Since 62
= 36  43  49 = 72
6  43  7
Estimate 43 to tenths, 43 6.5
≈
Divide 43 by 6.5
6.615
6.5 43.000
Average the divisor and the quotient
6.5 + 6.615
= 6.558
2
Divide 43 by 6.558
6.557
6.558 43.000
Now you can check that (6.557)2
 43  (6.558)2
. Therefore 43 is between 6.557
and 6.558. It is 6.56 to the nearest hundredth.
Example 4 Through successive trials on a calculator, compute 3
53 to the nearest
tenth.
Mathematics Grade 9
28
Solution:
33
= 27  53  64 = 43
. That is, 33
 53  43
. So 3  3
53  4
Try 3.5: 3.53
= 42.875 So 3.5  3
53  4
Try 3.7: 3.73
= 50.653 So 3.7  3
53  4
Try 3.8: 3.83
= 54.872 So 3.7  3
53  3.8
Try 3.75: 3.753
= 52.734375 So 3.75  3
53  3.8
Therefore, 3
53 is 3.8 to the nearest tenth.
B Meaning of fractional exponents
ACTIVITY 1.8
1 State another name for
1
4
2 .
2 What meaning can you give to
1
2
2 or 0.5
2 ?
3 Show that there is at most one positive number whose fifth root is 2.
By considering a table of powers of 3 and using a calculator, you can define
1
5
3 as 5
3 .
This choice would retain the property of exponents by which
5 1
1 5
5
5
3 3 3.
 
×
 
 
 
= =
 
 
Similarly, you can define
1
5n
, where n is a positive integer greater than 1, as 5.
n
In
general, you can define
1
n
b for any b∈ℝ and n a positive integer to be n
b whenever
n
b is a real number.
Example 5 Write the following in exponential form:
a 7 b 3
1
10
Solution:
a
1
2
7 7
= b
1
3
1
3
3
1 1
10
10
10
−
= =
Definition 1.10 The nth
power
If b ∈ R and n is a positive integer greater than 1, then
1
n
n
b b
=
Unit 1 The Number System
29
Example 6 Simplify:
a
1
2
25 b
1
3
( 8)
− c
1
6
64
Solution:
a
1
2
25 25 5
= = (Since 52
= 25)
b
1
3
3
( 8) 8 2
− = − = − (Since (−2)3
= −8)
c
1
6
6
64 64 2
= = (Since 26
= 64)
G r o u p W o r k 1 .
G r o u p W o r k 1 .
G r o u p W o r k 1 .
G r o u p W o r k 1 . 5
5
5
5
Simplify:
i a ( )
1
3
8 27
× b
1 1
3 3
8 27
×
ii a 3
8 27
× b 3 3
8 27
×
iii a ( )
1
2
36 49
× b
1 1
2 2
36 49
×
iv a 36 49
× b 36 49
×
What relationship do you observe between a and b in i, ii, iii and iv?
The observations from the above Group Work lead you to think that ( )
1 1 1
3 3 3
5 3 5 3
× = × .
This particular case suggests the following general property (Theorem).
Example 7 Simplify each of the following.
a
1 1
3 3
9 3
× b 5
5
16 2
×
Solution:
a ( )
1 1 1
3 3 3
9 3 9 3
× = × (by Theorem 1.2)
= ( )
1
3
27 (multiplication)
= 3 (33
= 27)
b 5
5 5
16 2 16 2
× = ×
= 5
32
= 2
Theorem 1.2
For any two real numbers a and b and for all integers 2
n ≥ , ( )
1 1 1
n n n
a b ab
=
Mathematics Grade 9
30
ACTIVITY 1.9
Simplify:
i a
1
5
1
5
64
2
b
1
5
64
2
 
 
 
ii a
1
2
1
2
8
2
b
1
2
8
2
 
 
 
iii a
1
3
1
3
27
729
b
1
3
27
729
 
 
 
What relationship do you observe between a and b in i, ii and iii?
The observations from the above Activity lead us to the following theorem:
Example 8 Simplify a
1
3
1
16
23
b
6
6
128
2
Solution:
a
1
1
3
16
2
3
=
16
2
1
3
 
 
 
(by Theorem 1.3)
=
1
3
8 = 2 (since 23
= 8)
b
6
6
6
128 128
2
2
=
= 6
64
= 2 (because 26
= 64)
Theorem 1.3
For any two real numbers a and b where b ≠ 0 and for all integers n ≥ 2,
1 1
1
n n
n
a a
b
b
 
=  
 
Unit 1 The Number System
31
ACTIVITY 1.10
1 Suggest, with reasons, a meaning for
a 2
7
2
b 2
9
2
in terms of 2
1
2
2 Suggest a relation between 5
3
2
and 5
1
2 .
Applying the property (am
)n
= amn
, you can write
9
1
10
7
 
 
 
as
9
10
7 . In general, you can say
1 p
q
a
 
 
 
 
=
p
q
a , where p and q are positive integers and a ≥ 0. Thus, you have the following
definition:
Exercise 1.5
1 Show that: a
1
3
64 4
= b
1
8
256 2
= c
1
3
125 5
=
2 Express each of the following without fractional exponents and without radical
signs:
a
1
4
81 b
1
2
9 c 6
64 d
1
3
27
8
 
 
 
e ( )
1
5
0.00032 f 4
0.0016 g 6
729
3 Explain each step of the following:
( ) ( ) ( )
1
1
3
3
27 125 3 3 3 5 5 5
× = × × × × ×
 
  = ( ) ( ) ( )
1
3
3 5 3 5 3 5
× × × × ×
 
 
3 5 15
= × =
4 In the same manner as in Question 3, simplify each of the following:
a ( )
1
2
25 121
× b ( )
1
4
625 16
× c ( )
1
5
1024 243
×
5 Express Theorem 1.2 using radical notation.
6 Show that:
a ( )
1 1 1
4 4 4
7 5 7 5
× = × b 5 3 5 3
× = ×
c
3 3 3
7 9 7 9
× = × d ( )
1 1 1
7 7 7
11 6 11 6
× = ×
Definition 1.11
For a ≥ 0 and p and q any two positive integers, ( )
1
=
p
p
p
q
q q
a a a
 
=
 
 
 
Mathematics Grade 9
32
7 Express in the simplest form:
a
1 1
6 6
32 2
× b
1 1
3 3
9 3
× c
1
1
6
6
1
128
2
 
× 
 
d 5
5
16 2
× e 3
3
16 4
× f
1 1
7 7
32 4
×
g
1
1 1 1
8
8 5 5
1
5 27 9
5
 
× × ×
 
 
h 5
3 5 3
1
5 8 4
5
× × ×
8 Express Theorem 1.3 using radical notation.
9 Simplify:
a
1
5
1
5
128
4
b
1
3
1
3
9
243
c
1
4
1
16
814
d
1
4
1
32
1624
e
3
3
16
2
f
5
5
64
2
g
6
6
512
8
h
3
3
625
5
10 Rewrite each of the following in the form
p
q
a :
a
9
1
5
13
 
 
 
b
11
1
5
12
 
 
 
c
5
1
6
11
 
 
 
11 Rewrite the following in the form
1 p
q
a
 
 
 
 
a
7
5
3 b
6
3
5 c
5
6
64 d
2
3
729
12 Rewrite the expressions in Question 10 using radicals.
13 Rewrite the expressions in Question 11 using radicals.
14 Express the following without fractional exponents or radical sign:
a
5
1
3
27
 
 
 
b
5
3
27 c
1
3
8
15 Simplify each of the following:
a
1
6
64 b
3
2
81 c
3
18
64
d
6
4
81 e
2
3
512 f
6
9
512
Unit 1 The Number System
33
C Simplification of radicals
ACTIVITY 1.11
1 Evaluate each of the following and discuss your result in groups.
a 3
3
( 2)
− b 2
( 3)
− c 4
4
( 5)
−
d 5 5
4 e 2
2 f 7
7
( 1)
−
2 Does the sign of your result depend on whether the index is odd or even?
Can you give a general rule for the result of
n n
a where a is a real number and
i n is an odd integer? ii n is an even integer?
To compute and simplify expressions involving radicals, it is often necessary to
distinguish between roots with odd indices and those with even indices.
For any real number a and a positive integer n,
=
n n
a a , if n is odd.
=
n n
a a , if n is even.
( )
5
5
2 = 2
− − , 3 3
,
x x
= ( )
2
2 = 2 = 2
− −
2
,
x x
= ( )
4
4
2 = 2 = 2
− − ,
4
4
x x
=
Example 9 Simplify each of the following:
a 2
y b 3 3
27x
− c 4
25x d 6
6
x e 3
4
x
Solution:
a
2
y y
= b ( )
3
3 3 3
27 3 3
x x x
− = − = −
c 4 2
25 5
x x
= = 5x2
d
6 6
x x
= e ( )
3
1
3 3
4 4
4
x x x
= =
A radical n
a is in simplest form, if the radicand a contains no factor that can be
expressed as an nth
power. For example 3
54 is not in simplest form because 33
is a
factor of 54.
Using this fact and the radical notations of Theorem 1.2 and Theorem 1.3, you can
simplify radicals.
Mathematics Grade 9
34
Example 10 Simplify each of the following:
a 48 b 3 3
9 3
× c 4
32
81
Solution:
a 48 16 3 16 3 4 3
= × = × =
b 3 3 3 3
9 3 9 3 27 3
× = × = =
c
4
4 4 4
4 4 4
4
32 16 2 16 16 2
2 2 2
81 81 81 3
81
×
= = × = × =
Exercise 1.6
1 Simplify each of the following:
a 8 b 5 32 c 2
3 8 x d 363
e 3
512 f
3 2
1
27
3
x y g 4
405
2 Simplify each of the following if possible. State restrictions where necessary.
a 50 b 2 36 c
1
72
3
d 2
3 8x e
3
a
f 0.27 g 63
− h
180
9
i 3
16 j 3
54
−
3 Identify the error and write the correct solution each of the following cases:
a A student simplified 28 to 25+3 and then to 5 3
b A student simplified 72 to 4 18 and then to 4 3
c A student simplified 9
7x and got 3
7
x
4 Simplify each of the following:
a 8 250 b 3 3
16 5
× c 4 4
5 125
×
d
2
7 14
7
× × e
3
3
81
3
f
12 96
3 6
g
3 2
2 98
0, 0.
14
x y
x y
xy
  h 4 3 2 18
×
Unit 1 The Number System
35
5 The number of units N produced by a company from the use of K units of capital
and L units of labour is given by 12 .
N LK
=
a What is the number of units produced, if there are 625 units of labour and
1024 units of capital?
b Discuss the effect on the production, if the units of labour and capital are
doubled.
Addition and subtraction of radicals
Which of the following do you think is correct?
1 2 + 8 = 10 2 19 3 =4
− 3 5 2 7 2 12 2
+ =
The above problems involve addition and subtraction of radicals. You define below the
concept of like radicals which is commonly used for this purpose.
For example,
i
1
3 5, 5 and 5 are like radicals.
2
− are like radicals.
ii 3
5 and 5 are not like radicals.
iii 11 and 7 are not like radicals.
By treating like radicals as like terms, you can add or subtract like radicals and express
them as a single radical. On the other hand, the sum of unlike radicals cannot be
expressed as a single radical unless they can be transformed into like radicals.
Example 11 Simplify each of the following:
a 2 8
+ b
1 1
3 12 3 2 27
3 9
− + +
Solution:
a 2 8 2 2 4 2 4 2 2 2 2
+ = + × = + = +
(1 2) 2 3 2
= + =
Definition 1.12
Radicals that have the same index and the same radicand are said to be like
radicals.
Mathematics Grade 9
36
b
1 1 1 3 1
3 12 3 2 27 3 4 3 3 2 9 3
3 9 3 3 9
− + + = × − + × + ×
=
3 1
3 4 3 3 2 9 3
9
9
× − + + ×
=
2 1
6 3 3 3 3
3 3
− + +
=
2 1
6 1 3 6 3
3 3
 
− + + =
 
 
Exercise 1.7
Simplify each of the following if possible. State restrictions where necessary.
1 a 2 5
× b 3 6
× c 21 5
× d 2 8
x x
×
e
2
2
f
10
4 3
g 3
50 2
y y
÷ h
9 40
3 10
i 3
3
4 16 2 2
÷ j
9 24 15 75
3 3
÷
2 a 2 3 5 3
+ b 9 2 5 2
− c 3 12
+
d 63 28
− e 75 48
− f ( )
6 12 3
−
g 2 2
2 50
x x
− h 3
3
5 54 2 2
− i
2
8 24 54 2 96
3
+ −
j
2
a ab b
a b
+ +
+
k ( ) 1 1
a b
a b
 
− +
 
 
3 a Find the square of 7 2 10.
−
b Simplify each of the following:
i 5 2 6 5 2 6
+ − − ii
7 24 7 24
2 2
+ −
+
iii ( )( )
2 2 2 2
1 1 1 1
p p p p
+ − − − + +
4 Suppose the braking distance d for a given automobile when it is travelling
v km/hr is approximated by 3 5
0.00021
d v
= m. Approximate the braking distance
when the car is travelling 64 km/hr.
Unit 1 The Number System
37
1.2.5 The Four Operations on Real Numbers
The following activity is designed to help you revise the four operations on the set of
rational numbers which you have done in your previous grades.
ACTIVITY 1.12
1 Apply the properties of the four operations in the set of rational
numbers to compute the following (mentally, if possible).
a
2 3 7
9 5 9
 
+ +
 
 
b
3 11 3 11
7 21 7 21
− − −
    
× +
    
    
c
3 5 3
7 6 7
−
 
+ +
 
 
d
9 23 7
7 27 9
− −
   
× ×
   
−
   
2 State a property that justifies each of the following statements.
a
2 3 3 2 3 3
3 2 5 3 2 5
− −
   
× = × ×
   
   
b
7 3 4 7 4 3
9 2 5 9 5 2
− − − −
   
+ = +
   
   
c
3 5 1 5
,
5 6 5 6
− − − −
       
+  +
       
       
since
3 1
5 5
− −

In this section, you will discuss operations on the set of real numbers. The properties
you have studied so far will help you to investigate many other properties of the set of
real numbers.
G r o u p W o r k 1 .
G r o u p W o r k 1 .
G r o u p W o r k 1 .
G r o u p W o r k 1 . 6
6
6
6
Work with a partner
Required:- scientific calculator
1 Try this
Copy and complete the following table. Then use a calculator to find each product
and complete the table.
Factors product product written as a power
23
× 22
101
× 101
3
1 1
5 5
− −
   
×
   
   
Mathematics Grade 9
38
2 Try this
Copy the following table. Use a calculator to find each quotient and complete the
table.
Division Quotient Quotient written as a power
105
÷ 101
35
÷ 32
4 2
1 1
2 2
   
÷
   
   
Discuss the two tables:
i a Compare the exponents of the factors to the exponents in the product.
What do you observe?
b Write a rule for determining the exponent of the product when you
multiply powers. Check your rule by multiplying 32
× 33
using a
calculator.
ii a Compare the exponents of the division expressions to the exponents in
the quotients. What pattern do you observe?
b Write a rule for determining the exponent in the quotient when you
divide powers. Check your rule by dividing 75
by 73
on a calculator.
3 Indicate whether each statement is false or true. If false, explain:
a Between any two rational numbers, there is always a rational number.
b The set of real numbers is the union of the set of rational numbers and the
set of irrational numbers.
c The set of rational numbers is closed under addition, subtraction,
multiplication and division excluding division by zero.
d The set of irrational numbers is closed under addition, subtraction,
multiplication and division.
4 Give examples to show each of the following:
a The product of two irrational numbers may be rational or irrational.
b The sum of two irrational numbers may be rational or irrational.
c The difference of two irrational numbers may be rational or irrational.
d The quotient of two irrational numbers may be rational or irrational.
5 Demonstrate with an example that the sum of an irrational number and a rational
number is irrational.
6 Demonstrate with an example that the product of an irrational number and a non-
zero rational number is irrational.
Unit 1 The Number System
39
7 Complete the following chart using the words ‘yes’ or ‘no’.
Number
Rational
number
Irrational
number
Real
number
2
3
2
3
−
3
2
1.23
1.20220222…
2
1.23
3
− ×
75 1.23
+
75 − 3
1.20220222… + 0.13113111…
Questions 3, 4, 5 and in particular Question 7 of the above Group Work lead you to
conclude that the set of real numbers is closed under addition, subtraction,
multiplication and division, excluding division by zero.
You recall that the set of rational numbers satisfy the commutative, associative and
distributive laws for addition and multiplication.
If you add, subtract, multiply or divide (except by 0) two rational numbers, you get a
rational number, that is, the set of rational numbers is closed with respect to addition,
subtraction, multiplication and division.
From Group work 1.6 you may have realized that the set of irrational numbers is not closed
under all the four operations, namely addition, subtraction, multiplication and division.
Do the following activity and discuss your results.
ACTIVITY 1.13
1 Find a + b, if
a 3 2 and 3 2
a b
= + = −
b 3 3 and 2 3
a b
= + = +
Mathematics Grade 9
40
2 Find a – b, if
a 3 and 3
a b
= = b 5 and 2
a b
= =
3 Find ab, if
a 3 1 and 3 1
a b
= − = + b 2 3 and 3 2
a b
= =
4 Find a ÷ b, if
a 5 2 and 3 2
a b
= = b 6 6 and 2 3
a b
= =
Let us see some examples of the four operations on real numbers.
Example 1 Add 2 3 3 2
a = + and 2 3 3
−
Solution ( ) ( )
2 3 3 2 2 3 3
+ + − = 2 3 3 2 2 3 3
+ + −
= ( ) ( )
3 2 3 2 3 1
− + +
= 3 4 2
− +
Example 2 Subtract 3 2 5
+ from 3 5 2 2
−
Solution: ( ) ( )
3 5 2 2 3 2 5 3 5 2 2 3 2 5
− − + = − − −
= 5(3 1) 2( 2 3)
− + − −
= 2 5 5 2
−
Example 3 Multiply
a 2 3 by 3 2 b 2 5 by 3 5
Solution:
a 2 3 3 2 6 6
× = b ( )
2
2 5 3 5 2 3 5 30
× = × × =
Example 4 Divide
a 8 6 by 2 3 b ( )
12 6 by 2 3
×
Solution:
a
8 6
8 6 2 3
2 3
÷ =
8 6
4 2
2 3
= × =
b ( ) 12 6
12 6 2 3
2 3
÷ × =
×
12 6
12
6
= =
Unit 1 The Number System
41
Rules of exponents hold for real numbers. That is, if a and b are nonzero numbers and m
and n are real numbers, then whenever the powers are defined, you have the following
laws of exponents.
1 m n m n
a a a +
× = 2 ( )
n
m mn
a a
= 3
m
m n
n
a
a
a
−
=
4 ( )
n
n n
a b ab
× = 5 , 0.
n
n
n
a a
b
b b
 
= ≠
 
 
ACTIVITY 1.14
1 Find the additive inverse of each of the following real numbers:
a 5 b
1
2
− c 2 1
+
d 2.45 e 2.1010010001. . .
2 Find the multiplicative inverse of each of the following real numbers:
a 3 b 5 c 1 3
− d
1
6
2
e 1.71 f
2
3
g 1.3
3 Explain each of the following steps:
( ) ( )
3 3
6 2 15 20 6 2 15 20
3 3
− × + = × − +
3 3
6 2 15 20
3 3
 
= × − × +
 
 
 
18 2 45
= 20
3 3
 
− +
 
 
 
9 2 2 9 5
= 20
3 3
 
× ×
− +
 
 
 
3 2 2 3 5
= 20
3 3
 
× × ×
− +
 
 
 
( )
= 2 2 5 20
− +
( )
= 2 2 5 2 5
 
+ − +
 
= 2
Mathematics Grade 9
42
Let us now examine the basic properties that govern addition and multiplication of real
numbers. You can list these basic properties as follows:
 Closure property:
The set R of real numbers is closed under addition and multiplication. This means that
the sum and product of two real numbers is a real number; that is, for all a, b ∈ R,
a + b ∈ R and ab ∈ R
 Addition and multiplication are commutative in ℝ:
That is, for all a, b ∈ R,
i a + b = b + a
ii ab = ba
 Addition and multiplication are associative in ℝ:
That is, for all, a, b, c ∈ R,
i (a + b) + c = a + ( b + c)
ii (ab)c = a(bc)
 Existence of additive and multiplicative identities:
There are real numbers 0 and 1 such that:
i a + 0 = 0 + a = a, for all a ∈ R.
ii a . 1 = 1 . a = a, for all a ∈ R.
 Existence of additive and multiplicative inverses:
i For each a ∈ R there exists –a ∈ R such that a + (–a) = 0 = (–a) + a, and
–a is called the additive inverse of a.
ii For each non-zero a ∈ R, there exists
1
a
∈ℝ such that
1 1
1 ,
a a
a a
× = = ×
   
   
   
and
1
a
is called the multiplicative inverse or reciprocal of a.
 Distributive property:
Multiplication is distributive over addition; that is, if a, b, c, ∈ R then:
i a ( b + c) = ab + ac
ii (b + c) a = ba + ca
Unit 1 The Number System
43
Exercise 1.8
1 Find the numerical value of each of the following:
a ( ) ( ) ( )
3
4 5 3
1 5 2 2
1
4 2 8 64
16
− −
 
× × × ×
 
 
b 176 2 275 1584 891
− + −
c ( )
3 5 1
15 1.04 5 6 5 0.02 300
5 9 18
− + − −
d 5
4
0.0001 0.00032
− e 3 4
2 0.125 0.0016
+
2 Simplify each of the following
a ( )
1
3
216 b
2 3
3 5
2 2
× c
5
1
2
3
 
 
 
d
3
4
1
4
7
49
e
1
1
8
4
3 25
× f
1
4
16 2
÷ g 4 3
7 h
5
5
32
243
3 What should be added to each of the following numbers to make it a rational
number? (There are many possible answers. In each case, give two answers.)
a 5 3
− b 2 5
− − c 4.383383338…
d 6.123456… e 10.3030003…
1.2.6 Limits of Accuracy
In this subsection, you shall discuss certain concepts such as approximation, accuracy in
measurements, significant figures (s.f), decimal places (d.p) and rounding off numbers.
In addition to this, you shall discuss how to give appropriate upper and lower bounds
for data to a specified accuracy (for example measured lengths).
ACTIVITY 1.15
1 Round off the number 28617 to the nearest
a 10,000 b 1000 c 100
2 Write the number i 7.864 ii 6.437 iii 4.56556555…
a to one decimal place b to two decimal places
3 Write the number 43.25 to
a two significant figures b three significant figures
4 The weight of an object is 5.4 kg.
Give the lower and upper bounds within which the weight of the object can lie.
Mathematics Grade 9
44
1 Counting and measuring
Counting and measuring are an integral part of our daily life. Most of us do so for
various reasons and at various occasions. For example you can count the money you
receive from someone, a tailor measures the length of the shirt he/she makes for us, and
a carpenter counts the number of screws required to make a desk.
Counting: The process of counting involves finding out the exact number of things.
For example, you do counting to find out the number of students in a class. The answer
is an exact number and is either correct or, if you have made a mistake, incorrect. On
many occasions, just an estimate is sufficient and the exact number is not required or
important.
Measuring: If you are finding the length of a football field, the weight of a person
or the time it takes to walk down to school, you are measuring. The answers are not
exact numbers because there could be errors in measurements.
2 Estimation
In many instances, exact numbers are not necessary or even desirable. In those
conditions, approximations are given. The approximations can take several forms.
Here you shall deal with the common types of approximations.
A Rounding
If 38,518 people attend a football game this figure can be reported to various levels of
accuracy.
To the nearest 10,000 this figure would be rounded up to 40,000.
To the nearest 1000 this figure would be rounded up to 39,000.
To the nearest 100 this figure would be rounded down to 38,500
In this type of situation, it is unlikely that the exact number would be reported.
B Decimal places
A number can also be approximated to a given number of decimal places (d.p). This
refers to the number of figures written after a decimal point.
Example 1
a Write 7.864 to 1 d.p. b Write 5.574 to 2 d.p.
Solution:
a The answer needs to be written with one number after the decimal point.
However, to do this, the second number after the decimal point also needs to
be considered. If it is 5 or more, then the first number is rounded up.
That is 7.864 is written as 7.9 to 1 d.p
Unit 1 The Number System
45
b The answer here is to be given with two numbers after the decimal point. In
this case, the third number after the decimal point needs to be considered.
As the third number after the decimal point is less than 5, the second
number is not rounded up.
That is 5.574 is written as 5.57 to 2 d.p.
Note that to approximate a number to 1 d.p means to approximate the number to the
nearest tenth. Similarly approximating a number to 2 decimal places means to
approximate to the nearest hundredth.
C Significant figures
Numbers can also be approximated to a given number of significant figures (s.f). In the
number 43.25 the 4 is the most significant figure as it has a value of 40. In contrast, the
5 is the least significant as it only has a value of 5 hundredths. When we desire to use
significant figures to indicate the accuracy of approximation, we count the number of
digits in the number from left to right, beginning at the first non-zero digit. This is
known as the number of significant figures.
Example 2
a Write 43.25 to 3 s.f. b Write 0.0043 to 1 s.f.
Solution:
a We want to write only the three most significant digits. However, the fourth
digit needs to be considered to see whether the third digit is to be rounded
up or not.
That is, 43.25 is written as 43.3 to 3 s.f.
b Notice that in this case 4 and 3 are the only significant digits. The number 4
is the most significant digit and is therefore the only one of the two to be
written in the answer.
That is 0.0043 is written as 0.004 to 1 s.f.
3 Accuracy
In the previous lesson, you have studied that numbers can be approximated:
a by rounding up
b by writing to a given number of decimal place and
c by expressing to a given number of significant figure.
In this lesson, you will learn how to give appropriate upper and lower bounds for data
to a specified accuracy (for example, numbers rounded off or numbers expressed to a
given number of significant figures).
Mathematics Grade 9
46
Numbers can be written to different degrees of accuracy.
For example, although 2.5, 2.50 and 2.500 may appear to represent the same number,
they actually do not. This is because they are written to different degrees of accuracy.
2.5 is rounded to one decimal place (or to the nearest tenths) and therefore any number
from 2.45 up to but not including 2.55 would be rounded to 2.5. On the number line this
would be represented as
As an inequality, it would be expressed as
2.45 ≤ 2.5  2.55
2.45 is known as the lower bound of 2.5, while
2.55 is known as the upper bound.
2.50 on the other hand is written to two decimal places and therefore only numbers from
2.495 up to but not including 2.505 would be rounded to 2.50. This, therefore,
represents a much smaller range of numbers than that being rounded to 2.5. Similarly,
the range of numbers being rounded to 2.500 would be even smaller.
Example 3 A girl’s height is given as 162 cm to the nearest centimetre.
i Work out the lower and upper bounds within which her height can lie.
ii Represent this range of numbers on a number line.
iii If the girl’s height is h cm, express this range as an inequality.
Solution:
i 162 cm is rounded to the nearest centimetre and therefore any measurement
of cm from 161.5 cm up to and not including 162.5 cm would be rounded to
162 cm.
Thus,
lower bound = 161.5 cm
upper bound = 162.5 cm
ii Range of numbers on the number line is represented as
iii When the girl’s height h cm is expressed as an inequality, it is given by
161.5 ≤ h  162.5.
161 161.5 162 162.5 163
2.45
2.4 2.5 2.55 2.6
Unit 1 The Number System
47
Effect of approximated numbers on calculations
When approximated numbers are added, subtracted and multiplied, their sums,
differences and products give a range of possible answers.
Example 4 The length and width of a rectangle are 6.7 cm and 4.4 cm, respectively.
Find their sum.
Solution: If the length l = 6.7 cm and the width w = 4.4 cm
Then 6.65 ≤ l  6.75 and 4.35 ≤ w  4.45
The lower bound of the sum is obtained by adding the two lower bounds.
Therefore, the minimum sum is 6.65 + 4.35 that is 11.00.
The upper bound of the sum is obtained by adding the two upper bounds.
Therefore, the maximum sum is 6.75 + 4.45 that is 11.20.
So, the sum lies between 11.00 cm and 11.20 cm.
Example 5 Find the lower and upper bounds for the following product, given that
each number is given to 1 decimal place.
3.4 × 7.6
Solution:
If x = 3.4 and y = 7.6 then 3.35 ≤ x  3.45 and 7.55 ≤ y  7.65
The lower bound of the product is obtained by multiplying the two lower bounds.
Therefore, the minimum product is 3.35 × 7.55 that is 25.2925
The upper bound of the product is obtained by multiplying the two upper bounds.
Therefore, the maximum product is 3.45 × 7.65 that is 26.3925.
So the product lies between 25.2925 and 26.3925.
Example 6 Calculate the upper and lower bounds to
54.5
,
36.0
given that each of the
numbers is accurate to 1 decimal place.
Solution: 54.5 lies in the range 54.45 ≤ x  54.55
36.0 lies in the range 35.95 ≤ x  36.05
The lower bound of the calculation is obtained by dividing the lower bound of the
numerator by the upper bound of the denominator.
So, the minimum value is 54.45÷36.05. i.e., 1.51 (2 decimal places).
The upper bound of the calculation is obtained by dividing the upper bound of the
numerator by the lower bound of the denominator.
So, the maximum value is 54.55 ÷35.95. i.e., 1.52 (2 decimal places).
Mathematics Grade 9
48
Exercise 1.9
1 Round the following numbers to the nearest 1000.
a 6856 b 74245 c 89000 d 99500
2 Round the following numbers to the nearest 100.
a 78540 b 950 c 14099 d 2984
3 Round the following numbers to the nearest 10.
a 485 b 692 c 8847 d 4 e 83
4 i Give the following to 1 d.p.
a 5.58 b 4.04 c 157.39 d 15.045
ii Round the following to the nearest tenth.
a 157.39 b 12.049 c 0.98 d 2.95
iii Give the following to 2 d.p.
a 6.473 b 9.587 c 0.014 d 99.996
iv Round the following to the nearest hundredth.
a 16.476 b 3.0037 c 9.3048 d 12.049
5 Write each of the following to the number of significant figures indicated in
brackets.
a 48599 (1 s.f) b 48599 (3 s.f) c 2.5728 (3 s.f)
d 2045 (2 s.f) e 0.08562 (1 s.f) f 0.954 (2 s.f)
g 0.00305 (2 s.f) h 0.954 (1 s.f)
6 Each of the following numbers is expressed to the nearest whole number.
i Give the upper and lower bounds of each.
ii Using x as the number, express the range in which the number lies as an
inequality.
a 6 b 83 c 151 d 1000
7 Each of the following numbers is correct to one decimal place.
i Give the upper and lower bounds of each.
ii Using x as the number, express the range in which the number lies as an
inequality.
a 3.8 b 15.6 c 1.0 d 0.3 e –0.2
8 Each of the following numbers is correct to two significant figures.
i Give the upper and lower bounds of each.
ii Using x as the number, express the range in which the number lies as an
inequality.
a 4.2 b 0.84 c 420 d 5000 e 0.045
Unit 1 The Number System
49
9 Calculate the upper and lower bounds for the following calculations, if each of
the numbers is given to 1 decimal place.
a 9.5 ×7.6 b 11.0 × 15.6 c
46.5
32.0
d
25.4
8.2
e
4.9 6.4
2.6
+
10 The mass of a sack of vegetables is given as 5.4 kg.
a Illustrate the lower and upper bounds of the mass on a number line.
b Using M kg for the mass, express the range of values in which it must lie, as
an inequality.
11 The masses to the nearest 0.5 kg of two parcels are 1.5 kg and 2.5 kg. Calculate
the lower and upper bounds of their combined mass.
12 Calculate upper and lower bounds for the perimeter of a school football field
shown, if its dimensions are correct to 1 decimal place.
109.7 m
48.8 m
Figure 1.9
13 Calculate upper and lower bounds for the length marked x cm in the rectangle
shown. The area and length are both given to 1 decimal place.
x
8.4 cm Area = 223.2 cm2
Figure 1.10
1.2.7 Scientific Notation (Standard form)
In science and technology, it is usual to see very large and very small numbers. For
instance:
The area of the African continent is about 30,000,000 km2
.
The diameter of a human cell is about 0.0000002 m.
Very large numbers and very small numbers may sometimes be difficult to work with or
write. Hence you often write very large or very small numbers in scientific notation,
also called standard form.
Mathematics Grade 9
50
Example 1 6
1.86 10
× -
is written in scientific notation.
4 3
8.735 10 and 7.08 10
× × -
are written in scientific notation.
1
14.73 10 , 0.0863 10
× ×
- 4
and 3.864
are not written in standard form (scientific
notation).
ACTIVITY 1.16
1 By what powers of 10 must you multiply 1.3 to get:
a 13? b 130? c 1300?
Copy and complete this table.
13 = 1.3 × 101
130 = 1.3 × 102
1,300 = 1.3 ×
13,000 =
1,300,000 =
2 Can you write numbers between 0 and 1 in scientific notation, for example
0.00013?
Copy and complete the following table.
13.0 = 1.3 × 10 = 1.3 × 101
1.3 = 1.3 × 1 = 1.3 × 100
0.13 = 1.3 ×
1
10
= 1.3 × 10−1
0.013 = 1.3 ×
1
100
=
0.0013 =
0.00013 =
0.000013 =
0.0000013 =
Note that if n is a positive integer, multiplying a number by 10n
moves its decimal point n
places to the right, and multiplying it by 10−n
moves the decimal point n places to the left.
Times 10 to
a power.
Number from 1 up to
but not including 10.
Unit 1 The Number System
51
Example 2 Express each of the following numbers in scientific notation:
a 243, 900,000 b 0.000000595
Solution:
a 243,900,000 = 2.439 × 108
.
The decimal point moves 8 places to the left.
b 0.000000595 = 5.95 × 10−7
.
The decimal point moves 7 places to the right.
Example 3 Express 2.483 × 105
in ordinary decimal notation.
Solution: 2.483 × 105
= 2.483 × 100,000 = 248,300.
Example 4 The diameter of a red blood cell is about 7.4 × 10–4
cm. Write this
diameter in ordinary decimal notation.
Solution: 7.4 × 10–4
= 7.4 × 4
1
10
= 7.4 ×
1
10,000
= 7.4 × 0.0001 = 0.00074.
So, the diameter of a red blood cell is about 0.00074 cm.
Calculators and computers also use scientific notation to display large numbers and
small numbers but sometimes only the exponent of 10 is shown. Calculators use a space
before the exponent, while computers use the letter E.
 The calculator display 5.23 06 means 5.23 × 106
. (5,230,000).
The following example shows how to enter a number with too many digits to fit on the
display screen into a calculator.
Example 5 Enter 0.00000000627 into a calculator.
Solution: First, write the number in scientific notation.
0.00000000627 = 6.27 × 10−9
Then, enter the number.
6.27 exp 9 /
+ − giving 6.27 – 09
Decimal
notation
Scientific
notation
Calculator
display
Computer
display
250,000 2.5 × 105
2.5 0.5 2.5 E + 5
0.00047 4.7 × 10−4
4.7 – 04 4.7 E – 4
Definition 1.13
A number is said to be in scientific notation (or standard form), if it is
written as a product of the form
a × 10k
where 1 ≤ a  10 and k is an integer.
Mathematics Grade 9
52
Exercise 1.10
1 Express each of the following numbers in scientific notation:
a 0.00767 b 5,750,000,000 c 0.00083
d 400,400 e 0.054
2 Express each of the following numbers in ordinary decimal notation:
a 4.882 × 105
b 1.19 × 10–5
c 2.021 × 102
3 Express the diameter of an electron which is about 0.0000000000004 cm in
scientific notation.
1.2.8 Rationalization
ACTIVITY 1.17
Find an approximate value, to two decimal places, for the following:
i
1
2
ii
2
2
In calculating this, the first step is to find an approximation of 2 in a reference book
or other reference material. (It is 1.414214.) In the calculation of
1
,
2
1 is divided by
1.414214… which is a difficult task. However, evaluating
2
2
as
1.414214
0.707107
2
≈
is easy.
Since
1
2
is equivalent to
2
2
(How?), you see that in order to evaluate an expression
with a radical in the denominator, first you should transform the expression into an
equivalent expression with a rational number in the denominator.
The technique of transferring the radical expression from the denominator to the
numerator is called rationalizing the denominator (changing the denominator into a
rational number).
The number that can be used as a multiplier to rationalize the denominator is called the
rationalizing factor. This is equivalent to 1.
Unit 1 The Number System
53
For instance, if n is an irrational number then
1
n
can be rationalized by multiplying
it by 1.
n
n
= So,
n
n
is the rationalizing factor.
Example 1 Rationalize the denominator in each of the following:
a
5 3
8 5
b
6
3
c 3
3
2
Solution:
a The rationalizing factor is
5
.
5
So,
5 3 5 3 5 5 15
= =
8 5 8 5 5 8 25
×
2
5 15 5 15 15
= = =
8 5 8
8 5 ×
b The rationalizing factor is
3
3
So,
2
6 6 3 6 3 6 3
= = = = 2 3
3
3 3 3 3
×
c The rationalizing factor is
3 2
3 2
2
2
because 3 3 3
2 4 8 2
× = =
So,
3 2 3 3
3 3 3 3
2 3
3 3 2 3 4 3 4
= . = =
2
2 2 2 2
If a radicand itself is a fraction
2
forexample
3
 
 
 
 
, then, it can be written in the
equivalent form
2
3
so that the procedure described above can be applied to rationalize
the denominator. Therefore,
2
2 2 2 3 6 6 6
= = = = =
3 3
3 3 3 9 3
×
In general,
For any non-negative integers a, b (b ≠ 0)
a a a b ab
b b
b b b
= = = .
Mathematics Grade 9
54
Exercise 1.11
Simplify each of the following. State restrictions where necessary. In each case, state
the rationalizing factor you use and express the final result with a rational denominator
in its lowest term.
a
2
2
b
2
6
c
5 2
4 10
d
12
27
e
5
18
f 3
3
2 3
g 3
1
4
h 2
9
a
i
3
3
20
4
j
4
5
More on rationalizations of denominators
ACTIVITY 1.18
Find the product of each of the following:
1 ( )( )
2 3 2 3
+ − 2 ( )( )
5 3 2 5 3 2
+ −
3
1 1
5 3 5 3
2 2
  
− +
  
  
You might have observed that the results of all of the above products are rational
numbers.
This leads you to the following conclusion:
Using the fact that
(a – b) (a + b) = a2
– b2
,
you can rationalize the denominators of expressions such as
1 1 1
, ,
a b a b a b
+ − −
where ,
a b are irrational numbers as follows.
i
( ) ( )
2 2
2
1 1 a b a b a b
a b
a b a b
a b a b
 
− − −
= = =
 
  −
+ −
+   −
ii
( )
2 2
2
1 1 a b a b a b
a b
a b a b a b a b
 
+ + +
= = =
 
  −
− − +
  −
iii
( ) ( ) ( )
2 2
1 1 a b a b a b
a b
a b a b
a b a b
 
+ + +
= = =
 
  −
− +
−   −
Unit 1 The Number System
55
Example 2 Rationalize the denominator of each of the following:
a
5
1 2
−
b
3
6 3 2
+
Solution:
a The rationalizing factor is
1+ 2
1 + 2
So
( )
2
2
5 5(1 2) 5 5 2
1 2 (1 2)(1 2) 1 2
+ +
= =
− − + −
5 + 5 2
= = 5 5 2
1 2
− −
−
b The rationalizing factor is
6 3 2
6 3 2
−
−
So
( )
( )
( ) ( )
2 2
3 6 3 2
3 3 6 3 2
= =
6 + 3 2 6 3 2
6 + 3 2 6 3 2
−
−
− −
( )
3 6 3 2
=
6 18
−
−
= ( )
1
6 3 2
4
− −
3 2 6
4
−
=
Exercise 1.12
Rationalize the denominator of each of the following:
a
1
3 5
−
b
18
5 3
−
c
2
5 3
−
d
3 4
3 2
+
−
e
10
7 2
−
f
3 2 + 3
3 2 2 3
−
g
1
2 3 1
+ −
Mathematics Grade 9
56
1.2.9 Euclid's Division Algorithm
A The division algorithm
ACTIVITY 1.19
1 Is the set of non-negative integers (whole numbers) closed under
division?
2 Consider any two non-negative integers a and b.
a What does the statement a is a multiple of b mean?
b Is it always possible to find a non-negative integer c such that a = bc?
If a and b are any two non-negative integers, then a ÷ b (b ≠ 0) is some non-negative integer
c (if it exists) such that a = bc. However, since the set of non-negative integers is not closed
under division, it is clear that exact division is not possible for every pair of non-negative
integers.
For example, it is not possible to compute 17 ÷ 5 in the set of non-negative integers, as
17 ÷ 5 is not a non-negative integer.
15 = 3 × 5 and 20 = 4 × 5. Since there is no non-negative integer between 3 and 4, and
since 17 lies between 15 and 20, you conclude that there is no non-negative integer c
such that 17 = c × 5.
You observe, however, that by adding 2 to each side of the equation 15 = 3 × 5 you can
express it as 17 = (3 × 5) + 2. Furthermore, such an equation is useful. For instance it
will provide a correct answer to a problem such as: If 5 girls have Birr 17 to share, how
many Birr will each girl get? Examples of this sort lead to the following theorem called
the Division Algorithm.
In the theorem, a is called the dividend, q is called the quotient, b is called the
divisor, and r is called the remainder.
Example 1 Write a in the form b × q + r where 0 ≤ r  b,
a If a = 47 and b = 7 b If a = 111 and b = 3 c If a = 5 and b = 8
Theorem 1.4 Division algorithm
Let a and b be two non-negative integers and b ≠ 0, then there exist
unique non-negative integers q and r, such that,
a = (q × b) + r with 0 ≤ r  b.
Unit 1 The Number System
57
Solution:
a 6 b 37 c 0
7 47 3 111 8 5
42 9 0
5 21 5
q = 6 and r = 5 21 q = 0 and r = 5
∴ 47 = 7 (6) + 5 0 ∴ 5 = 8 (0) + 5.
q = 37 and r = 0
∴ 111 = 3 (37) + 0
Exercise 1.13
For each of the following pairs of numbers, let a be the first number of the pair and b
the second number. Find q and r for each pair such that a = b × q + r, where 0 ≤ r  b:
a 72, 11 b 16, 9 c 11, 18
d 106, 13 e 176, 21 f 25, 39
B The Euclidean algorithm
ACTIVITY 1.20
Given two numbers 60 and 36
1 Find GCF (60, 36).
2 Divide 60 by 36 and find the GCF of 36 and the remainder.
3 Divide 36 by the remainder you got in Step 2. Then, find the GCF of the two
remainders, that is, the remainder you got in Step 2 and the one you got in step 3.
4 Compare the three GCFs you got.
5 Generalize your results.
The above Activity leads you to another method for finding the GCF of two numbers,
which is called Euclidean algorithm. We state this algorithm as a theorem.
Theorem 1.5 Euclidean algorithm
If a, b, q and r are positive integers such that
a = q × b + r, then, GCF (a, b) = GCF (b, r).
Mathematics Grade 9
58
Example 2 Find GCF (224, 84).
Solution: To find GCF (224, 84), you first divide 224 by 84. The divisor and
remainder of this division are then used as dividend and divisor,
respectively, in a succeeding division. The process is repeated until a
remainder 0 is obtained.
The complete process to find GCF (224, 84) is shown below.
Euclidean algorithm
Computation Division algorithm
form
Application of Euclidean
Algorithm
2
84 224
168
56
1
56 84
56
28
2
28 56
56
0
224 = (2 × 84) + 56
84 = (1 × 56) + 28
56 = (2 × 28) + 0
GCF (224, 84) = GCF (84, 56)
GCF (84, 56) = GCF (56, 28)
GCF (56, 28) = 28 (by inspection)
Conclusion GCF (224, 84) = 28.
Exercise 1.14
1 For the above example, verify directly that
GCF (224, 84) = GCF (84, 56) = GCF (56, 28).
2 Find the GCF of each of the following pairs of numbers by using the Euclidean
Algorithm:
a 18; 12 b 269; 88 c 143; 39
d 1295; 407 e 85; 68 f 7286; 1684
Unit 1 The Number System
59
Key Terms
bar notation principal nth
root
composite number principal square root
divisible radical sign
division algorithm radicand
factor rational number
fundamental theorem of arithmetic rationalization
greatest common factor (GCF) real number
irrational number repeating decimal
least common multiple (LCM) repetend
multiple scientific notation
perfect square significant digits
prime factorization significant figures
prime number terminating decimal
Summary
Summary
1 The sets of Natural numbers, Whole numbers, Integers and Rational numbers
denoted by ℕ, W, ℤ, and ℚ, respectively are described by
ℕ = {1, 2, 3,…} W = {0, 1, 2,…} ℤ = {…,−3, −2, −1, 0, 1, 2, 3,…}
: , , 0
a
a b b
b
 
= ∈ ∈ ≠
 
 
ℚ ℤ ℤ
2 a A composite number is a natural number that has more than two factors.
b A prime number is a natural number that has exactly two distinct factors, 1
and itself.
c Prime numbers that differ by two are called twin primes.
d When a natural number is expressed as a product of factors that are all
prime, then the expression is called the prime factorization of the number.
Mathematics Grade 9
60
e Fundamental theorem of arithmetic.
Every composite number can be expressed (factorized) as a product of
primes, and this factorization is unique, apart from the order in which the
prime factors occur.
3 a The greatest common factor (GCF) of two or more numbers is the
greatest factor that is common to all numbers.
b The least common multiple (LCM) of two or more numbers is the smallest
or least of the common multiples of the numbers.
4 a Any rational number can be expressed as a repeating decimal or a
terminating decimal.
b Any terminating decimal or repeating decimal is a rational number.
5 Irrational numbers are decimal numbers that neither repeat nor terminate.
6 The set of real numbers denoted by ℝ is defined by
ℝ = {x: x is rational or x is irrational}
7 The set of irrational numbers is not closed under addition, subtraction,
multiplication and division.
8 The sum of an irrational and a rational number is always an irrational number.
9 For any real number b and positive integer n  1
1
n
n
b b
= (Whenever n
b is a real number)
10 For all real numbers a and b ≠ 0 for which the radicals are defined and for all
integers n ≥ 2:
i n n n
ab a b
= ii
n
n
n
a a
b b
=
11 A number is said to be written in scientific notation (standard notation), if it is
written in the form a × 10k
where 1 ≤ a  10 and k is an integer.
12 Let a and b be two non-negative integers and b ≠ 0, then there exist unique non-
negative integers q and r such that a = (q × b) + r with 0 ≤ r  b.
13 If a, b, q and r are positive integers such that a = q × b + r, then
GCF (a, b) = GCF (b, r).
Unit 1 The Number System
61
Review Exercises on Unit 1
1 Determine whether each of the following numbers is divisible by 2, 3, 4, 5, 6, 8, 9
or 10:
a 533 b 4, 299 c 111
2 Find the prime factorization of:
a 150 b 202 c 63
3 Find the GCF for each set of numbers given below:
a 16; 64 b 160; 320; 480
4 Express each of the following fractions or mixed numbers as a decimal:
a
5
8
b
16
33
c
4
5
9
d
1
3
7
5 Express each of the following decimals as a fraction or mixed number in its
simplest form:
a 0.65 b – 0.075 c 0.16 d 24.54
− e 0.02
−
6 Arrange each of the following sets of rational numbers in increasing order:
a
71 3 23
, ,
100 2 30
− b 3.2,3.22,3.23,3.23
c
2 11 16 67
, , ,
3 18 27 100
7 Write each of the following expressions in its simplest form:
a 180 b
169
196
c 3
250 d 2 3 3 2 180
+ +
8 Give equivalent expression, containing fractional exponents, for each of the
following:
a 15 b a b
+ c 3 x y
− d 4
13
16
9 Express the following numbers as fractions with rational denominators:
a
1
2 +1
b
5
3
c
5
3 7
−
+
d 4
13
16
Mathematics Grade 9
62
10 Simplify
a ( ) ( )
3 7 2 7 12
+ + − b ( ) ( )
2 5 2 5
+ + −
c 2 6 3 54
÷ d ( )
2 3 7 2 7
+ −
11 If 5 2.236 and 10 3.162
≈ ≈ , find the value of 10 20 40 5 80
+ + − −
12 If
2 3
6 ,
3 2 2 3
x y
+
= +
−
find the values of x and y.
13 Express each of the following numbers in scientific notation:
a 7,410,00 b 0.0000648 c 0.002056 d 12.4 × 10–6
14 Simplify each of the following and give the answer in scientific notation:
a 109
× 10– 6
× 27 b
4 2
7
796 10 10
10
−
−
× ×
c 0.00032 × 0.002
15 The formula 3.56
d h
= km estimates the distance a person can see to the horizon,
where h is the height of the eyes of the person from the ground in metre. Suppose
you are in a building such that your eye level is 20 m above the ground. Estimate
how far you can see to the horizon.
h = 20 m
d
Figure 1.11
Unit Outcomes:
After completing this unit, you should be able to:
identify equations involving exponents and radicals, systems of two linear
equations, equations involving absolute values and quadratic equations.
solve each of these equations.
Main Contents
2.1 Equations involving exponents and radicals
2.2 Systems of linear equations in two variables
2.3 Equations involving absolute value
2.4 Quadratic equations
Key Terms
Summary
Review Exercises
U
Un
ni
it
t
S
S
S
S
S
S
S
SO
O
O
O
O
O
O
OL
L
L
L
L
L
L
LU
U
U
U
U
U
U
UT
T
T
T
T
T
T
TI
I
I
I
I
I
I
IO
O
O
O
O
O
O
ON
N
N
N
N
N
N
N O
O
O
O
O
O
O
OF
F
F
F
F
F
F
F E
E
E
E
E
E
E
EQ
Q
Q
Q
Q
Q
Q
QU
U
U
U
U
U
U
UA
A
A
A
A
A
A
AT
T
T
T
T
T
T
TI
I
I
I
I
I
I
IO
O
O
O
O
O
O
ON
N
N
N
N
N
N
N
Mathematics Grade 9
64
INTRODUCTION
In earlier grades, you have learnt
also learned about linear equations in one variable and the methods to solve them. In the
present unit, we discuss further about equations involving exponents, radicals, and
absolute values. You shall also learn about systems of linear equations in two variables,
quadratic equations in single variable, and the methods to solve them.
2.1 EQUATIONS INVOLVING EXPONENTS
AND RADICALS
Equations are equality of expressions. There are different types of equations that depend
on the variable(s) considered. When the variable in use
is said to be an equation involving exponents.
1 Determine whether or not each of the following is true.
a 24
× 25
= 220
d 2n
× 22
= 22n
2 Express each of the following
a 8 b
The above Activity leads you to revise the rules of exponents
Example 1 Solve each of the following equations.
a 3
x =
Solution:
a 3
x =
2 2
3
x =
x = 9
Therefore x = 9.
b To solve x3
= 8
the nth
root of b
x3
= 8
3
8 2
x = =
INTRODUCTION
you have learnt about algebraic equations and their classification.
about linear equations in one variable and the methods to solve them. In the
present unit, we discuss further about equations involving exponents, radicals, and
shall also learn about systems of linear equations in two variables,
ratic equations in single variable, and the methods to solve them.
EQUATIONS INVOLVING EXPONENTS
AND RADICALS
Equations are equality of expressions. There are different types of equations that depend
on the variable(s) considered. When the variable in use has an exponent
is said to be an equation involving exponents.
ACTIVITY 2.1
er or not each of the following is true.
b (32
)3
= 36
c ( )
1
2 2
5 5
=
e 2x
= 8 is equivalent to x = 3.
following numbers in power form.
27 c 625 d 343
leads you to revise the rules of exponents that you discussed in
Solve each of the following equations.
b 3
8
x = c 2 16
x
=
Squaring both sides
= 8, recall that for any real number a and b, if an
b.
8 2
= =
about algebraic equations and their classification. You
about linear equations in one variable and the methods to solve them. In the
present unit, we discuss further about equations involving exponents, radicals, and
shall also learn about systems of linear equations in two variables,
EQUATIONS INVOLVING EXPONENTS
Equations are equality of expressions. There are different types of equations that depend
exponent other than 1, it
that you discussed in Unit 1.
n
= b, then a is
c To solve 2 16
x
=
In 2x
= 24
, the bases are equal and
Therefore x = 4 is the solution.
Solve each of the following equations
a 2 2
8 2
x x+
=
The following rule is very useful in solving such equations.
Rule: For a  0, ax
= a
Example 2 Solve 2 1 2
3 3
x x
+ −
Solution: By using the rule, since 3  0,
2x + 1 = x – 2. From this we can see that the solution is
Example 3 Solve each of the following equations
a 2 1
8 2
x x+
=
Solution:
a 2 1
8 2
x x+
=
( )
3 2 1
2 2
x x+
=
3 2 1
2 2
x x+
=
3x = 2x + 1
Therefore x = 1.
b 3 3
9 27
x x
−
=
( ) ( )
3 3
2 3
3 3
x x
−
=
( ) ( )
2 3 3 3
3 3
x x
−
=
2 6 9
3 3
x x
−
=
2 6 9
x x
− =
7 6
x = −
Therefore
6
.
7
x = −
c 3 2 5
3 3
x x+
=
( )
1
2 5
3
3 3
x x+
=
Unit 2 Solutions of Equations
2 16
= , first express 4
2 16 as 2 2
x x
= = .
, the bases are equal and hence the exponents must be equal.
= 4 is the solution.
ACTIVITY 2.2
following equations.
b 1
4 2
x x
+
= c 2
5 25 x
=
The following rule is very useful in solving such equations.
ay
, if and only if x = y.
2 1 2
3 3
x x
+ −
=
By using the rule, since 3  0, 2 1 2
3 3
x x
+ −
= , if and only if the exponents
From this we can see that the solution is x = –3.
Solve each of the following equations.
b 3 3
9 27
x x
−
= c 3 2 5
3 3
x x+
=
Expressing 8 as a power of 2
Applying laws of exponents
x x
3 3
x x
Expressing 9 and 27 as powers of 3
)
x x
Applying laws of exponents
Applying laws of exponents
Solutions of Equations
65
the exponents must be equal.
if and only if the exponents
Mathematics Grade 9
66
2 5
3
3 3
x
x+
=
2 5
3
x
x
= +
(
3 2 5
= +
x x
x = 6x + 15
−5x = 15
Therefore x = – 3.
1 Solve each of the following equations
a 3 27
x
=
d 5 2 1
81
243
x+
=
g 3
(3 1) 64
x + =
2 Solve ( ) (
2 2
2 3 3 1 .
x x
+ = −
3 Solve each of the following equations
a 2 1 2 1
9 27 81
x x x
− +
=
c 3 4 3 4 1
16 2 64
x x x
+ − +
=
2.2 SYSTEMS OF LINEAR EQUATIONS IN
TWO VARIABLES
Recall that, for real numbers
called a linear equation. The numbers
1 Solve each of the following linear equations
a x – 2 = 7
c 2x = 4
2 How many solutions do you
2 5
)
3 2 5
= +
+ 15
Exercise 2.1
Solve each of the following equations.
b
1
16
4
x
 
=
 
 
c
 
 
 
e 2 2 1
9 27
x x+
= f 16 2
h 3 2 1
81 3
x x
−
=
)
2 2
2 3 3 1 .
x x
+ = −
following equations.
2 1 2 1
x x x
− +
b
2
2 2 3 2
1
9 243
81
x
x x
+
+ − −
 
=
 
 
3 4 3 4 1
16 2 64
x x x
+ − +
SYSTEMS OF LINEAR EQUATIONS IN
TWO VARIABLES
or real numbers a and b, any equation of the form ax + b = 0
he numbers a and b are called coefficients of the equation.
ACTIVITY 2.3
Solve each of the following linear equations.
b x + 7 = 3
d 2x – 5 = 7 e 3x + 5 = 14
How many solutions do you get for each equation?
3 1
1
32
16
x−
 
=
 
 
4 3
16 2
x x
+
=
SYSTEMS OF LINEAR EQUATIONS IN
= 0, where a ≠ 0 is
of the equation.
+ 5 = 14
Observe that each equation has exactly one solution. In general, any linear equation
one variable has one solution.
G r o u p W o r k 2 . 1
G r o u p W o r k 2 . 1
G r o u p W o r k 2 . 1
G r o u p W o r k 2 . 1
Form a group and do the following
1 Solve each of the following equations
a (
7 3 2 3 2
x x
− = +
c (
2 4 2 5
x x
+ = +
2 How many solutions do you get
3 What can you conclude about number of solutions?
From the Group Work, observe that
solutions or no solution.
Linear equations in two variables
We discussed how we solve
ax + b = 0. What do you think the solution is
1 Which of the following are linear equations in two variables?
a 2x – y = 5
d 2x – y2
= 7
2 How many solutions
3 A house was rented for
per m3
.
a Write an equation for the total cost
b If the total cost
an equation.
Definition 2.1
Any equation that can be reduced to the form
and a ≠ 0, is called a linear equation
Unit 2 Solutions of Equations
Observe that each equation has exactly one solution. In general, any linear equation
has one solution.
G r o u p W o r k 2 . 1
G r o u p W o r k 2 . 1
G r o u p W o r k 2 . 1
G r o u p W o r k 2 . 1
Form a group and do the following.
Solve each of the following equations.
)
7 3 2 3 2
− = + b ( ) ( )
3 2 4 2 3 6
x x
− + = − −
)
2 4 2 5
+ = +
How many solutions do you get for each equation?
What can you conclude about number of solutions?
observe that such equations can have one solution, infinite
Linear equations in two variables
discussed how we solve equations with one variable that can be reduced to the
What do you think the solution is, if the equation is given as
ACTIVITY 2.4
Which of the following are linear equations in two variables?
b –x + 7 = y c 2x+3 = 4
e
1 1
6
x y
+ =
How many solutions are there for each of the linear equations in two variables
for Birr 2,000 per month plus Birr 2 for water consumption
rite an equation for the total cost of x-years rent and 200 m3
If the total cost for x-years rent and y m3
of water used is Birr 106
ny equation that can be reduced to the form ax + b = 0, where
linear equation in one variable.
Solutions of Equations
67
Observe that each equation has exactly one solution. In general, any linear equation in
can have one solution, infinite
equations with one variable that can be reduced to the form
if the equation is given as y = ax + b?
of the linear equations in two variables?
irr 2 for water consumption
3
of water used.
irr 106,000 write
where a, b ∈ℝ
Mathematics Student G9_2.pdf
Mathematics Student G9_2.pdf
Mathematics Student G9_2.pdf
Mathematics Student G9_2.pdf
Mathematics Student G9_2.pdf
Mathematics Student G9_2.pdf
Mathematics Student G9_2.pdf
Mathematics Student G9_2.pdf
Mathematics Student G9_2.pdf
Mathematics Student G9_2.pdf
Mathematics Student G9_2.pdf
Mathematics Student G9_2.pdf
Mathematics Student G9_2.pdf
Mathematics Student G9_2.pdf
Mathematics Student G9_2.pdf
Mathematics Student G9_2.pdf
Mathematics Student G9_2.pdf
Mathematics Student G9_2.pdf
Mathematics Student G9_2.pdf
Mathematics Student G9_2.pdf
Mathematics Student G9_2.pdf
Mathematics Student G9_2.pdf
Mathematics Student G9_2.pdf
Mathematics Student G9_2.pdf
Mathematics Student G9_2.pdf
Mathematics Student G9_2.pdf
Mathematics Student G9_2.pdf
Mathematics Student G9_2.pdf
Mathematics Student G9_2.pdf
Mathematics Student G9_2.pdf
Mathematics Student G9_2.pdf
Mathematics Student G9_2.pdf
Mathematics Student G9_2.pdf
Mathematics Student G9_2.pdf
Mathematics Student G9_2.pdf
Mathematics Student G9_2.pdf
Mathematics Student G9_2.pdf
Mathematics Student G9_2.pdf
Mathematics Student G9_2.pdf
Mathematics Student G9_2.pdf
Mathematics Student G9_2.pdf
Mathematics Student G9_2.pdf
Mathematics Student G9_2.pdf
Mathematics Student G9_2.pdf
Mathematics Student G9_2.pdf
Mathematics Student G9_2.pdf
Mathematics Student G9_2.pdf
Mathematics Student G9_2.pdf
Mathematics Student G9_2.pdf
Mathematics Student G9_2.pdf
Mathematics Student G9_2.pdf
Mathematics Student G9_2.pdf
Mathematics Student G9_2.pdf
Mathematics Student G9_2.pdf
Mathematics Student G9_2.pdf
Mathematics Student G9_2.pdf
Mathematics Student G9_2.pdf
Mathematics Student G9_2.pdf
Mathematics Student G9_2.pdf
Mathematics Student G9_2.pdf
Mathematics Student G9_2.pdf
Mathematics Student G9_2.pdf
Mathematics Student G9_2.pdf
Mathematics Student G9_2.pdf
Mathematics Student G9_2.pdf
Mathematics Student G9_2.pdf
Mathematics Student G9_2.pdf
Mathematics Student G9_2.pdf
Mathematics Student G9_2.pdf
Mathematics Student G9_2.pdf
Mathematics Student G9_2.pdf
Mathematics Student G9_2.pdf
Mathematics Student G9_2.pdf
Mathematics Student G9_2.pdf
Mathematics Student G9_2.pdf
Mathematics Student G9_2.pdf
Mathematics Student G9_2.pdf
Mathematics Student G9_2.pdf
Mathematics Student G9_2.pdf
Mathematics Student G9_2.pdf
Mathematics Student G9_2.pdf
Mathematics Student G9_2.pdf
Mathematics Student G9_2.pdf
Mathematics Student G9_2.pdf
Mathematics Student G9_2.pdf
Mathematics Student G9_2.pdf
Mathematics Student G9_2.pdf
Mathematics Student G9_2.pdf
Mathematics Student G9_2.pdf
Mathematics Student G9_2.pdf
Mathematics Student G9_2.pdf
Mathematics Student G9_2.pdf
Mathematics Student G9_2.pdf
Mathematics Student G9_2.pdf
Mathematics Student G9_2.pdf
Mathematics Student G9_2.pdf
Mathematics Student G9_2.pdf
Mathematics Student G9_2.pdf
Mathematics Student G9_2.pdf
Mathematics Student G9_2.pdf
Mathematics Student G9_2.pdf
Mathematics Student G9_2.pdf
Mathematics Student G9_2.pdf
Mathematics Student G9_2.pdf
Mathematics Student G9_2.pdf
Mathematics Student G9_2.pdf
Mathematics Student G9_2.pdf
Mathematics Student G9_2.pdf
Mathematics Student G9_2.pdf
Mathematics Student G9_2.pdf
Mathematics Student G9_2.pdf
Mathematics Student G9_2.pdf
Mathematics Student G9_2.pdf
Mathematics Student G9_2.pdf
Mathematics Student G9_2.pdf
Mathematics Student G9_2.pdf
Mathematics Student G9_2.pdf
Mathematics Student G9_2.pdf
Mathematics Student G9_2.pdf
Mathematics Student G9_2.pdf
Mathematics Student G9_2.pdf
Mathematics Student G9_2.pdf
Mathematics Student G9_2.pdf
Mathematics Student G9_2.pdf
Mathematics Student G9_2.pdf
Mathematics Student G9_2.pdf
Mathematics Student G9_2.pdf
Mathematics Student G9_2.pdf
Mathematics Student G9_2.pdf
Mathematics Student G9_2.pdf
Mathematics Student G9_2.pdf
Mathematics Student G9_2.pdf
Mathematics Student G9_2.pdf
Mathematics Student G9_2.pdf
Mathematics Student G9_2.pdf
Mathematics Student G9_2.pdf
Mathematics Student G9_2.pdf
Mathematics Student G9_2.pdf
Mathematics Student G9_2.pdf
Mathematics Student G9_2.pdf
Mathematics Student G9_2.pdf
Mathematics Student G9_2.pdf
Mathematics Student G9_2.pdf
Mathematics Student G9_2.pdf
Mathematics Student G9_2.pdf
Mathematics Student G9_2.pdf
Mathematics Student G9_2.pdf
Mathematics Student G9_2.pdf
Mathematics Student G9_2.pdf
Mathematics Student G9_2.pdf
Mathematics Student G9_2.pdf
Mathematics Student G9_2.pdf
Mathematics Student G9_2.pdf
Mathematics Student G9_2.pdf
Mathematics Student G9_2.pdf
Mathematics Student G9_2.pdf
Mathematics Student G9_2.pdf
Mathematics Student G9_2.pdf
Mathematics Student G9_2.pdf
Mathematics Student G9_2.pdf
Mathematics Student G9_2.pdf
Mathematics Student G9_2.pdf
Mathematics Student G9_2.pdf
Mathematics Student G9_2.pdf
Mathematics Student G9_2.pdf
Mathematics Student G9_2.pdf
Mathematics Student G9_2.pdf
Mathematics Student G9_2.pdf
Mathematics Student G9_2.pdf
Mathematics Student G9_2.pdf
Mathematics Student G9_2.pdf
Mathematics Student G9_2.pdf
Mathematics Student G9_2.pdf
Mathematics Student G9_2.pdf
Mathematics Student G9_2.pdf
Mathematics Student G9_2.pdf
Mathematics Student G9_2.pdf
Mathematics Student G9_2.pdf
Mathematics Student G9_2.pdf
Mathematics Student G9_2.pdf
Mathematics Student G9_2.pdf
Mathematics Student G9_2.pdf
Mathematics Student G9_2.pdf
Mathematics Student G9_2.pdf
Mathematics Student G9_2.pdf
Mathematics Student G9_2.pdf
Mathematics Student G9_2.pdf
Mathematics Student G9_2.pdf
Mathematics Student G9_2.pdf
Mathematics Student G9_2.pdf
Mathematics Student G9_2.pdf
Mathematics Student G9_2.pdf
Mathematics Student G9_2.pdf
Mathematics Student G9_2.pdf
Mathematics Student G9_2.pdf
Mathematics Student G9_2.pdf
Mathematics Student G9_2.pdf
Mathematics Student G9_2.pdf
Mathematics Student G9_2.pdf
Mathematics Student G9_2.pdf
Mathematics Student G9_2.pdf
Mathematics Student G9_2.pdf
Mathematics Student G9_2.pdf
Mathematics Student G9_2.pdf
Mathematics Student G9_2.pdf
Mathematics Student G9_2.pdf
Mathematics Student G9_2.pdf
Mathematics Student G9_2.pdf
Mathematics Student G9_2.pdf
Mathematics Student G9_2.pdf
Mathematics Student G9_2.pdf
Mathematics Student G9_2.pdf
Mathematics Student G9_2.pdf
Mathematics Student G9_2.pdf
Mathematics Student G9_2.pdf
Mathematics Student G9_2.pdf
Mathematics Student G9_2.pdf
Mathematics Student G9_2.pdf
Mathematics Student G9_2.pdf
Mathematics Student G9_2.pdf
Mathematics Student G9_2.pdf
Mathematics Student G9_2.pdf
Mathematics Student G9_2.pdf
Mathematics Student G9_2.pdf
Mathematics Student G9_2.pdf
Mathematics Student G9_2.pdf
Mathematics Student G9_2.pdf
Mathematics Student G9_2.pdf
Mathematics Student G9_2.pdf
Mathematics Student G9_2.pdf
Mathematics Student G9_2.pdf
Mathematics Student G9_2.pdf
Mathematics Student G9_2.pdf
Mathematics Student G9_2.pdf
Mathematics Student G9_2.pdf
Mathematics Student G9_2.pdf
Mathematics Student G9_2.pdf
Mathematics Student G9_2.pdf
Mathematics Student G9_2.pdf
Mathematics Student G9_2.pdf
Mathematics Student G9_2.pdf
Mathematics Student G9_2.pdf
Mathematics Student G9_2.pdf
Mathematics Student G9_2.pdf
Mathematics Student G9_2.pdf
Mathematics Student G9_2.pdf
Mathematics Student G9_2.pdf
Mathematics Student G9_2.pdf
Mathematics Student G9_2.pdf

More Related Content

Similar to Mathematics Student G9_2.pdf

Logic for everyone
Logic for everyoneLogic for everyone
Logic for everyoneble nature
 
4 ma0 3hr_que_20150106
4 ma0 3hr_que_201501064 ma0 3hr_que_20150106
4 ma0 3hr_que_20150106AnneRostom
 
Modul fizik cakna kelantan spm 2014 k3 set 2 dan skema
Modul fizik cakna kelantan spm 2014 k3 set 2 dan skemaModul fizik cakna kelantan spm 2014 k3 set 2 dan skema
Modul fizik cakna kelantan spm 2014 k3 set 2 dan skemaCikgu Pejal
 
9th - Maths - EM - www.tntextbooks.online.pdf
9th - Maths - EM - www.tntextbooks.online.pdf9th - Maths - EM - www.tntextbooks.online.pdf
9th - Maths - EM - www.tntextbooks.online.pdfSankar Tiruchencode
 
chem year 10 midterm.pdf
chem year 10 midterm.pdfchem year 10 midterm.pdf
chem year 10 midterm.pdfBarmingaKamuren
 
Statistics firstfive
Statistics firstfiveStatistics firstfive
Statistics firstfiveSukirti Garg
 
modul 4 add maths 07
modul 4 add maths 07modul 4 add maths 07
modul 4 add maths 07Sasi Villa
 
Introduction to artificial_intelligence_syllabus
Introduction to artificial_intelligence_syllabusIntroduction to artificial_intelligence_syllabus
Introduction to artificial_intelligence_syllabusAndres Mendez-Vazquez
 
WEEK 7 mathematics 7, quarter 3 week 7..
WEEK 7 mathematics 7, quarter 3 week 7..WEEK 7 mathematics 7, quarter 3 week 7..
WEEK 7 mathematics 7, quarter 3 week 7..ArmanPuyao
 
Summer_Holiday_home_work__class_9_2023.pdf
Summer_Holiday_home_work__class_9_2023.pdfSummer_Holiday_home_work__class_9_2023.pdf
Summer_Holiday_home_work__class_9_2023.pdfVanshuMugvana
 
ix-number system-ppt(2).pptx
ix-number system-ppt(2).pptxix-number system-ppt(2).pptx
ix-number system-ppt(2).pptxRajkumarknms
 
Numerical methods by Jeffrey R. Chasnov
Numerical methods by Jeffrey R. ChasnovNumerical methods by Jeffrey R. Chasnov
Numerical methods by Jeffrey R. Chasnovankushnathe
 
Algorithms For Differential Games With Bounded Control And State
Algorithms For Differential Games With Bounded Control And StateAlgorithms For Differential Games With Bounded Control And State
Algorithms For Differential Games With Bounded Control And StateSarah Brown
 

Similar to Mathematics Student G9_2.pdf (20)

201977 1-1-3-pb
201977 1-1-3-pb201977 1-1-3-pb
201977 1-1-3-pb
 
Logic for everyone
Logic for everyoneLogic for everyone
Logic for everyone
 
no maths
no mathsno maths
no maths
 
4 ma0 3hr_que_20150106
4 ma0 3hr_que_201501064 ma0 3hr_que_20150106
4 ma0 3hr_que_20150106
 
Modul fizik cakna kelantan spm 2014 k3 set 2 dan skema
Modul fizik cakna kelantan spm 2014 k3 set 2 dan skemaModul fizik cakna kelantan spm 2014 k3 set 2 dan skema
Modul fizik cakna kelantan spm 2014 k3 set 2 dan skema
 
Sequence and series
Sequence and seriesSequence and series
Sequence and series
 
9th - Maths - EM - www.tntextbooks.online.pdf
9th - Maths - EM - www.tntextbooks.online.pdf9th - Maths - EM - www.tntextbooks.online.pdf
9th - Maths - EM - www.tntextbooks.online.pdf
 
chem year 10 midterm.pdf
chem year 10 midterm.pdfchem year 10 midterm.pdf
chem year 10 midterm.pdf
 
intersection and union
intersection and unionintersection and union
intersection and union
 
Statistics firstfive
Statistics firstfiveStatistics firstfive
Statistics firstfive
 
modul 4 add maths 07
modul 4 add maths 07modul 4 add maths 07
modul 4 add maths 07
 
Introduction to artificial_intelligence_syllabus
Introduction to artificial_intelligence_syllabusIntroduction to artificial_intelligence_syllabus
Introduction to artificial_intelligence_syllabus
 
WEEK 7 mathematics 7, quarter 3 week 7..
WEEK 7 mathematics 7, quarter 3 week 7..WEEK 7 mathematics 7, quarter 3 week 7..
WEEK 7 mathematics 7, quarter 3 week 7..
 
Summer_Holiday_home_work__class_9_2023.pdf
Summer_Holiday_home_work__class_9_2023.pdfSummer_Holiday_home_work__class_9_2023.pdf
Summer_Holiday_home_work__class_9_2023.pdf
 
ix-number system-ppt(2).pptx
ix-number system-ppt(2).pptxix-number system-ppt(2).pptx
ix-number system-ppt(2).pptx
 
Example
ExampleExample
Example
 
Abstract algebra
Abstract algebraAbstract algebra
Abstract algebra
 
abstract algebra
abstract algebraabstract algebra
abstract algebra
 
Numerical methods by Jeffrey R. Chasnov
Numerical methods by Jeffrey R. ChasnovNumerical methods by Jeffrey R. Chasnov
Numerical methods by Jeffrey R. Chasnov
 
Algorithms For Differential Games With Bounded Control And State
Algorithms For Differential Games With Bounded Control And StateAlgorithms For Differential Games With Bounded Control And State
Algorithms For Differential Games With Bounded Control And State
 

More from BeniamTekeste

PPT- rational and irrational numbers.ppt
PPT- rational and irrational numbers.pptPPT- rational and irrational numbers.ppt
PPT- rational and irrational numbers.pptBeniamTekeste
 
ERROR_DETECTION.pptx
ERROR_DETECTION.pptxERROR_DETECTION.pptx
ERROR_DETECTION.pptxBeniamTekeste
 
Layer_arc_and_OSI_MODEL.ppt
Layer_arc_and_OSI_MODEL.pptLayer_arc_and_OSI_MODEL.ppt
Layer_arc_and_OSI_MODEL.pptBeniamTekeste
 
Data Center Multi-tier Model Overview.pptx
Data Center Multi-tier Model Overview.pptxData Center Multi-tier Model Overview.pptx
Data Center Multi-tier Model Overview.pptxBeniamTekeste
 

More from BeniamTekeste (7)

math.pptx
math.pptxmath.pptx
math.pptx
 
Call-4-Exam-DD.pdf
Call-4-Exam-DD.pdfCall-4-Exam-DD.pdf
Call-4-Exam-DD.pdf
 
PPT- rational and irrational numbers.ppt
PPT- rational and irrational numbers.pptPPT- rational and irrational numbers.ppt
PPT- rational and irrational numbers.ppt
 
ERROR_DETECTION.pptx
ERROR_DETECTION.pptxERROR_DETECTION.pptx
ERROR_DETECTION.pptx
 
Layer_arc_and_OSI_MODEL.ppt
Layer_arc_and_OSI_MODEL.pptLayer_arc_and_OSI_MODEL.ppt
Layer_arc_and_OSI_MODEL.ppt
 
Data Center Multi-tier Model Overview.pptx
Data Center Multi-tier Model Overview.pptxData Center Multi-tier Model Overview.pptx
Data Center Multi-tier Model Overview.pptx
 
About Cyber.pptx
About Cyber.pptxAbout Cyber.pptx
About Cyber.pptx
 

Recently uploaded

A Critique of the Proposed National Education Policy Reform
A Critique of the Proposed National Education Policy ReformA Critique of the Proposed National Education Policy Reform
A Critique of the Proposed National Education Policy ReformChameera Dedduwage
 
Industrial Policy - 1948, 1956, 1973, 1977, 1980, 1991
Industrial Policy - 1948, 1956, 1973, 1977, 1980, 1991Industrial Policy - 1948, 1956, 1973, 1977, 1980, 1991
Industrial Policy - 1948, 1956, 1973, 1977, 1980, 1991RKavithamani
 
Contemporary philippine arts from the regions_PPT_Module_12 [Autosaved] (1).pptx
Contemporary philippine arts from the regions_PPT_Module_12 [Autosaved] (1).pptxContemporary philippine arts from the regions_PPT_Module_12 [Autosaved] (1).pptx
Contemporary philippine arts from the regions_PPT_Module_12 [Autosaved] (1).pptxRoyAbrique
 
Student login on Anyboli platform.helpin
Student login on Anyboli platform.helpinStudent login on Anyboli platform.helpin
Student login on Anyboli platform.helpinRaunakKeshri1
 
Introduction to AI in Higher Education_draft.pptx
Introduction to AI in Higher Education_draft.pptxIntroduction to AI in Higher Education_draft.pptx
Introduction to AI in Higher Education_draft.pptxpboyjonauth
 
Measures of Central Tendency: Mean, Median and Mode
Measures of Central Tendency: Mean, Median and ModeMeasures of Central Tendency: Mean, Median and Mode
Measures of Central Tendency: Mean, Median and ModeThiyagu K
 
Kisan Call Centre - To harness potential of ICT in Agriculture by answer farm...
Kisan Call Centre - To harness potential of ICT in Agriculture by answer farm...Kisan Call Centre - To harness potential of ICT in Agriculture by answer farm...
Kisan Call Centre - To harness potential of ICT in Agriculture by answer farm...Krashi Coaching
 
POINT- BIOCHEMISTRY SEM 2 ENZYMES UNIT 5.pptx
POINT- BIOCHEMISTRY SEM 2 ENZYMES UNIT 5.pptxPOINT- BIOCHEMISTRY SEM 2 ENZYMES UNIT 5.pptx
POINT- BIOCHEMISTRY SEM 2 ENZYMES UNIT 5.pptxSayali Powar
 
Grant Readiness 101 TechSoup and Remy Consulting
Grant Readiness 101 TechSoup and Remy ConsultingGrant Readiness 101 TechSoup and Remy Consulting
Grant Readiness 101 TechSoup and Remy ConsultingTechSoup
 
The Most Excellent Way | 1 Corinthians 13
The Most Excellent Way | 1 Corinthians 13The Most Excellent Way | 1 Corinthians 13
The Most Excellent Way | 1 Corinthians 13Steve Thomason
 
Accessible design: Minimum effort, maximum impact
Accessible design: Minimum effort, maximum impactAccessible design: Minimum effort, maximum impact
Accessible design: Minimum effort, maximum impactdawncurless
 
Nutritional Needs Presentation - HLTH 104
Nutritional Needs Presentation - HLTH 104Nutritional Needs Presentation - HLTH 104
Nutritional Needs Presentation - HLTH 104misteraugie
 
Organic Name Reactions for the students and aspirants of Chemistry12th.pptx
Organic Name Reactions  for the students and aspirants of Chemistry12th.pptxOrganic Name Reactions  for the students and aspirants of Chemistry12th.pptx
Organic Name Reactions for the students and aspirants of Chemistry12th.pptxVS Mahajan Coaching Centre
 
Introduction to ArtificiaI Intelligence in Higher Education
Introduction to ArtificiaI Intelligence in Higher EducationIntroduction to ArtificiaI Intelligence in Higher Education
Introduction to ArtificiaI Intelligence in Higher Educationpboyjonauth
 
The basics of sentences session 2pptx copy.pptx
The basics of sentences session 2pptx copy.pptxThe basics of sentences session 2pptx copy.pptx
The basics of sentences session 2pptx copy.pptxheathfieldcps1
 
“Oh GOSH! Reflecting on Hackteria's Collaborative Practices in a Global Do-It...
“Oh GOSH! Reflecting on Hackteria's Collaborative Practices in a Global Do-It...“Oh GOSH! Reflecting on Hackteria's Collaborative Practices in a Global Do-It...
“Oh GOSH! Reflecting on Hackteria's Collaborative Practices in a Global Do-It...Marc Dusseiller Dusjagr
 

Recently uploaded (20)

A Critique of the Proposed National Education Policy Reform
A Critique of the Proposed National Education Policy ReformA Critique of the Proposed National Education Policy Reform
A Critique of the Proposed National Education Policy Reform
 
Industrial Policy - 1948, 1956, 1973, 1977, 1980, 1991
Industrial Policy - 1948, 1956, 1973, 1977, 1980, 1991Industrial Policy - 1948, 1956, 1973, 1977, 1980, 1991
Industrial Policy - 1948, 1956, 1973, 1977, 1980, 1991
 
Contemporary philippine arts from the regions_PPT_Module_12 [Autosaved] (1).pptx
Contemporary philippine arts from the regions_PPT_Module_12 [Autosaved] (1).pptxContemporary philippine arts from the regions_PPT_Module_12 [Autosaved] (1).pptx
Contemporary philippine arts from the regions_PPT_Module_12 [Autosaved] (1).pptx
 
Student login on Anyboli platform.helpin
Student login on Anyboli platform.helpinStudent login on Anyboli platform.helpin
Student login on Anyboli platform.helpin
 
Introduction to AI in Higher Education_draft.pptx
Introduction to AI in Higher Education_draft.pptxIntroduction to AI in Higher Education_draft.pptx
Introduction to AI in Higher Education_draft.pptx
 
Mattingly "AI & Prompt Design: The Basics of Prompt Design"
Mattingly "AI & Prompt Design: The Basics of Prompt Design"Mattingly "AI & Prompt Design: The Basics of Prompt Design"
Mattingly "AI & Prompt Design: The Basics of Prompt Design"
 
TataKelola dan KamSiber Kecerdasan Buatan v022.pdf
TataKelola dan KamSiber Kecerdasan Buatan v022.pdfTataKelola dan KamSiber Kecerdasan Buatan v022.pdf
TataKelola dan KamSiber Kecerdasan Buatan v022.pdf
 
Measures of Central Tendency: Mean, Median and Mode
Measures of Central Tendency: Mean, Median and ModeMeasures of Central Tendency: Mean, Median and Mode
Measures of Central Tendency: Mean, Median and Mode
 
Staff of Color (SOC) Retention Efforts DDSD
Staff of Color (SOC) Retention Efforts DDSDStaff of Color (SOC) Retention Efforts DDSD
Staff of Color (SOC) Retention Efforts DDSD
 
Kisan Call Centre - To harness potential of ICT in Agriculture by answer farm...
Kisan Call Centre - To harness potential of ICT in Agriculture by answer farm...Kisan Call Centre - To harness potential of ICT in Agriculture by answer farm...
Kisan Call Centre - To harness potential of ICT in Agriculture by answer farm...
 
INDIA QUIZ 2024 RLAC DELHI UNIVERSITY.pptx
INDIA QUIZ 2024 RLAC DELHI UNIVERSITY.pptxINDIA QUIZ 2024 RLAC DELHI UNIVERSITY.pptx
INDIA QUIZ 2024 RLAC DELHI UNIVERSITY.pptx
 
POINT- BIOCHEMISTRY SEM 2 ENZYMES UNIT 5.pptx
POINT- BIOCHEMISTRY SEM 2 ENZYMES UNIT 5.pptxPOINT- BIOCHEMISTRY SEM 2 ENZYMES UNIT 5.pptx
POINT- BIOCHEMISTRY SEM 2 ENZYMES UNIT 5.pptx
 
Grant Readiness 101 TechSoup and Remy Consulting
Grant Readiness 101 TechSoup and Remy ConsultingGrant Readiness 101 TechSoup and Remy Consulting
Grant Readiness 101 TechSoup and Remy Consulting
 
The Most Excellent Way | 1 Corinthians 13
The Most Excellent Way | 1 Corinthians 13The Most Excellent Way | 1 Corinthians 13
The Most Excellent Way | 1 Corinthians 13
 
Accessible design: Minimum effort, maximum impact
Accessible design: Minimum effort, maximum impactAccessible design: Minimum effort, maximum impact
Accessible design: Minimum effort, maximum impact
 
Nutritional Needs Presentation - HLTH 104
Nutritional Needs Presentation - HLTH 104Nutritional Needs Presentation - HLTH 104
Nutritional Needs Presentation - HLTH 104
 
Organic Name Reactions for the students and aspirants of Chemistry12th.pptx
Organic Name Reactions  for the students and aspirants of Chemistry12th.pptxOrganic Name Reactions  for the students and aspirants of Chemistry12th.pptx
Organic Name Reactions for the students and aspirants of Chemistry12th.pptx
 
Introduction to ArtificiaI Intelligence in Higher Education
Introduction to ArtificiaI Intelligence in Higher EducationIntroduction to ArtificiaI Intelligence in Higher Education
Introduction to ArtificiaI Intelligence in Higher Education
 
The basics of sentences session 2pptx copy.pptx
The basics of sentences session 2pptx copy.pptxThe basics of sentences session 2pptx copy.pptx
The basics of sentences session 2pptx copy.pptx
 
“Oh GOSH! Reflecting on Hackteria's Collaborative Practices in a Global Do-It...
“Oh GOSH! Reflecting on Hackteria's Collaborative Practices in a Global Do-It...“Oh GOSH! Reflecting on Hackteria's Collaborative Practices in a Global Do-It...
“Oh GOSH! Reflecting on Hackteria's Collaborative Practices in a Global Do-It...
 

Mathematics Student G9_2.pdf

  • 2. 7DNH*RRGDUHRI 7DNH*RRGDUHRI 7DNH*RRGDUHRI 7DNH*RRGDUHRI 7KLV7H[WERRN 7KLV7H[WERRN 7KLV7H[WERRN 7KLV7H[WERRN dŚŝƐƚĞdžƚŬŝƐƚŚĞƉƌŽƉĞƌƚLJŽĨLJŽƵƌƐĐŚŽŽů͘ dĂŬĞŐŽŽĚĐĂƌĞŶŽƚƚŽĚĂŵĂŐĞŽƌůŽƐĞŝƚ͘ ,ĞƌĞĂƌĞϭϬŝĚĞĂƐƚŽŚĞůƉƚĂŬĞĐĂƌĞŽĨƚŚĞŬ͗ ϭ͘ ŽǀĞƌƚŚĞŬǁŝƚŚƉƌŽƚĞĐƚŝǀĞŵĂƚĞƌŝĂů͕ƐƵĐŚĂƐƉůĂƐƚŝĐ͕ŽůĚ ŶĞǁƐƉĂƉĞƌƐŽƌŵĂŐĂnjŝŶĞƐ͘ Ϯ͘ ůǁĂLJƐŬĞĞƉƚŚĞŬŝŶĂĐůĞĂŶĚƌLJƉůĂĐĞ͘ ϯ͘ ĞƐƵƌĞLJŽƵƌŚĂŶĚƐĂƌĞĐůĞĂŶǁŚĞŶLJŽƵƵƐĞƚŚĞŬ͘ ϰ͘ ŽŶŽƚǁƌŝƚĞŽŶƚŚĞĐŽǀĞƌŽƌŝŶƐŝĚĞƉĂŐĞƐ͘ ϱ͘ hƐĞĂƉŝĞĐĞŽĨƉĂƉĞƌŽƌĐĂƌĚďŽĂƌĚĂƐĂŬŵĂƌŬ͘ ϲ͘ EĞǀĞƌƚĞĂƌŽƌĐƵƚŽƵƚĂŶLJƉŝĐƚƵƌĞƐŽƌƉĂŐĞƐ͘ ϳ͘ ZĞƉĂŝƌĂŶLJƚŽƌŶƉĂŐĞƐǁŝƚŚƉĂƐƚĞŽƌƚĂƉĞ͘ ϴ͘ WĂĐŬƚŚĞŬĐĂƌĞĨƵůůLJǁŚĞŶLJŽƵƉůĂĐĞŝƚŝŶLJŽƵƌƐĐŚŽŽůďĂŐ͘ ϵ͘ ,ĂŶĚůĞƚŚĞŬǁŝƚŚĐĂƌĞǁŚĞŶƉĂƐƐŝŶŐŝƚƚŽĂŶŽƚŚĞƌƉĞƌƐŽŶ͘ ϭϬ͘ tŚĞŶƵƐŝŶŐĂŶĞǁŬĨŽƌƚŚĞĨŝƌƐƚƚŝŵĞ͕ůĂLJŝƚŽŶŝƚƐďĂĐŬ͘KƉĞŶ ŽŶůLJĂĨĞǁƉĂŐĞƐĂƚĂƚŝŵĞ͘WƌĞƐƐůŝŐŚƚůLJĂůŽŶŐƚŚĞďŽƵŶĚĞĚŐĞĂƐ LJŽƵƚƵƌŶƚŚĞƉĂŐĞƐ͘dŚŝƐǁŝůůŬĞĞƉƚŚĞĐŽǀĞƌŝŶŐŽŽĚĐŽŶĚŝƚŝŽŶ͘
  • 3. MATHEMATICS STUDENT TEXTBOOK Authors, Editors and Reviewers: C.K. Bansal (B.Sc., M.A.) Rachel Mary Z Dawod Adem Kassa Michael Berhanu Guta Evaluators: Tesfaye Ayele Dagnachew Yalew Tekeste Woldetensai FEDERAL DEMOCRATIC REPUBLIC OF ETHIOPIA MATHEMATICS STUDENT TEXTBOOK GRADE GRADE GRADE GRADE 9 9 9 9 Authors, Editors and Reviewers: (B.Sc., M.A.) Rachel Mary Z. (B.Sc.) (M.Sc.) Kassa Michael (M.Sc.) Berhanu Guta (Ph.D.) Tesfaye Ayele Dagnachew Yalew Tekeste Woldetensai FEDERAL DEMOCRATIC REPUBLIC OF ETHIOPIA MINISTRY OF EDUCATION MATHEMATICS STUDENT TEXTBOOK 9 9 9 9
  • 4. Published E.C. 2002 by the Federal Democratic Republic of Ethiopia, Ministry of Education, under the General Education Quality Improvement Project (GEQIP) supported by IDA Credit No. 4535-ET, the Fast Track Initiative Catalytic Fund and the Governments of Finland, Italy, Netherlands and the United Kingdom. © 2010 by the Federal Democratic Republic of Ethiopia, Ministry of Education. All rights reserved. No part of this book may be reproduced, stored in a retrieval system or transmitted in any form or by any means including electronic, mechanical, magnetic or other, without prior written permission of the Ministry of Education or licensing in accordance with the Federal Democratic Republic of Ethiopia, Federal Negarit Gazeta, Proclamation No. 410/2004 – Copyright and Neighbouring Rights Protection. The Ministry of Education wishes to thank the many individuals, groups and other bodies involved – directly and indirectly – in publishing this textbook and the accompanying teacher guide. Copyrighted materials used by permission of their owners. If you are the owner of copyrighted material not cited or improperly cited, please contact with the Ministry of Education, Head Office, Arat Kilo, (PO Box 1367), Addis Ababa, Ethiopia. PHOTO CREDIT: p.106 and 292-Encarta Encyclopedia, 2009 edition. p.126-http:// en.wikipedia.org p.277-http://en.wikipedia.org Developed and Printed by STAR EDUCATIONAL BOOKS DISTRIBUTORS Pvt. Ltd. 24/4800, Bharat Ram Road, Daryaganj, New Delhi – 110002, INDIA and ASTER NEGA PUBLISHING ENTERPRISE P.O. Box 21073 ADDIS ABABA, ETHIOPIA under GEQIP Contract No. ET-MoE/GEQIP/IDA/ICB/G01/09. ISBN 978-99944-2-042-1
  • 5. U Numerical Value Arabic Numeral 1 2 3 5 10 20 21 100 Babylonian Egyptian Hieroglyphic I II III IIIII Λ ΛΛ IΛΛ ϑ Greek Herodianic Ι ΙΙ ΙΙΙ Γ ∆ ∆∆ ∆∆Ι Η Roman I II III V X XX XXI C Ethiopian Geez 1 2 3 5 0 ! !1 ) Contents Unit 1 1 The Number System........................ 1 1.1 Revision on the set of rational numbers...........2 1.2 The real number system....................................13 Key Terms ............................................. 59 Summary .............................................. 59 Review Exercises on Unit 1..................... 61 Unit 2 2 Solutions of Equations .................63 2.1 Equations involving exponents and radicals...............................................................64 2.2 Systems of linear equations in two variables.............................................................66 2.3 Equations involving absolute value .................82 2.4 Quadratic equations.........................................86 Key Terms ............................................. 101 Summary .............................................. 102 Review Exercises on Unit 2..................... 103 Unit 3 3 Further on Sets 105 3.1 Ways to describe sets........................................106 3.2 The notation of sets............................................110 3.3 Operations on sets.............................................119 Key Terms ............................................. 135 Summary .............................................. 135 Review Exercises on Unit 3..................... 136 Unit 4 4 Relations and Functions .......... 139 4.1 Relations.............................................................140 4.2 Functions ............................................................149 4.3 Graphs of functions ...........................................157 Key Terms ............................................. 169 Summary .............................................. 170 Review Exercises on Unit 4..................... 171
  • 6. Contents Unit 5 5 Geometry and Measurement ... 175 5.1 Regular polygons ..............................................176 5.2 Further on congruency and similarity..............190 5.3 Further on trigonometry.....................................210 5.4 Circles.................................................................221 5.5 Measurement.....................................................232 Key Terms ............................................. 243 Summary .............................................. 243 Review Exercises on Unit 5..................... 247 Unit 6 6 Statistics and Probability.......... 249 6.1 Statistical data ...................................................250 6.2 Probability ..........................................................277 Key Terms ............................................. 286 Summary .............................................. 286 Review Exercises on Unit 6..................... 288 Unit 7 7 Vectors in Two Dimensions...... 291 7.1 Introduction to Vectors and scalars.................292 7.2 Representation of a vector ...............................294 7.3 Addition and subtraction of vectors and multiplication of a vector by a scalar.............. 299 7.4 Position vector of a point ..................................307 Key Terms ............................................. 311 Summary .............................................. 311 Review Exercises on Unit 7..................... 312 Table of Trigonometric Functions...............................313
  • 7. Numerical Value Arabic Numeral 1 2 3 5 10 20 21 100 Babylonian Egyptian Hieroglyphic I II III IIIII Λ ΛΛ IΛΛ ϑ Greek Herodianic Ι ΙΙ ΙΙΙ Γ ∆ ∆∆ ∆∆Ι Η Roman I II III V X XX XXI C Ethiopian Geez 1 2 3 5 0 ! !1 ) Unit Outcomes: After completing this unit, you should be able to: know basic concepts and important facts about real numbers. justify methods and procedures in computation with real numbers. solve mathematical problems involving real numbers. Main Contents 1.1 Revision on the set of rational numbers 1.2 The real number system Key Terms Summary Review Exercises U Un ni it t T T T T T T T TH H H H H H H HE E E E E E E E N N N N N N N NU U U U U U U UM M M M M M M MB B B B B B B BE E E E E E E ER R R R R R R R S S S S S S S SY Y Y Y Y Y Y YS S S S S S S ST T T T T T T TE E E E E E E EM M M M M M M M
  • 8. Mathematics Grade 9 2 INTRODUCTION In earlier grades, you have learnt about rational numbers, their properties, and basic mathematical operations upon them. After a review of your knowledge about rational numbers, you will continue studying the number systems in the present unit. Here, you will learn about irrational numbers and real numbers, their properties and basic operations upon them. Also, you will discuss some related concepts such as approximation, accuracy, and scientific notation. 1.1 REVISION ON THE SET OF RATIONAL NUMBERS ACTIVITY 1.1 The diagram below shows the relationships between the sets of Natural numbers, Whole numbers, Integers and Rational numbers. Use this diagram to answer Questions 1 and 2 given below. Justify your answers. 1 To which set(s) of numbers does each of the following numbers belong? a 27 b –17 c –7 2 3 d 0.625 e 0.615 2 i Define the set of: a Natural numbers b Whole numbers c Integers d Rational numbers ii What relations do these sets have? Figure 1.1 1.1.1 Natural Numbers, Integers, Prime Numbers and Composite Numbers In this subsection, you will revise important facts about the sets of natural numbers, prime numbers, composite numbers and integers. You have learnt several facts about these sets in previous grades, in Grade 7 in particular. Working through Activity 1.2 below will refresh your memory! 1, 2, 3, . . . 0 –11 –3 0 7 . 3 4 1 2 −
  • 9. Unit 1 The Number System 3 ACTIVITY 1.2 1 For each of the following statements write 'true' if the statement is correct or 'false' otherwise. If your answer is 'false', justify by giving a counter example or reason. a The set {1, 2, 3, . . .} describes the set of natural numbers. b The set {1, 2, 3, . . .}⋃{. . . −3, −2, −1} describes the set of integers. c 57 is a composite number. d {1} ∩ {Prime numbers} = ∅. e {Prime numbers}⋃{Composite number} = {1, 2, 3, . . .}. f {Odd numbers} ∩{Composite numbers} ≠ ∅. g 48 is a multiple of 12. h 5 is a factor of 72. i 621 is divisible by 3. j {Factors of 24} ∩ {Factors of 87} = {1, 2, 3}. k {Multiples of 6} ∩ {Multiples of 4} = {12, 24}. l 2 2 2 3 5 × × is the prime factorization of 180. 2 Given two natural numbers a and b, what is meant by: a a is a factor of b b a is divisible by b c a is a multiple of b From your lower grade mathematics, recall that; The set of natural numbers, denoted by N, is described by N = {1, 2, 3,…} The set of whole numbers, denoted by W, is described by W = {0, 1, 2, 3,…} The set of integers, denoted by Z, is described by Z = {...,–3, –2, –1, 0, 1, 2, 3,...} Given two natural numbers m and p, m is called a multiple of p if there is a natural number q such that m = p × q. In this case, p is called a factor or divisor of m. We also say m is divisible by p. Similarly, q is also a factor or divisor of m, and m is divisible by q.
  • 10. Mathematics Grade 9 4 For example, 621 is a multiple of 3 because 621 = 3 × 207. Note: 1 is neither prime nor composite. G r o u p W o r k 1 . 1 G r o u p W o r k 1 . 1 G r o u p W o r k 1 . 1 G r o u p W o r k 1 . 1 1 List all factors of 24. How many factors did you find? 2 The area of a rectangle is 432 sq. units. The measurements of the length and width of the rectangle are expressed by natural numbers. Find all the possible dimensions (length and width) of the rectangle. 3 Find the prime factorization of 360. The following rules can help you to determine whether a number is divisible by 2, 3, 4, 5, 6, 8, 9 or 10. Divisibility test A number is divisible by: 2, if its unit’s digit is divisible by 2. 3, if the sum of its digits is divisible by 3. 4, if the number formed by its last two digits is divisible by 4. 5, if its unit’s digit is either 0 or 5. 6, if it is divisible by 2 and 3. 8, if the number formed by its last three digits is divisible by 8. 9, if the sum of its digits is divisible by 9. 10, if its unit’s digit is 0. Observe that divisibility test for 7 is not stated here as it is beyond the scope of your present level. Definition 1.1 Prime numbers and composite numbers ♦ ♦ ♦ A natural number that has exactly two distinct factors, namely 1 and itself, is called a prime number. ♦ ♦ ♦ A natural number that has more than two factors is called a composite number.
  • 11. Unit 1 The Number System 5 Example 1 Use the divisibility test to determine whether 2,416 is divisible by 2, 3, 4, 5, 6, 8, 9 and 10. Solution: ♦ 2, 416 is divisible by 2 because the unit’s digit 6 is divisible by 2. ♦ 2,416 is divisible by 4 because 16 (the number formed by the last two digits) is divisible by 4. ♦ 2,416 is divisible by 8 because the number formed by the last three digits (416) is divisible by 8. ♦ 2,416 is not divisible by 5 because the unit’s digit is not 0 or 5. ♦ Similarly you can check that 2,416 is not divisible by 3, 6, 9, and 10. Therefore, 2,416 is divisible by 2, 4 and 8 but not by 3, 5, 6, 9 and 10. A factor of a composite number is called a prime factor, if it is a prime number. For instance, 2 and 5 are both prime factors of 20. Every composite number can be written as a product of prime numbers. To find the prime factors of any composite number, begin by expressing the number as a product of two factors where at least one of the factors is prime. Then, continue to factorize each resulting composite factor until all the factors are prime numbers. When a number is expressed as a product of its prime factors, the expression is called the prime factorization of the number. For example, the prime factorization of 60 is 60 = 2 × 2 × 3 × 5 = 22 × 3 × 5. The prime factorization of 60 is also found by using a factoring tree. Note that the set {2, 3, 5} is a set of prime factors of 60. Is this set unique? This property leads us to state the Fundamental Theorem of Arithmetic. You can use the divisibility tests to check whether or not a prime number divides a given number. Theorem 1.1 Fundamental theorem of arithmetic Every composite number can be expressed (factorized) as a product of primes. This factorization is unique, apart from the order in which the prime factors occur. 2 60 2 30 3 5 15
  • 12. Mathematics Grade 9 6 Example 2 Find the prime factorization of 1,530. Solution: Start dividing 1,530 by its smallest prime factor. If the quotient is a composite number, find a prime factor of the quotient in the same way. Repeat the procedure until the quotient is a prime number as shown below. 1,530 765 765 255 255 85 85 Prime factors 17 2 3 ; and 17 is a prime 3 5 number. ÷ = ÷ = ÷ = ÷ = ↓ Therefore, 1,530 = 2 × 32 × 5 × 17. 1.1.2 Common Factors and Common Multiples In this subsection, you will revise the concepts of common factors and common multiples of two or more natural numbers. Related to this, you will also revise the greatest common factor and the least common multiple of two or more natural numbers. A Common factors and the greatest common factor ACTIVITY 1.3 1 Given the numbers 30 and 45, a find the common factors of the two numbers. b find the greatest common factor of the two numbers. 2 Given the numbers 36, 42 and 48, a find the common factors of the three numbers. b find the greatest common factor of the three numbers. Given two or more natural numbers, a number which is a factor of all of them is called a common factor. Numbers may have more than one common factor. The greatest of the common factors is called the greatest common factor (GCF) or the highest common factor (HCF) of the numbers. The greatest common factor of two numbers a and b is denoted by GCF (a, b). Example 1 Find the greatest common factor of: a 36 and 60. b 32 and 27.
  • 13. Unit 1 The Number System 7 Solution: a First, make lists of the factors of 36 and 60, using sets. Let F36 and F60 be the sets of factors of 36 and 60, respectively. Then, F36 = {1, 2, 3, 4, 6, 9, 12, 18, 36} F60 = {1, 2, 3, 4, 5, 6, 10, 12, 15, 20, 30, 60} You can use the diagram to summarize the information. Notice that the common factors are shaded in green. They are 1, 2, 3, 4, 6 and 12 and the greatest is 12. i.e., F36 ∩ F60 = {1, 2, 3, 4, 6, 12} Therefore, GCF (36, 60) = 12. b Similarly, F32 = {1, 2, 4, 8, 16, 32} and F27 = {1, 3, 9, 27} Therefore, F32 ∩ F27 = {1} Thus, GCF (32, 27) = 1 Two or more natural numbers that have a GCF of 1 are called relatively prime. G r o u p W o r k 1 . G r o u p W o r k 1 . G r o u p W o r k 1 . G r o u p W o r k 1 . 2 2 2 2 Let a = 1800 and b = 756 1 Write: a the prime factorization of a and b b the prime factors that are common to both a and b. Now look at these common prime factors; the lowest powers of them (in the two prime factorizations) should be 22 and 32 . c What is the product of these lowest powers? d Write down the highest powers of the common prime factors. e What is the product of these highest powers? Definition 1.2 The greatest common factor (GCF) of two or more natural numbers is the greatest natural number that is a factor of all of the given numbers. Figure 1.3 F32 F27 2 4 8 16 32 3 9 27 1 Figure 1.2 F36 F60 9 18 36 5 10 15 20 30 60 1 2 3 4 6 12
  • 14. Mathematics Grade 9 8 2 a Compare the result of 1c with the GCF of the given numbers. Are they the same? b Compare the result of 1e with the GCF of the given numbers. Are they the same? The above Group Work leads you to another alternative method to find the GCF of numbers. This method (which is a quicker way to find the GCF) is called the prime factorization method. In this method, the GCF of a given set of numbers is the product of their common prime factors, each power to the smallest number of times it appears in the prime factorization of any of the numbers. Example 2 Use the prime factorization method to find GCF (180, 216, 540). Solution: Step 1 Express the numbers 180, 216 and 540 in their prime factorization. 180 = 22 × 32 × 5; 216 = 23 × 33 ; 540 = 22 × 33 × 5 Step 2 As you see from the prime factorizations of 180, 216 and 540, the numbers 2 and 3 are common prime factors. So, GCF (180, 216, 540) is the product of these common prime factors with the smallest respective exponents in any of the numbers. ∴ GCF (180, 216, 540) = 22 × 32 = 36. B Common multiples and the least common multiple G r o u p W o r k 1 . G r o u p W o r k 1 . G r o u p W o r k 1 . G r o u p W o r k 1 . 3 3 3 3 For this group work, you need 2 coloured pencils. Work with a partner Try this: ∗ ∗ ∗ ∗ List the natural numbers from 1 to 100 on a sheet of paper. ∗ ∗ ∗ ∗ Cross out all the multiples of 10. ∗ ∗ ∗ ∗ Using a different colour, cross out all the multiples of 8. Discuss: 1 Which numbers were crossed out by both colours? 2 How would you describe these numbers? 3 What is the least number crossed out by both colours? What do you call this number?
  • 15. Unit 1 The Number System 9 Example 3 Find LCM (8, 9). Solution: Let M8 and M9 be the sets of multiples of 8 and 9 respectively. M8 ={ } 8,16, 24, 32, 40, 48, 56, 64, , 80, 88 72 ,... M9 = { } 9,18, 27, 36, 45, 54, 63, , 81, 90 72 ,... Therefore LCM (8, 9) = 72 Prime factorization can also be used to find the LCM of a set of two or more than two numbers. A common multiple contains all the prime factors of each number in the set. The LCM is the product of each of these prime factors to the greatest number of times it appears in the prime factorization of the numbers. Example 4 Use the prime factorization method to find LCM (9, 21, 24). Solution: 9 = 3 × 3 = 32 21 = 3 × 7 24 = 2 × 2 × 2 ×3 = 23 × 3 Therefore, LCM (9, 21, 24) = 23 × 32 × 7 = 504. ACTIVITY 1.4 1 Find: a The GCF and LCM of 36 and 48 b GCF (36, 48) × LCM ( 36, 48) c 36 × 48 2 Discuss and generalize your results. For any natural numbers a and b, GCF (a, b) × LCM (a, b) = a × b. Definition 1.3 For any two natural numbers a and b, the least common multiple of a and b denoted by LCM (a, b), is the smallest multiple of both a and b. The prime factors that appear in these factorizations are 2, 3 and 7. Considering the greatest number of times each prime factor appears, we can get 23 , 32 and 7, respectively.
  • 16. Mathematics Grade 9 10 1.1.3 Rational Numbers H H H I I I S S S T T T O O O R R R I I I C C C A A A L L L N N N O O O T T T E E E : :: About 5,000 years ago, Egyptians used hieroglyphics to represent numbers. The Egyptian concept of fractions was mostly limited to fractions with numerator 1. The hieroglyphic was placed under the symbol to indicate the number as a denominator. Study the examples of Egyptian fractions. Recall that the set of integers is given by Z = {…, – 3, – 2, – 1, 0, 1, 2, 3,…} Using the set of integers, we define the set of rational numbers as follows: Through the following diagram, you can show how sets within rational numbers are related to each other. Note that natural numbers, whole numbers and integers are included in the set of rational numbers. This is because integers such as 4 and −7 can be written as 4 7 and 1 1 − . The set of rational numbers also includes terminating and repeating decimal numbers because terminating and repeating decimals can be written as fractions. Definition 1.4 Rational number Any number that can be expressed in the form a b , where a and b are integers and b ≠ 0, is called a rational number. The set of rational numbers, denoted by Q, is the set described by Q = : and are integers and 0 a a b b b   ≠     . 1,000 1 10 100 1 1 1 1 1 2 3 10 20 100 4 6 0 2 2 3 3 3 −8 −7 1 −9 3 1 2 − 2 5 Figure 1.4
  • 17. Unit 1 The Number System 11 For example, −1.3 can be written as 13 29 and 0.29 as 10 100 − − − . Mixed numbers are also included in the set of rational numbers because any mixed number 30 45 2 3 5 2 = = . 3 3 5 3 × × × × can be written as an improper fraction. For example, 2 2 3 can be written as 8 . 3 When a rational number is expressed as a fraction, it is often expressed in simplest form (lowest terms). A fraction a b is in simplest form when GCF (a, b) = 1. Example 1 Write 30 45 in simplest form. Solution: 30 45 2 3 5 2 = = . 3 3 5 3 × × × × (by factorization and cancellation) Hence 30 45 when expressed in lowest terms (simplest form) is 2 . 3 Exercise 1.1 1 Determine whether each of the following numbers is prime or composite: a 45 b 23 c 91 d 153 2 Prime numbers that differ by two are called twin primes. i Which of the following pairs are twin primes? a 3 and 5 b 13 and 17 c 5 and 7 ii List all pairs of twin primes that are less than 30. 3 Determine whether each of the following numbers is divisible by 2, 3, 4, 5, 6, 8, 9 or 10: a 48 b 153 c 2,470 d 144 e 12,357 4 a Is 3 a factor of 777? b Is 989 divisible by 9? c Is 2,348 divisible by 4? 5 Find three different ways to write 84 as a product of two natural numbers. 6 Find the prime factorization of: a 25 b 36 c 117 d 3,825
  • 18. Mathematics Grade 9 12 7 Is the value of 2a + 3b prime or composite when a = 11 and b = 7? 8 Write all the common factors of 30 and 42. 9 Find: a GCF (24, 36) b GCF (35, 49, 84) 10 Find the GCF of 2 × 33 × 52 and 23 × 3 × 52 . 11 Write three numbers that have a GCF of 7. 12 List the first six multiples of each of the following numbers: a 7 b 5 c 14 d 25 e 150 13 Find: a LCM (12, 16) b LCM (10, 12, 14) c LCM (15, 18) d LCM (7, 10) 14 When will the LCM of two numbers be the product of the numbers? 15 Write each of the following fractions in simplest form: a 3 9 − b 24 120 c 48 72 d 72 98 16 How many factors does each of the following numbers have? a 12 b 18 c 24 d 72 17 Find the value of an odd natural number x if LCM (x, 40) = 1400. 18 There are between 50 and 60 eggs in a basket. When Mohammed counts by 3’s, there are 2 eggs left over. When he counts by 5’s there are 4 left over. How many eggs are there in the basket? 19 The GCF of two numbers is 3 and the LCM is 180. If one of the numbers is 45, what is the other number? 20 i Let a, b, c, d be non-zero integers. Show that each of the following is a rational number: a a c b d + b a c b d − c a c b d × d a c b d ÷ What do you conclude from these results? ii Find two rational numbers between 1 3 and 3 4 .
  • 19. Unit 1 The Number System 13 1.2 THE REAL NUMBER SYSTEM 1.2.1 Representation of Rational Numbers by Decimals In this subsection, you will learn how to express rational numbers in the form of fractions and decimals. ACTIVITY 1.5 1 a What do we mean by a ‘decimal number’? b Give some examples of decimal numbers. 2 How do you represent 3 1 and 4 3 as decimals? 3 Can you write 0.4 and 1.34 as the ratio (or quotient) of two integers? Remember that a fraction is another way of writing division of one quantity by another. Any fraction of natural numbers can be expressed as a decimal by dividing the numerator by the denominator. Example 1 Show that 3 8 and 7 12 can each be expressed as a decimal. Solution: 3 8 means 3 ÷ 8 7 12 means 7 ÷ 12 0.375 0.5833 . . . 8 3.000 12 7.0000 24 60 60 100 56 96 40 40 40 36 0 40 36 4 ∴ 3 = 0.375 8 ∴ 7 12 = 0.5833 . . .
  • 20. Mathematics Grade 9 14 The fraction (rational number) 3 8 can be expressed as the decimal 0.375. A decimal like 0.375 is called a terminating decimal because the division ends or terminates, when the remainder is zero. The fraction 7 12 can be expressed as the decimal 0.58333… (Here, the digit 3 repeats and the division does not terminate.) A decimal like 0.58333... is called a repeating decimal. To show a repeating digit or a block of repeating digits in a repeating decimal number, we put a bar above the repeating digit (or block of digits). For example 0.58333… can be written as 0.583 , and 0.0818181… can be written as 0.081. This method of writing a repeating decimal is known as bar notation. The portion of a decimal that repeats is called the repetend. For example, In 0.583333… =0.583 , the repetend is 3. In 1.777… = 1.7 , the repetend is 7. In 0.00454545… = 0.0045 , the repetend is 45. To generalize: Any rational number a b can be expressed as a decimal by dividing the numerator a by the denominator b. When you divide a by b, one of the following two cases will occur. Case 1 The division process ends or terminates when a remainder of zero is obtained. In this case, the decimal is called a terminating decimal. Case 2 The division process does not terminate as the remainder never becomes zero. Such a decimal is called a repeating decimal. Expressing terminating and repeating decimals as fractions Every terminating decimal can be expressed as a fraction (a ratio of two integers) with a denominator of 10, 100, 1000 and so on. Example 2 Express each of the following decimals as a fraction in its simplest form (lowest terms): a 0.85 b 1.3456
  • 21. Unit 1 The Number System 15 Solution: a 100 85 17 0.85 0.85 100 100 20 = × = = (Why?) b 4 4 10000 10 13456 841 1.3456 1.3456 1.3456 10000 10 10000 625 = × = × = = If d is a terminating decimal number that has n digits after a decimal point, then we rewrite d as 10 10 n n d d × = The right side of the equation gives the fractional form of d. For example, if d = 2.128, then n = 3. ∴ 2.128 = 3 3 10 2.128 2128 266 = = 10 1000 125 × Repeating decimals can also be expressed as fractions (ratios of two integers). Example 3 Express each of the following decimals as a fraction (ratio of two integers): a 0.7 b 0.25 Solution: a Let 0.7 0.777 d = = … then, 10d = 7.777… (multiplying d by 10 because 1 digit repeats) Subtract d = 0.777… (to eliminate the repeating part 0.777…) 10d = 7.777… 1 1d = 0.777… 2 (d = 1d) 9d = 7 (subtracting expression 2 from expression 1) ∴ d = 7 9 (dividing both sides by 9) Hence 0. 7 7 = 9 b Let d = 0.25 = 0.252525… Then, 100d = 25.2525… (multiplying d by 100 because 2 digits repeat)
  • 22. Mathematics Grade 9 16 100d = 25.252525… 1d = 0.252525… 99d = 25 ∴ d = 25 99 So, 25 0.25 99 = In Example 3a, one digit repeats. So, you multiplied d by 10. In Example 3b, two digits repeat. So you multiplied d by 100. The algebra used in the above example can be generalized as follows: In general, if d is a repeating decimal with k non-repeating and p repeating digits after the decimal point, then the formula ( ) 10 10 10 10 k p k k p k d d + + − = − is used to change the decimal to the fractional form of d. Example 4 Express the decimal 0.375 as a fraction. Solution: Let 0.375 d = , then, k = 1 (number of non-repeating digits) p = 2 (number of repeating digits) and k + p = 1 + 2 = 3. ( ) ( ) ( ) ( ) 3 1 3 3 3 1 10 10 10 10 10 10 10 10 10 10 10 10 k p k k p k d d d d d + + − − − ⇒ = = = − − − 3 10 0.375 10 0.375 990 × − × = 375.75 3.75 372 990 990 − = = From Examples 1, 2, 3 and 4, you conclude the following: i Every rational number can be expressed as either a terminating decimal or a repeating decimal. ii Every terminating or repeating decimal represents a rational number. (subtracting 1d from 100d eliminates the repeating part 0.2525…)
  • 23. Unit 1 The Number System 17 Exercise 1.2 1 Express each of the following rational numbers as a decimal: a 4 9 b 3 25 c 11 7 d 2 5 3 − e 3706 100 f 22 7 2 Write each of the following as a decimal and then as a fraction in its lowest term: a three tenths b four thousandths c twelve hundredths d three hundred and sixty nine thousandths. 3 Write each of the following in metres as a fraction and then as a decimal: a 4 mm b 6 cm and 4 mm c 56 cm and 4 mm Hint: Recall that 1 metre(m) = 100 centimetres(cm) = 1000 millimetres(mm). 4 From each of the following fractions, identify those that can be expressed as terminating decimals: a 5 13 b 7 10 c 69 64 d 11 60 e 11 80 f 17 125 g 5 12 h 4 11 Generalize your observation. 5 Express each of the following decimals as a fraction or mixed number in simplest form: a 0.88 b 0.77 c 0.83 d 7.08 e 0.5252 f 1.003 − 6 Express each of the following decimals using bar notation: a 0.454545… b 0.1345345… 7 Express each of the following decimals without bar notation. (In each case use at least ten digits after the decimal point) a 0.13 b 0.305 − c 0.381 8 Verify each of the following computations by converting the decimals to fractions: a 0.275 0.714 0.989 + = b 0.6 1.142857 0.476190 − = −
  • 24. Mathematics Grade 9 18 1.2.2 Irrational Numbers Remember that terminating or repeating decimals are rational numbers, since they can be expressed as fractions. The square roots of perfect squares are also rational numbers. For example, 4 is a rational number since 2 4 = 2 = 1 . Similarly, 0.09 is a rational number because, 0.09 0.3 = is a rational number. If x2 = 4, then what do you think is the value of x? 4 2. x = ± = ± Therefore x is a rational number. What if x2 = 3? In Figure 1.4 of Section 1.1.3, where do numbers like 2 and 5 fit? Notice what happens when you find 2 and 5 with your calculator: If you first press the button 2 and then the square-root button, you will find 2 on the display. i.e., 2 : 2 √ = 1.414213562… 5 : 5 √ = 2.236067977… Note that many scientific calculators, such as Casio ones, work the same as the written order, i.e., instead of pressing 2 and then the √ button, you press the √ button and then 2. Before using any calculator, it is always advisable to read the user’s manual. Note that the decimal numbers for 2 and 5 do not terminate, nor do they have a pattern of repeating digits. Therefore, these numbers are not rational numbers. Such numbers are called irrational numbers. In general, if a is a natural number that is not a perfect square, then a is an irrational number. Example 1 Determine whether each of the following numbers is rational or irrational. a 0.16666 . . . b 0.16116111611116111116 . . . c π Solution: a In 0.16666 . . . the decimal has a repeating pattern. It is a rational number and can be expressed as 1 6 . b This decimal has a pattern that neither repeats nor terminates. It is an irrational number. Study Hint Most calculators round answers but some truncate answers. i.e., they cut off at a certain point, ignoring subsequent digits.
  • 25. Unit 1 The Number System 19 c π = 3.1415926… This decimal does not repeat or terminate. It is an irrational number. (The fraction 22 7 is an approximation to the value of π. It is not the exact value!). In Example 1, b and c lead us to the following fact: A decimal number that is neither terminating nor repeating is an irrational number. 1 Locating irrational numbers on the number line G r o u p W o r k 1 . G r o u p W o r k 1 . G r o u p W o r k 1 . G r o u p W o r k 1 . 4 4 4 4 You will need a compass and straight edge to perform the following: 1 To locate 2 on the number line: Draw a number line. At the point corresponding to 1 on the number line, construct a perpendicular line segment 1 unit long. Draw a line segment from the point corresponding to 0 to the top of the 1 unit segment and label it as c. Use the Pythagorean Theorem to show that c is 2 unit long. Open the compass to the length of c. With the tip of the compass at the point corresponding to 0, draw an arc that intersects the number line at B. The distance from the point corresponding to 0 to B is 2 units. 2 To locate 5 on the number line: Find two numbers whose squares have a sum of 5. One pair that works is 1 and 2, since 12 + 22 = 5. Draw a number line. At the point corresponding to 2, on the number line, construct a perpendicular line segment 1 unit long. Draw the line segment shown from the point corresponding to 0 to the top of the 1 unit segment. Label it as c. 0 1 2 3 Figure 1.6 c 1 2 0 1 2 c 1 1 B Figure 1.5
  • 26. Mathematics Grade 9 20 The Pythagorean theorem can be used to show that c is 5 units long. c2 = 12 + 22 = 5 c = 5 Open the compass to the length of c. With the tip of the compass at the point corresponding to 0, draw an arc intersecting the number line at B. The distance from the point corresponding to 0 to B is 5 units. ACTIVITY 1.6 1 Locate each of the following on the number line, by using geometrical construction: a 3 b – 2 c 6 2 Explain how 2 can be used to locate: a 3 b 6 3 Locate each of the following on the number line: a 1 + 2 b –2 + 2 c 3 – 2 Example 2 Show that 3 2 + is an irrational number. Solution: To show that 3 2 + is not a rational number, let us begin by assuming that 3 2 + is rational. i.e., 3 2 a b + = where a and b are integers, b ≠ 0. Then 3 2 3 . a a b b b − = − = Since a – 3b and b are integers (Why?), 3 a b b − is a rational number, meaning that 3 2 a b b −   =     is rational, which is false. As the assumption that 3 2 + is rational has led to a false conclusion, the assumption must be false. Therefore, 3 2 + is an irrational number. Definition 1.5 Irrational number An irrational number is a number that cannot be expressed as a b , such that a and b are integers and b ≠ 0. 0 1 2 3 5 c 1 B Figure 1.7
  • 27. Unit 1 The Number System 21 ACTIVITY 1.7 Evaluate the following: 1 0.3030030003 . . . + 0.1414414441 . . . 2 0.5757757775 . . . – 0.242442444 . . . 3 ( ) ( ) 3 + 2 3 2 × − 4 12 3 ÷ From Example 2 and Activity 1.7, you can generalize the following facts: i The sum of any rational number and an irrational number is an irrational number. ii The set of irrational numbers is not closed with respect to addition, subtraction, multiplication and division. iii If p is a positive integer that is not a perfect square, then a + b p is irrational where a and b are integers and b ≠ 0. For example, 3 2 + and 2 2 3 − are irrational numbers. Exercise 1.3 1 Identify each of the following numbers as rational or irrational: a 5 6 b 2.34 c 0.1213141516... − d 0.81 e 0.121121112... f 5 2 − g 3 72 h 1 + 3 2 Give two examples of irrational numbers, one in the form of a radical and the other in the form of a non-terminating decimal. 3 For each of the following, decide whether the statement is 'true' or 'false'. If your answer is 'false', give a counter example to justify. a The sum of any two irrational numbers is an irrational number. b The sum of any two rational numbers is a rational number. c The sum of any two terminating decimals is a terminating decimal. d The product of a rational number and an irrational number is irrational.
  • 28. Mathematics Grade 9 22 1.2.3 Real Numbers In Section 1.2.1, you observed that every rational number is either a terminating decimal or a repeating decimal. Conversely, any terminating or repeating decimal is a rational number. Moreover, in Section 1.2.2 you learned that decimals which are neither terminating nor repeating exist. For example, 0.1313313331… Such decimals are defined to be irrational numbers. So a decimal number can be a rational or an irrational number. It can be shown that every decimal number, be it rational or irrational, can be associated with a unique point on the number line and conversely that every point on the number line can be associated with a unique decimal number, either rational or irrational. This is usually expressed by saying that there exists a one-to-one correspondence between the sets C and D where these sets are defined as follows. C = {P : P is a point on the number line} D = {d : d is a decimal number } The above discussion leads us to the following definition. The set of real numbers and its subsets are shown in the adjacent diagram. From the preceding discussion, you can see that there exists a one-to-one correspondence between the set R and the set C = {P:P is a point on the number line}. Definition 1.6 Real numbers A number is called a real number, if and only if it is either a rational number or an irrational number. The set of real numbers, denoted by R, can be described as the union of the sets of rational and irrational numbers. R = { x : x is a rational number or an irrational number.} –4 –1 Whole Numbers 0 1 2 Integers -2 Rational Numbers . . 0 7 0 4 3 5 19 4 − . . . . 2 1 0100100010 5 π − Irrational Numbers Real Numbers Figure 1.8
  • 29. Unit 1 The Number System 23 It is good to understand and appreciate the existence of a one-to-one correspondence between any two of the following sets. 1 D = {x : x is a decimal number} 2 P = {x : x is a point on the number line} 3 R = {x : x is a real number} Since all real numbers can be located on the number line, the number line can be used to compare and order all real numbers. For example, using the number line you can tell that 3 0, 2 2. − Example 1 Arrange the following numbers in ascending order: 5 3 , 0.8, . 6 2 Solution: Use a calculator to convert 5 6 and 3 2 to decimals 5 ÷ 6 = 0.83333... and 3 2 = 0.866025... ÷ Since 0.8 0.83 0.866025..., the numbers when arranged in ascending order are 5 3 0.8, , . 6 2 However, there are algebraic methods of comparing and ordering real numbers. Here are two important properties of order. 1 Trichotomy property For any two real numbers a and b, one and only one of the following is true a b or a = b or a b. 2 Transitive property of order For any three real numbers a, b and c, if a b and b c, then, a c.
  • 30. Mathematics Grade 9 24 A third property, stated below, can be derived from the Trichotomy Property and the Transitive Property of Order. For any two non-negative real numbers a and b, if a2 b2 , then a b. You can use this property to compare two numbers without using a calculator. For example, let us compare 5 6 and 3 . 2 2 2 5 25 3 3 27 = , = = 6 36 2 4 36               Since 2 2 5 3 , 6 2               it follows that 5 3 . 6 2 Exercise 1.4 1 Compare the numbers a and b using the symbol or . a 6 , 0.6 4 a b = = b a = 0.432, b = 0.437 c a = – 0.128, b = – 0.123 2 State whether each set (a – e given below) is closed under each of the following operations: i addition ii subtraction iii multiplication iv division a N the set of natural numbers. b Z the set of integers. c Q the set of rational numbers. d The set of irrational numbers. e R the set of real numbers. 1.2.4 Exponents and Radicals A Roots and radicals In this subsection, you will define the roots and radicals of numbers and discuss their properties. Computations of expressions involving radicals and fractional exponents are also considered.
  • 31. Unit 1 The Number System 25 Roots H H H I I I S S S T T T O O O R R R I I I C C C A A A L L L N N N O O O T T T E E E : :: The Pythagorean School of ancient Greece focused on the study of philosophy, mathematics and natural science. The students, called Pythagoreans, made many advances in these fields. One of their studies was to symbolize numbers. By drawing pictures of various numbers, patterns can be discovered. For example, some whole numbers can be represented by drawing dots arranged in squares. • 1 1 × 1 • • • • 4 2 × 2 • • • • • • • • • 9 3 × 3 • • • • • • • • • • • • • • • • 16 4 × 4 • • • • • • • • • • • • • • • • • • • • • • • • • • • • • • • • • • • • • • • • • • • • • • • • • • • • • • • • • • • • • • • • 64 8 × 8 Numbers that can be pictured in squares of dots are called perfect squares or square numbers. The number of dots in each row or column in the square is a square root of the perfect square. The perfect square 9 has a square root of 3, because there are 3 rows and 3 columns. You say 8 is a square root of 64, because 64 = 8 × 8 or 82 . Perfect squares also include decimals and fractions like 0.09 and 4 . 9 Since ( ) 2 0.3 0.09 = and 2 2 4 = 3 9       , it is also true that (–8)2 = 64 and (–12)2 = 144. So, you may say that –8 is also a square root of 64 and –12 is a square root of 144. The positive square root of a number is called the principal square root. The symbol , called a radical sign, is used to indicate the principal square root. Definition 1.7 Square root For any two real numbers a and b, if a2 = b, then a is a square root of b.
  • 32. Mathematics Grade 9 26 The symbol 25 is read as the principal square root of 25 or just the square root of 25 and 25 − is read as the negative square root of 25. If b is a positive real number, b is a positive real number. Negative real numbers do not have square roots in the set of real numbers since a2 ≥ 0 for any number a. The square root of zero is zero. Similarly, since 43 = 64, you say that 64 is the cube of 4 and 4 is the cube root of 64. That is written as 3 4 64 = . The symbol 3 4 64 = is read as the principal cube root of 64 or just the cube root of 64. Each real number has exactly one cube root. (– 3)3 = – 27 so, 3 27 = − −3 03 = 0 so, 3 0 0. = You may now generalize as follows: Example 1 a – 3 is a cube root of – 27 because (– 3)3 = – 27 b 4 is a cube root of 64 because 43 = 64 i If b 0 and n is even, there is no real nth root of b, because an even power of any real number is a non-negative number. ii The symbol n is called a radical sign, the expression n b is called a radical, n is called the index and b is called the radicand. When no index is written, the radical sign indicates square root. Definition 1.9 Principal nth root If b is any real number and n is a positive integer greater than 1, then, the principal nth root of b, denoted by n b is defined as the positive root of , if 0. the negative root of , if 0 and is odd. 0, if 0. th th n n b b b n b b n b   =   =  Definition 1.8 The nth root For any two real numbers a and b, and positive integer n, if an = b, then a is called an nth root of b.
  • 33. Unit 1 The Number System 27 Example 2 a 4 16 = 2 because 24 = 16 b 0.04 = 0.2 because (0. 2)2 = 0.04 c 3 1000 = 10 − − because (– 10)3 = – 1000 Numbers such as 3 3 23, 35 and 10 are irrational numbers and cannot be written as terminating or repeating decimals. However, it is possible to approximate irrational numbers as closely as desired using decimals. These rational approximations can be found through successive trials, using a scientific calculator. The method of successive trials uses the following property: For any three positive real numbers a, b and c and a positive integer n if an b cn , then n a b c . Example 3 Find a rational approximation of 43 to the nearest hundredth. Solution: Use the above property and divide-and-average on a calculator. Since 62 = 36 43 49 = 72 6 43 7 Estimate 43 to tenths, 43 6.5 ≈ Divide 43 by 6.5 6.615 6.5 43.000 Average the divisor and the quotient 6.5 + 6.615 = 6.558 2 Divide 43 by 6.558 6.557 6.558 43.000 Now you can check that (6.557)2 43 (6.558)2 . Therefore 43 is between 6.557 and 6.558. It is 6.56 to the nearest hundredth. Example 4 Through successive trials on a calculator, compute 3 53 to the nearest tenth.
  • 34. Mathematics Grade 9 28 Solution: 33 = 27 53 64 = 43 . That is, 33 53 43 . So 3 3 53 4 Try 3.5: 3.53 = 42.875 So 3.5 3 53 4 Try 3.7: 3.73 = 50.653 So 3.7 3 53 4 Try 3.8: 3.83 = 54.872 So 3.7 3 53 3.8 Try 3.75: 3.753 = 52.734375 So 3.75 3 53 3.8 Therefore, 3 53 is 3.8 to the nearest tenth. B Meaning of fractional exponents ACTIVITY 1.8 1 State another name for 1 4 2 . 2 What meaning can you give to 1 2 2 or 0.5 2 ? 3 Show that there is at most one positive number whose fifth root is 2. By considering a table of powers of 3 and using a calculator, you can define 1 5 3 as 5 3 . This choice would retain the property of exponents by which 5 1 1 5 5 5 3 3 3.   ×       = =     Similarly, you can define 1 5n , where n is a positive integer greater than 1, as 5. n In general, you can define 1 n b for any b∈ℝ and n a positive integer to be n b whenever n b is a real number. Example 5 Write the following in exponential form: a 7 b 3 1 10 Solution: a 1 2 7 7 = b 1 3 1 3 3 1 1 10 10 10 − = = Definition 1.10 The nth power If b ∈ R and n is a positive integer greater than 1, then 1 n n b b =
  • 35. Unit 1 The Number System 29 Example 6 Simplify: a 1 2 25 b 1 3 ( 8) − c 1 6 64 Solution: a 1 2 25 25 5 = = (Since 52 = 25) b 1 3 3 ( 8) 8 2 − = − = − (Since (−2)3 = −8) c 1 6 6 64 64 2 = = (Since 26 = 64) G r o u p W o r k 1 . G r o u p W o r k 1 . G r o u p W o r k 1 . G r o u p W o r k 1 . 5 5 5 5 Simplify: i a ( ) 1 3 8 27 × b 1 1 3 3 8 27 × ii a 3 8 27 × b 3 3 8 27 × iii a ( ) 1 2 36 49 × b 1 1 2 2 36 49 × iv a 36 49 × b 36 49 × What relationship do you observe between a and b in i, ii, iii and iv? The observations from the above Group Work lead you to think that ( ) 1 1 1 3 3 3 5 3 5 3 × = × . This particular case suggests the following general property (Theorem). Example 7 Simplify each of the following. a 1 1 3 3 9 3 × b 5 5 16 2 × Solution: a ( ) 1 1 1 3 3 3 9 3 9 3 × = × (by Theorem 1.2) = ( ) 1 3 27 (multiplication) = 3 (33 = 27) b 5 5 5 16 2 16 2 × = × = 5 32 = 2 Theorem 1.2 For any two real numbers a and b and for all integers 2 n ≥ , ( ) 1 1 1 n n n a b ab =
  • 36. Mathematics Grade 9 30 ACTIVITY 1.9 Simplify: i a 1 5 1 5 64 2 b 1 5 64 2       ii a 1 2 1 2 8 2 b 1 2 8 2       iii a 1 3 1 3 27 729 b 1 3 27 729       What relationship do you observe between a and b in i, ii and iii? The observations from the above Activity lead us to the following theorem: Example 8 Simplify a 1 3 1 16 23 b 6 6 128 2 Solution: a 1 1 3 16 2 3 = 16 2 1 3       (by Theorem 1.3) = 1 3 8 = 2 (since 23 = 8) b 6 6 6 128 128 2 2 = = 6 64 = 2 (because 26 = 64) Theorem 1.3 For any two real numbers a and b where b ≠ 0 and for all integers n ≥ 2, 1 1 1 n n n a a b b   =    
  • 37. Unit 1 The Number System 31 ACTIVITY 1.10 1 Suggest, with reasons, a meaning for a 2 7 2 b 2 9 2 in terms of 2 1 2 2 Suggest a relation between 5 3 2 and 5 1 2 . Applying the property (am )n = amn , you can write 9 1 10 7       as 9 10 7 . In general, you can say 1 p q a         = p q a , where p and q are positive integers and a ≥ 0. Thus, you have the following definition: Exercise 1.5 1 Show that: a 1 3 64 4 = b 1 8 256 2 = c 1 3 125 5 = 2 Express each of the following without fractional exponents and without radical signs: a 1 4 81 b 1 2 9 c 6 64 d 1 3 27 8       e ( ) 1 5 0.00032 f 4 0.0016 g 6 729 3 Explain each step of the following: ( ) ( ) ( ) 1 1 3 3 27 125 3 3 3 5 5 5 × = × × × × ×     = ( ) ( ) ( ) 1 3 3 5 3 5 3 5 × × × × ×     3 5 15 = × = 4 In the same manner as in Question 3, simplify each of the following: a ( ) 1 2 25 121 × b ( ) 1 4 625 16 × c ( ) 1 5 1024 243 × 5 Express Theorem 1.2 using radical notation. 6 Show that: a ( ) 1 1 1 4 4 4 7 5 7 5 × = × b 5 3 5 3 × = × c 3 3 3 7 9 7 9 × = × d ( ) 1 1 1 7 7 7 11 6 11 6 × = × Definition 1.11 For a ≥ 0 and p and q any two positive integers, ( ) 1 = p p p q q q a a a   =      
  • 38. Mathematics Grade 9 32 7 Express in the simplest form: a 1 1 6 6 32 2 × b 1 1 3 3 9 3 × c 1 1 6 6 1 128 2   ×    d 5 5 16 2 × e 3 3 16 4 × f 1 1 7 7 32 4 × g 1 1 1 1 8 8 5 5 1 5 27 9 5   × × ×     h 5 3 5 3 1 5 8 4 5 × × × 8 Express Theorem 1.3 using radical notation. 9 Simplify: a 1 5 1 5 128 4 b 1 3 1 3 9 243 c 1 4 1 16 814 d 1 4 1 32 1624 e 3 3 16 2 f 5 5 64 2 g 6 6 512 8 h 3 3 625 5 10 Rewrite each of the following in the form p q a : a 9 1 5 13       b 11 1 5 12       c 5 1 6 11       11 Rewrite the following in the form 1 p q a         a 7 5 3 b 6 3 5 c 5 6 64 d 2 3 729 12 Rewrite the expressions in Question 10 using radicals. 13 Rewrite the expressions in Question 11 using radicals. 14 Express the following without fractional exponents or radical sign: a 5 1 3 27       b 5 3 27 c 1 3 8 15 Simplify each of the following: a 1 6 64 b 3 2 81 c 3 18 64 d 6 4 81 e 2 3 512 f 6 9 512
  • 39. Unit 1 The Number System 33 C Simplification of radicals ACTIVITY 1.11 1 Evaluate each of the following and discuss your result in groups. a 3 3 ( 2) − b 2 ( 3) − c 4 4 ( 5) − d 5 5 4 e 2 2 f 7 7 ( 1) − 2 Does the sign of your result depend on whether the index is odd or even? Can you give a general rule for the result of n n a where a is a real number and i n is an odd integer? ii n is an even integer? To compute and simplify expressions involving radicals, it is often necessary to distinguish between roots with odd indices and those with even indices. For any real number a and a positive integer n, = n n a a , if n is odd. = n n a a , if n is even. ( ) 5 5 2 = 2 − − , 3 3 , x x = ( ) 2 2 = 2 = 2 − − 2 , x x = ( ) 4 4 2 = 2 = 2 − − , 4 4 x x = Example 9 Simplify each of the following: a 2 y b 3 3 27x − c 4 25x d 6 6 x e 3 4 x Solution: a 2 y y = b ( ) 3 3 3 3 27 3 3 x x x − = − = − c 4 2 25 5 x x = = 5x2 d 6 6 x x = e ( ) 3 1 3 3 4 4 4 x x x = = A radical n a is in simplest form, if the radicand a contains no factor that can be expressed as an nth power. For example 3 54 is not in simplest form because 33 is a factor of 54. Using this fact and the radical notations of Theorem 1.2 and Theorem 1.3, you can simplify radicals.
  • 40. Mathematics Grade 9 34 Example 10 Simplify each of the following: a 48 b 3 3 9 3 × c 4 32 81 Solution: a 48 16 3 16 3 4 3 = × = × = b 3 3 3 3 9 3 9 3 27 3 × = × = = c 4 4 4 4 4 4 4 4 32 16 2 16 16 2 2 2 2 81 81 81 3 81 × = = × = × = Exercise 1.6 1 Simplify each of the following: a 8 b 5 32 c 2 3 8 x d 363 e 3 512 f 3 2 1 27 3 x y g 4 405 2 Simplify each of the following if possible. State restrictions where necessary. a 50 b 2 36 c 1 72 3 d 2 3 8x e 3 a f 0.27 g 63 − h 180 9 i 3 16 j 3 54 − 3 Identify the error and write the correct solution each of the following cases: a A student simplified 28 to 25+3 and then to 5 3 b A student simplified 72 to 4 18 and then to 4 3 c A student simplified 9 7x and got 3 7 x 4 Simplify each of the following: a 8 250 b 3 3 16 5 × c 4 4 5 125 × d 2 7 14 7 × × e 3 3 81 3 f 12 96 3 6 g 3 2 2 98 0, 0. 14 x y x y xy h 4 3 2 18 ×
  • 41. Unit 1 The Number System 35 5 The number of units N produced by a company from the use of K units of capital and L units of labour is given by 12 . N LK = a What is the number of units produced, if there are 625 units of labour and 1024 units of capital? b Discuss the effect on the production, if the units of labour and capital are doubled. Addition and subtraction of radicals Which of the following do you think is correct? 1 2 + 8 = 10 2 19 3 =4 − 3 5 2 7 2 12 2 + = The above problems involve addition and subtraction of radicals. You define below the concept of like radicals which is commonly used for this purpose. For example, i 1 3 5, 5 and 5 are like radicals. 2 − are like radicals. ii 3 5 and 5 are not like radicals. iii 11 and 7 are not like radicals. By treating like radicals as like terms, you can add or subtract like radicals and express them as a single radical. On the other hand, the sum of unlike radicals cannot be expressed as a single radical unless they can be transformed into like radicals. Example 11 Simplify each of the following: a 2 8 + b 1 1 3 12 3 2 27 3 9 − + + Solution: a 2 8 2 2 4 2 4 2 2 2 2 + = + × = + = + (1 2) 2 3 2 = + = Definition 1.12 Radicals that have the same index and the same radicand are said to be like radicals.
  • 42. Mathematics Grade 9 36 b 1 1 1 3 1 3 12 3 2 27 3 4 3 3 2 9 3 3 9 3 3 9 − + + = × − + × + × = 3 1 3 4 3 3 2 9 3 9 9 × − + + × = 2 1 6 3 3 3 3 3 3 − + + = 2 1 6 1 3 6 3 3 3   − + + =     Exercise 1.7 Simplify each of the following if possible. State restrictions where necessary. 1 a 2 5 × b 3 6 × c 21 5 × d 2 8 x x × e 2 2 f 10 4 3 g 3 50 2 y y ÷ h 9 40 3 10 i 3 3 4 16 2 2 ÷ j 9 24 15 75 3 3 ÷ 2 a 2 3 5 3 + b 9 2 5 2 − c 3 12 + d 63 28 − e 75 48 − f ( ) 6 12 3 − g 2 2 2 50 x x − h 3 3 5 54 2 2 − i 2 8 24 54 2 96 3 + − j 2 a ab b a b + + + k ( ) 1 1 a b a b   − +     3 a Find the square of 7 2 10. − b Simplify each of the following: i 5 2 6 5 2 6 + − − ii 7 24 7 24 2 2 + − + iii ( )( ) 2 2 2 2 1 1 1 1 p p p p + − − − + + 4 Suppose the braking distance d for a given automobile when it is travelling v km/hr is approximated by 3 5 0.00021 d v = m. Approximate the braking distance when the car is travelling 64 km/hr.
  • 43. Unit 1 The Number System 37 1.2.5 The Four Operations on Real Numbers The following activity is designed to help you revise the four operations on the set of rational numbers which you have done in your previous grades. ACTIVITY 1.12 1 Apply the properties of the four operations in the set of rational numbers to compute the following (mentally, if possible). a 2 3 7 9 5 9   + +     b 3 11 3 11 7 21 7 21 − − −      × +           c 3 5 3 7 6 7 −   + +     d 9 23 7 7 27 9 − −     × ×     −     2 State a property that justifies each of the following statements. a 2 3 3 2 3 3 3 2 5 3 2 5 − −     × = × ×         b 7 3 4 7 4 3 9 2 5 9 5 2 − − − −     + = +         c 3 5 1 5 , 5 6 5 6 − − − −         + +                 since 3 1 5 5 − − In this section, you will discuss operations on the set of real numbers. The properties you have studied so far will help you to investigate many other properties of the set of real numbers. G r o u p W o r k 1 . G r o u p W o r k 1 . G r o u p W o r k 1 . G r o u p W o r k 1 . 6 6 6 6 Work with a partner Required:- scientific calculator 1 Try this Copy and complete the following table. Then use a calculator to find each product and complete the table. Factors product product written as a power 23 × 22 101 × 101 3 1 1 5 5 − −     ×        
  • 44. Mathematics Grade 9 38 2 Try this Copy the following table. Use a calculator to find each quotient and complete the table. Division Quotient Quotient written as a power 105 ÷ 101 35 ÷ 32 4 2 1 1 2 2     ÷         Discuss the two tables: i a Compare the exponents of the factors to the exponents in the product. What do you observe? b Write a rule for determining the exponent of the product when you multiply powers. Check your rule by multiplying 32 × 33 using a calculator. ii a Compare the exponents of the division expressions to the exponents in the quotients. What pattern do you observe? b Write a rule for determining the exponent in the quotient when you divide powers. Check your rule by dividing 75 by 73 on a calculator. 3 Indicate whether each statement is false or true. If false, explain: a Between any two rational numbers, there is always a rational number. b The set of real numbers is the union of the set of rational numbers and the set of irrational numbers. c The set of rational numbers is closed under addition, subtraction, multiplication and division excluding division by zero. d The set of irrational numbers is closed under addition, subtraction, multiplication and division. 4 Give examples to show each of the following: a The product of two irrational numbers may be rational or irrational. b The sum of two irrational numbers may be rational or irrational. c The difference of two irrational numbers may be rational or irrational. d The quotient of two irrational numbers may be rational or irrational. 5 Demonstrate with an example that the sum of an irrational number and a rational number is irrational. 6 Demonstrate with an example that the product of an irrational number and a non- zero rational number is irrational.
  • 45. Unit 1 The Number System 39 7 Complete the following chart using the words ‘yes’ or ‘no’. Number Rational number Irrational number Real number 2 3 2 3 − 3 2 1.23 1.20220222… 2 1.23 3 − × 75 1.23 + 75 − 3 1.20220222… + 0.13113111… Questions 3, 4, 5 and in particular Question 7 of the above Group Work lead you to conclude that the set of real numbers is closed under addition, subtraction, multiplication and division, excluding division by zero. You recall that the set of rational numbers satisfy the commutative, associative and distributive laws for addition and multiplication. If you add, subtract, multiply or divide (except by 0) two rational numbers, you get a rational number, that is, the set of rational numbers is closed with respect to addition, subtraction, multiplication and division. From Group work 1.6 you may have realized that the set of irrational numbers is not closed under all the four operations, namely addition, subtraction, multiplication and division. Do the following activity and discuss your results. ACTIVITY 1.13 1 Find a + b, if a 3 2 and 3 2 a b = + = − b 3 3 and 2 3 a b = + = +
  • 46. Mathematics Grade 9 40 2 Find a – b, if a 3 and 3 a b = = b 5 and 2 a b = = 3 Find ab, if a 3 1 and 3 1 a b = − = + b 2 3 and 3 2 a b = = 4 Find a ÷ b, if a 5 2 and 3 2 a b = = b 6 6 and 2 3 a b = = Let us see some examples of the four operations on real numbers. Example 1 Add 2 3 3 2 a = + and 2 3 3 − Solution ( ) ( ) 2 3 3 2 2 3 3 + + − = 2 3 3 2 2 3 3 + + − = ( ) ( ) 3 2 3 2 3 1 − + + = 3 4 2 − + Example 2 Subtract 3 2 5 + from 3 5 2 2 − Solution: ( ) ( ) 3 5 2 2 3 2 5 3 5 2 2 3 2 5 − − + = − − − = 5(3 1) 2( 2 3) − + − − = 2 5 5 2 − Example 3 Multiply a 2 3 by 3 2 b 2 5 by 3 5 Solution: a 2 3 3 2 6 6 × = b ( ) 2 2 5 3 5 2 3 5 30 × = × × = Example 4 Divide a 8 6 by 2 3 b ( ) 12 6 by 2 3 × Solution: a 8 6 8 6 2 3 2 3 ÷ = 8 6 4 2 2 3 = × = b ( ) 12 6 12 6 2 3 2 3 ÷ × = × 12 6 12 6 = =
  • 47. Unit 1 The Number System 41 Rules of exponents hold for real numbers. That is, if a and b are nonzero numbers and m and n are real numbers, then whenever the powers are defined, you have the following laws of exponents. 1 m n m n a a a + × = 2 ( ) n m mn a a = 3 m m n n a a a − = 4 ( ) n n n a b ab × = 5 , 0. n n n a a b b b   = ≠     ACTIVITY 1.14 1 Find the additive inverse of each of the following real numbers: a 5 b 1 2 − c 2 1 + d 2.45 e 2.1010010001. . . 2 Find the multiplicative inverse of each of the following real numbers: a 3 b 5 c 1 3 − d 1 6 2 e 1.71 f 2 3 g 1.3 3 Explain each of the following steps: ( ) ( ) 3 3 6 2 15 20 6 2 15 20 3 3 − × + = × − + 3 3 6 2 15 20 3 3   = × − × +       18 2 45 = 20 3 3   − +       9 2 2 9 5 = 20 3 3   × × − +       3 2 2 3 5 = 20 3 3   × × × − +       ( ) = 2 2 5 20 − + ( ) = 2 2 5 2 5   + − +   = 2
  • 48. Mathematics Grade 9 42 Let us now examine the basic properties that govern addition and multiplication of real numbers. You can list these basic properties as follows: Closure property: The set R of real numbers is closed under addition and multiplication. This means that the sum and product of two real numbers is a real number; that is, for all a, b ∈ R, a + b ∈ R and ab ∈ R Addition and multiplication are commutative in ℝ: That is, for all a, b ∈ R, i a + b = b + a ii ab = ba Addition and multiplication are associative in ℝ: That is, for all, a, b, c ∈ R, i (a + b) + c = a + ( b + c) ii (ab)c = a(bc) Existence of additive and multiplicative identities: There are real numbers 0 and 1 such that: i a + 0 = 0 + a = a, for all a ∈ R. ii a . 1 = 1 . a = a, for all a ∈ R. Existence of additive and multiplicative inverses: i For each a ∈ R there exists –a ∈ R such that a + (–a) = 0 = (–a) + a, and –a is called the additive inverse of a. ii For each non-zero a ∈ R, there exists 1 a ∈ℝ such that 1 1 1 , a a a a × = = ×             and 1 a is called the multiplicative inverse or reciprocal of a. Distributive property: Multiplication is distributive over addition; that is, if a, b, c, ∈ R then: i a ( b + c) = ab + ac ii (b + c) a = ba + ca
  • 49. Unit 1 The Number System 43 Exercise 1.8 1 Find the numerical value of each of the following: a ( ) ( ) ( ) 3 4 5 3 1 5 2 2 1 4 2 8 64 16 − −   × × × ×     b 176 2 275 1584 891 − + − c ( ) 3 5 1 15 1.04 5 6 5 0.02 300 5 9 18 − + − − d 5 4 0.0001 0.00032 − e 3 4 2 0.125 0.0016 + 2 Simplify each of the following a ( ) 1 3 216 b 2 3 3 5 2 2 × c 5 1 2 3       d 3 4 1 4 7 49 e 1 1 8 4 3 25 × f 1 4 16 2 ÷ g 4 3 7 h 5 5 32 243 3 What should be added to each of the following numbers to make it a rational number? (There are many possible answers. In each case, give two answers.) a 5 3 − b 2 5 − − c 4.383383338… d 6.123456… e 10.3030003… 1.2.6 Limits of Accuracy In this subsection, you shall discuss certain concepts such as approximation, accuracy in measurements, significant figures (s.f), decimal places (d.p) and rounding off numbers. In addition to this, you shall discuss how to give appropriate upper and lower bounds for data to a specified accuracy (for example measured lengths). ACTIVITY 1.15 1 Round off the number 28617 to the nearest a 10,000 b 1000 c 100 2 Write the number i 7.864 ii 6.437 iii 4.56556555… a to one decimal place b to two decimal places 3 Write the number 43.25 to a two significant figures b three significant figures 4 The weight of an object is 5.4 kg. Give the lower and upper bounds within which the weight of the object can lie.
  • 50. Mathematics Grade 9 44 1 Counting and measuring Counting and measuring are an integral part of our daily life. Most of us do so for various reasons and at various occasions. For example you can count the money you receive from someone, a tailor measures the length of the shirt he/she makes for us, and a carpenter counts the number of screws required to make a desk. Counting: The process of counting involves finding out the exact number of things. For example, you do counting to find out the number of students in a class. The answer is an exact number and is either correct or, if you have made a mistake, incorrect. On many occasions, just an estimate is sufficient and the exact number is not required or important. Measuring: If you are finding the length of a football field, the weight of a person or the time it takes to walk down to school, you are measuring. The answers are not exact numbers because there could be errors in measurements. 2 Estimation In many instances, exact numbers are not necessary or even desirable. In those conditions, approximations are given. The approximations can take several forms. Here you shall deal with the common types of approximations. A Rounding If 38,518 people attend a football game this figure can be reported to various levels of accuracy. To the nearest 10,000 this figure would be rounded up to 40,000. To the nearest 1000 this figure would be rounded up to 39,000. To the nearest 100 this figure would be rounded down to 38,500 In this type of situation, it is unlikely that the exact number would be reported. B Decimal places A number can also be approximated to a given number of decimal places (d.p). This refers to the number of figures written after a decimal point. Example 1 a Write 7.864 to 1 d.p. b Write 5.574 to 2 d.p. Solution: a The answer needs to be written with one number after the decimal point. However, to do this, the second number after the decimal point also needs to be considered. If it is 5 or more, then the first number is rounded up. That is 7.864 is written as 7.9 to 1 d.p
  • 51. Unit 1 The Number System 45 b The answer here is to be given with two numbers after the decimal point. In this case, the third number after the decimal point needs to be considered. As the third number after the decimal point is less than 5, the second number is not rounded up. That is 5.574 is written as 5.57 to 2 d.p. Note that to approximate a number to 1 d.p means to approximate the number to the nearest tenth. Similarly approximating a number to 2 decimal places means to approximate to the nearest hundredth. C Significant figures Numbers can also be approximated to a given number of significant figures (s.f). In the number 43.25 the 4 is the most significant figure as it has a value of 40. In contrast, the 5 is the least significant as it only has a value of 5 hundredths. When we desire to use significant figures to indicate the accuracy of approximation, we count the number of digits in the number from left to right, beginning at the first non-zero digit. This is known as the number of significant figures. Example 2 a Write 43.25 to 3 s.f. b Write 0.0043 to 1 s.f. Solution: a We want to write only the three most significant digits. However, the fourth digit needs to be considered to see whether the third digit is to be rounded up or not. That is, 43.25 is written as 43.3 to 3 s.f. b Notice that in this case 4 and 3 are the only significant digits. The number 4 is the most significant digit and is therefore the only one of the two to be written in the answer. That is 0.0043 is written as 0.004 to 1 s.f. 3 Accuracy In the previous lesson, you have studied that numbers can be approximated: a by rounding up b by writing to a given number of decimal place and c by expressing to a given number of significant figure. In this lesson, you will learn how to give appropriate upper and lower bounds for data to a specified accuracy (for example, numbers rounded off or numbers expressed to a given number of significant figures).
  • 52. Mathematics Grade 9 46 Numbers can be written to different degrees of accuracy. For example, although 2.5, 2.50 and 2.500 may appear to represent the same number, they actually do not. This is because they are written to different degrees of accuracy. 2.5 is rounded to one decimal place (or to the nearest tenths) and therefore any number from 2.45 up to but not including 2.55 would be rounded to 2.5. On the number line this would be represented as As an inequality, it would be expressed as 2.45 ≤ 2.5 2.55 2.45 is known as the lower bound of 2.5, while 2.55 is known as the upper bound. 2.50 on the other hand is written to two decimal places and therefore only numbers from 2.495 up to but not including 2.505 would be rounded to 2.50. This, therefore, represents a much smaller range of numbers than that being rounded to 2.5. Similarly, the range of numbers being rounded to 2.500 would be even smaller. Example 3 A girl’s height is given as 162 cm to the nearest centimetre. i Work out the lower and upper bounds within which her height can lie. ii Represent this range of numbers on a number line. iii If the girl’s height is h cm, express this range as an inequality. Solution: i 162 cm is rounded to the nearest centimetre and therefore any measurement of cm from 161.5 cm up to and not including 162.5 cm would be rounded to 162 cm. Thus, lower bound = 161.5 cm upper bound = 162.5 cm ii Range of numbers on the number line is represented as iii When the girl’s height h cm is expressed as an inequality, it is given by 161.5 ≤ h 162.5. 161 161.5 162 162.5 163 2.45 2.4 2.5 2.55 2.6
  • 53. Unit 1 The Number System 47 Effect of approximated numbers on calculations When approximated numbers are added, subtracted and multiplied, their sums, differences and products give a range of possible answers. Example 4 The length and width of a rectangle are 6.7 cm and 4.4 cm, respectively. Find their sum. Solution: If the length l = 6.7 cm and the width w = 4.4 cm Then 6.65 ≤ l 6.75 and 4.35 ≤ w 4.45 The lower bound of the sum is obtained by adding the two lower bounds. Therefore, the minimum sum is 6.65 + 4.35 that is 11.00. The upper bound of the sum is obtained by adding the two upper bounds. Therefore, the maximum sum is 6.75 + 4.45 that is 11.20. So, the sum lies between 11.00 cm and 11.20 cm. Example 5 Find the lower and upper bounds for the following product, given that each number is given to 1 decimal place. 3.4 × 7.6 Solution: If x = 3.4 and y = 7.6 then 3.35 ≤ x 3.45 and 7.55 ≤ y 7.65 The lower bound of the product is obtained by multiplying the two lower bounds. Therefore, the minimum product is 3.35 × 7.55 that is 25.2925 The upper bound of the product is obtained by multiplying the two upper bounds. Therefore, the maximum product is 3.45 × 7.65 that is 26.3925. So the product lies between 25.2925 and 26.3925. Example 6 Calculate the upper and lower bounds to 54.5 , 36.0 given that each of the numbers is accurate to 1 decimal place. Solution: 54.5 lies in the range 54.45 ≤ x 54.55 36.0 lies in the range 35.95 ≤ x 36.05 The lower bound of the calculation is obtained by dividing the lower bound of the numerator by the upper bound of the denominator. So, the minimum value is 54.45÷36.05. i.e., 1.51 (2 decimal places). The upper bound of the calculation is obtained by dividing the upper bound of the numerator by the lower bound of the denominator. So, the maximum value is 54.55 ÷35.95. i.e., 1.52 (2 decimal places).
  • 54. Mathematics Grade 9 48 Exercise 1.9 1 Round the following numbers to the nearest 1000. a 6856 b 74245 c 89000 d 99500 2 Round the following numbers to the nearest 100. a 78540 b 950 c 14099 d 2984 3 Round the following numbers to the nearest 10. a 485 b 692 c 8847 d 4 e 83 4 i Give the following to 1 d.p. a 5.58 b 4.04 c 157.39 d 15.045 ii Round the following to the nearest tenth. a 157.39 b 12.049 c 0.98 d 2.95 iii Give the following to 2 d.p. a 6.473 b 9.587 c 0.014 d 99.996 iv Round the following to the nearest hundredth. a 16.476 b 3.0037 c 9.3048 d 12.049 5 Write each of the following to the number of significant figures indicated in brackets. a 48599 (1 s.f) b 48599 (3 s.f) c 2.5728 (3 s.f) d 2045 (2 s.f) e 0.08562 (1 s.f) f 0.954 (2 s.f) g 0.00305 (2 s.f) h 0.954 (1 s.f) 6 Each of the following numbers is expressed to the nearest whole number. i Give the upper and lower bounds of each. ii Using x as the number, express the range in which the number lies as an inequality. a 6 b 83 c 151 d 1000 7 Each of the following numbers is correct to one decimal place. i Give the upper and lower bounds of each. ii Using x as the number, express the range in which the number lies as an inequality. a 3.8 b 15.6 c 1.0 d 0.3 e –0.2 8 Each of the following numbers is correct to two significant figures. i Give the upper and lower bounds of each. ii Using x as the number, express the range in which the number lies as an inequality. a 4.2 b 0.84 c 420 d 5000 e 0.045
  • 55. Unit 1 The Number System 49 9 Calculate the upper and lower bounds for the following calculations, if each of the numbers is given to 1 decimal place. a 9.5 ×7.6 b 11.0 × 15.6 c 46.5 32.0 d 25.4 8.2 e 4.9 6.4 2.6 + 10 The mass of a sack of vegetables is given as 5.4 kg. a Illustrate the lower and upper bounds of the mass on a number line. b Using M kg for the mass, express the range of values in which it must lie, as an inequality. 11 The masses to the nearest 0.5 kg of two parcels are 1.5 kg and 2.5 kg. Calculate the lower and upper bounds of their combined mass. 12 Calculate upper and lower bounds for the perimeter of a school football field shown, if its dimensions are correct to 1 decimal place. 109.7 m 48.8 m Figure 1.9 13 Calculate upper and lower bounds for the length marked x cm in the rectangle shown. The area and length are both given to 1 decimal place. x 8.4 cm Area = 223.2 cm2 Figure 1.10 1.2.7 Scientific Notation (Standard form) In science and technology, it is usual to see very large and very small numbers. For instance: The area of the African continent is about 30,000,000 km2 . The diameter of a human cell is about 0.0000002 m. Very large numbers and very small numbers may sometimes be difficult to work with or write. Hence you often write very large or very small numbers in scientific notation, also called standard form.
  • 56. Mathematics Grade 9 50 Example 1 6 1.86 10 × - is written in scientific notation. 4 3 8.735 10 and 7.08 10 × × - are written in scientific notation. 1 14.73 10 , 0.0863 10 × × - 4 and 3.864 are not written in standard form (scientific notation). ACTIVITY 1.16 1 By what powers of 10 must you multiply 1.3 to get: a 13? b 130? c 1300? Copy and complete this table. 13 = 1.3 × 101 130 = 1.3 × 102 1,300 = 1.3 × 13,000 = 1,300,000 = 2 Can you write numbers between 0 and 1 in scientific notation, for example 0.00013? Copy and complete the following table. 13.0 = 1.3 × 10 = 1.3 × 101 1.3 = 1.3 × 1 = 1.3 × 100 0.13 = 1.3 × 1 10 = 1.3 × 10−1 0.013 = 1.3 × 1 100 = 0.0013 = 0.00013 = 0.000013 = 0.0000013 = Note that if n is a positive integer, multiplying a number by 10n moves its decimal point n places to the right, and multiplying it by 10−n moves the decimal point n places to the left. Times 10 to a power. Number from 1 up to but not including 10.
  • 57. Unit 1 The Number System 51 Example 2 Express each of the following numbers in scientific notation: a 243, 900,000 b 0.000000595 Solution: a 243,900,000 = 2.439 × 108 . The decimal point moves 8 places to the left. b 0.000000595 = 5.95 × 10−7 . The decimal point moves 7 places to the right. Example 3 Express 2.483 × 105 in ordinary decimal notation. Solution: 2.483 × 105 = 2.483 × 100,000 = 248,300. Example 4 The diameter of a red blood cell is about 7.4 × 10–4 cm. Write this diameter in ordinary decimal notation. Solution: 7.4 × 10–4 = 7.4 × 4 1 10 = 7.4 × 1 10,000 = 7.4 × 0.0001 = 0.00074. So, the diameter of a red blood cell is about 0.00074 cm. Calculators and computers also use scientific notation to display large numbers and small numbers but sometimes only the exponent of 10 is shown. Calculators use a space before the exponent, while computers use the letter E. The calculator display 5.23 06 means 5.23 × 106 . (5,230,000). The following example shows how to enter a number with too many digits to fit on the display screen into a calculator. Example 5 Enter 0.00000000627 into a calculator. Solution: First, write the number in scientific notation. 0.00000000627 = 6.27 × 10−9 Then, enter the number. 6.27 exp 9 / + − giving 6.27 – 09 Decimal notation Scientific notation Calculator display Computer display 250,000 2.5 × 105 2.5 0.5 2.5 E + 5 0.00047 4.7 × 10−4 4.7 – 04 4.7 E – 4 Definition 1.13 A number is said to be in scientific notation (or standard form), if it is written as a product of the form a × 10k where 1 ≤ a 10 and k is an integer.
  • 58. Mathematics Grade 9 52 Exercise 1.10 1 Express each of the following numbers in scientific notation: a 0.00767 b 5,750,000,000 c 0.00083 d 400,400 e 0.054 2 Express each of the following numbers in ordinary decimal notation: a 4.882 × 105 b 1.19 × 10–5 c 2.021 × 102 3 Express the diameter of an electron which is about 0.0000000000004 cm in scientific notation. 1.2.8 Rationalization ACTIVITY 1.17 Find an approximate value, to two decimal places, for the following: i 1 2 ii 2 2 In calculating this, the first step is to find an approximation of 2 in a reference book or other reference material. (It is 1.414214.) In the calculation of 1 , 2 1 is divided by 1.414214… which is a difficult task. However, evaluating 2 2 as 1.414214 0.707107 2 ≈ is easy. Since 1 2 is equivalent to 2 2 (How?), you see that in order to evaluate an expression with a radical in the denominator, first you should transform the expression into an equivalent expression with a rational number in the denominator. The technique of transferring the radical expression from the denominator to the numerator is called rationalizing the denominator (changing the denominator into a rational number). The number that can be used as a multiplier to rationalize the denominator is called the rationalizing factor. This is equivalent to 1.
  • 59. Unit 1 The Number System 53 For instance, if n is an irrational number then 1 n can be rationalized by multiplying it by 1. n n = So, n n is the rationalizing factor. Example 1 Rationalize the denominator in each of the following: a 5 3 8 5 b 6 3 c 3 3 2 Solution: a The rationalizing factor is 5 . 5 So, 5 3 5 3 5 5 15 = = 8 5 8 5 5 8 25 × 2 5 15 5 15 15 = = = 8 5 8 8 5 × b The rationalizing factor is 3 3 So, 2 6 6 3 6 3 6 3 = = = = 2 3 3 3 3 3 3 × c The rationalizing factor is 3 2 3 2 2 2 because 3 3 3 2 4 8 2 × = = So, 3 2 3 3 3 3 3 3 2 3 3 3 2 3 4 3 4 = . = = 2 2 2 2 2 If a radicand itself is a fraction 2 forexample 3         , then, it can be written in the equivalent form 2 3 so that the procedure described above can be applied to rationalize the denominator. Therefore, 2 2 2 2 3 6 6 6 = = = = = 3 3 3 3 3 9 3 × In general, For any non-negative integers a, b (b ≠ 0) a a a b ab b b b b b = = = .
  • 60. Mathematics Grade 9 54 Exercise 1.11 Simplify each of the following. State restrictions where necessary. In each case, state the rationalizing factor you use and express the final result with a rational denominator in its lowest term. a 2 2 b 2 6 c 5 2 4 10 d 12 27 e 5 18 f 3 3 2 3 g 3 1 4 h 2 9 a i 3 3 20 4 j 4 5 More on rationalizations of denominators ACTIVITY 1.18 Find the product of each of the following: 1 ( )( ) 2 3 2 3 + − 2 ( )( ) 5 3 2 5 3 2 + − 3 1 1 5 3 5 3 2 2    − +       You might have observed that the results of all of the above products are rational numbers. This leads you to the following conclusion: Using the fact that (a – b) (a + b) = a2 – b2 , you can rationalize the denominators of expressions such as 1 1 1 , , a b a b a b + − − where , a b are irrational numbers as follows. i ( ) ( ) 2 2 2 1 1 a b a b a b a b a b a b a b a b   − − − = = =     − + − +   − ii ( ) 2 2 2 1 1 a b a b a b a b a b a b a b a b   + + + = = =     − − − +   − iii ( ) ( ) ( ) 2 2 1 1 a b a b a b a b a b a b a b a b   + + + = = =     − − + −   −
  • 61. Unit 1 The Number System 55 Example 2 Rationalize the denominator of each of the following: a 5 1 2 − b 3 6 3 2 + Solution: a The rationalizing factor is 1+ 2 1 + 2 So ( ) 2 2 5 5(1 2) 5 5 2 1 2 (1 2)(1 2) 1 2 + + = = − − + − 5 + 5 2 = = 5 5 2 1 2 − − − b The rationalizing factor is 6 3 2 6 3 2 − − So ( ) ( ) ( ) ( ) 2 2 3 6 3 2 3 3 6 3 2 = = 6 + 3 2 6 3 2 6 + 3 2 6 3 2 − − − − ( ) 3 6 3 2 = 6 18 − − = ( ) 1 6 3 2 4 − − 3 2 6 4 − = Exercise 1.12 Rationalize the denominator of each of the following: a 1 3 5 − b 18 5 3 − c 2 5 3 − d 3 4 3 2 + − e 10 7 2 − f 3 2 + 3 3 2 2 3 − g 1 2 3 1 + −
  • 62. Mathematics Grade 9 56 1.2.9 Euclid's Division Algorithm A The division algorithm ACTIVITY 1.19 1 Is the set of non-negative integers (whole numbers) closed under division? 2 Consider any two non-negative integers a and b. a What does the statement a is a multiple of b mean? b Is it always possible to find a non-negative integer c such that a = bc? If a and b are any two non-negative integers, then a ÷ b (b ≠ 0) is some non-negative integer c (if it exists) such that a = bc. However, since the set of non-negative integers is not closed under division, it is clear that exact division is not possible for every pair of non-negative integers. For example, it is not possible to compute 17 ÷ 5 in the set of non-negative integers, as 17 ÷ 5 is not a non-negative integer. 15 = 3 × 5 and 20 = 4 × 5. Since there is no non-negative integer between 3 and 4, and since 17 lies between 15 and 20, you conclude that there is no non-negative integer c such that 17 = c × 5. You observe, however, that by adding 2 to each side of the equation 15 = 3 × 5 you can express it as 17 = (3 × 5) + 2. Furthermore, such an equation is useful. For instance it will provide a correct answer to a problem such as: If 5 girls have Birr 17 to share, how many Birr will each girl get? Examples of this sort lead to the following theorem called the Division Algorithm. In the theorem, a is called the dividend, q is called the quotient, b is called the divisor, and r is called the remainder. Example 1 Write a in the form b × q + r where 0 ≤ r b, a If a = 47 and b = 7 b If a = 111 and b = 3 c If a = 5 and b = 8 Theorem 1.4 Division algorithm Let a and b be two non-negative integers and b ≠ 0, then there exist unique non-negative integers q and r, such that, a = (q × b) + r with 0 ≤ r b.
  • 63. Unit 1 The Number System 57 Solution: a 6 b 37 c 0 7 47 3 111 8 5 42 9 0 5 21 5 q = 6 and r = 5 21 q = 0 and r = 5 ∴ 47 = 7 (6) + 5 0 ∴ 5 = 8 (0) + 5. q = 37 and r = 0 ∴ 111 = 3 (37) + 0 Exercise 1.13 For each of the following pairs of numbers, let a be the first number of the pair and b the second number. Find q and r for each pair such that a = b × q + r, where 0 ≤ r b: a 72, 11 b 16, 9 c 11, 18 d 106, 13 e 176, 21 f 25, 39 B The Euclidean algorithm ACTIVITY 1.20 Given two numbers 60 and 36 1 Find GCF (60, 36). 2 Divide 60 by 36 and find the GCF of 36 and the remainder. 3 Divide 36 by the remainder you got in Step 2. Then, find the GCF of the two remainders, that is, the remainder you got in Step 2 and the one you got in step 3. 4 Compare the three GCFs you got. 5 Generalize your results. The above Activity leads you to another method for finding the GCF of two numbers, which is called Euclidean algorithm. We state this algorithm as a theorem. Theorem 1.5 Euclidean algorithm If a, b, q and r are positive integers such that a = q × b + r, then, GCF (a, b) = GCF (b, r).
  • 64. Mathematics Grade 9 58 Example 2 Find GCF (224, 84). Solution: To find GCF (224, 84), you first divide 224 by 84. The divisor and remainder of this division are then used as dividend and divisor, respectively, in a succeeding division. The process is repeated until a remainder 0 is obtained. The complete process to find GCF (224, 84) is shown below. Euclidean algorithm Computation Division algorithm form Application of Euclidean Algorithm 2 84 224 168 56 1 56 84 56 28 2 28 56 56 0 224 = (2 × 84) + 56 84 = (1 × 56) + 28 56 = (2 × 28) + 0 GCF (224, 84) = GCF (84, 56) GCF (84, 56) = GCF (56, 28) GCF (56, 28) = 28 (by inspection) Conclusion GCF (224, 84) = 28. Exercise 1.14 1 For the above example, verify directly that GCF (224, 84) = GCF (84, 56) = GCF (56, 28). 2 Find the GCF of each of the following pairs of numbers by using the Euclidean Algorithm: a 18; 12 b 269; 88 c 143; 39 d 1295; 407 e 85; 68 f 7286; 1684
  • 65. Unit 1 The Number System 59 Key Terms bar notation principal nth root composite number principal square root divisible radical sign division algorithm radicand factor rational number fundamental theorem of arithmetic rationalization greatest common factor (GCF) real number irrational number repeating decimal least common multiple (LCM) repetend multiple scientific notation perfect square significant digits prime factorization significant figures prime number terminating decimal Summary Summary 1 The sets of Natural numbers, Whole numbers, Integers and Rational numbers denoted by ℕ, W, ℤ, and ℚ, respectively are described by ℕ = {1, 2, 3,…} W = {0, 1, 2,…} ℤ = {…,−3, −2, −1, 0, 1, 2, 3,…} : , , 0 a a b b b   = ∈ ∈ ≠     ℚ ℤ ℤ 2 a A composite number is a natural number that has more than two factors. b A prime number is a natural number that has exactly two distinct factors, 1 and itself. c Prime numbers that differ by two are called twin primes. d When a natural number is expressed as a product of factors that are all prime, then the expression is called the prime factorization of the number.
  • 66. Mathematics Grade 9 60 e Fundamental theorem of arithmetic. Every composite number can be expressed (factorized) as a product of primes, and this factorization is unique, apart from the order in which the prime factors occur. 3 a The greatest common factor (GCF) of two or more numbers is the greatest factor that is common to all numbers. b The least common multiple (LCM) of two or more numbers is the smallest or least of the common multiples of the numbers. 4 a Any rational number can be expressed as a repeating decimal or a terminating decimal. b Any terminating decimal or repeating decimal is a rational number. 5 Irrational numbers are decimal numbers that neither repeat nor terminate. 6 The set of real numbers denoted by ℝ is defined by ℝ = {x: x is rational or x is irrational} 7 The set of irrational numbers is not closed under addition, subtraction, multiplication and division. 8 The sum of an irrational and a rational number is always an irrational number. 9 For any real number b and positive integer n 1 1 n n b b = (Whenever n b is a real number) 10 For all real numbers a and b ≠ 0 for which the radicals are defined and for all integers n ≥ 2: i n n n ab a b = ii n n n a a b b = 11 A number is said to be written in scientific notation (standard notation), if it is written in the form a × 10k where 1 ≤ a 10 and k is an integer. 12 Let a and b be two non-negative integers and b ≠ 0, then there exist unique non- negative integers q and r such that a = (q × b) + r with 0 ≤ r b. 13 If a, b, q and r are positive integers such that a = q × b + r, then GCF (a, b) = GCF (b, r).
  • 67. Unit 1 The Number System 61 Review Exercises on Unit 1 1 Determine whether each of the following numbers is divisible by 2, 3, 4, 5, 6, 8, 9 or 10: a 533 b 4, 299 c 111 2 Find the prime factorization of: a 150 b 202 c 63 3 Find the GCF for each set of numbers given below: a 16; 64 b 160; 320; 480 4 Express each of the following fractions or mixed numbers as a decimal: a 5 8 b 16 33 c 4 5 9 d 1 3 7 5 Express each of the following decimals as a fraction or mixed number in its simplest form: a 0.65 b – 0.075 c 0.16 d 24.54 − e 0.02 − 6 Arrange each of the following sets of rational numbers in increasing order: a 71 3 23 , , 100 2 30 − b 3.2,3.22,3.23,3.23 c 2 11 16 67 , , , 3 18 27 100 7 Write each of the following expressions in its simplest form: a 180 b 169 196 c 3 250 d 2 3 3 2 180 + + 8 Give equivalent expression, containing fractional exponents, for each of the following: a 15 b a b + c 3 x y − d 4 13 16 9 Express the following numbers as fractions with rational denominators: a 1 2 +1 b 5 3 c 5 3 7 − + d 4 13 16
  • 68. Mathematics Grade 9 62 10 Simplify a ( ) ( ) 3 7 2 7 12 + + − b ( ) ( ) 2 5 2 5 + + − c 2 6 3 54 ÷ d ( ) 2 3 7 2 7 + − 11 If 5 2.236 and 10 3.162 ≈ ≈ , find the value of 10 20 40 5 80 + + − − 12 If 2 3 6 , 3 2 2 3 x y + = + − find the values of x and y. 13 Express each of the following numbers in scientific notation: a 7,410,00 b 0.0000648 c 0.002056 d 12.4 × 10–6 14 Simplify each of the following and give the answer in scientific notation: a 109 × 10– 6 × 27 b 4 2 7 796 10 10 10 − − × × c 0.00032 × 0.002 15 The formula 3.56 d h = km estimates the distance a person can see to the horizon, where h is the height of the eyes of the person from the ground in metre. Suppose you are in a building such that your eye level is 20 m above the ground. Estimate how far you can see to the horizon. h = 20 m d Figure 1.11
  • 69. Unit Outcomes: After completing this unit, you should be able to: identify equations involving exponents and radicals, systems of two linear equations, equations involving absolute values and quadratic equations. solve each of these equations. Main Contents 2.1 Equations involving exponents and radicals 2.2 Systems of linear equations in two variables 2.3 Equations involving absolute value 2.4 Quadratic equations Key Terms Summary Review Exercises U Un ni it t S S S S S S S SO O O O O O O OL L L L L L L LU U U U U U U UT T T T T T T TI I I I I I I IO O O O O O O ON N N N N N N N O O O O O O O OF F F F F F F F E E E E E E E EQ Q Q Q Q Q Q QU U U U U U U UA A A A A A A AT T T T T T T TI I I I I I I IO O O O O O O ON N N N N N N N
  • 70. Mathematics Grade 9 64 INTRODUCTION In earlier grades, you have learnt also learned about linear equations in one variable and the methods to solve them. In the present unit, we discuss further about equations involving exponents, radicals, and absolute values. You shall also learn about systems of linear equations in two variables, quadratic equations in single variable, and the methods to solve them. 2.1 EQUATIONS INVOLVING EXPONENTS AND RADICALS Equations are equality of expressions. There are different types of equations that depend on the variable(s) considered. When the variable in use is said to be an equation involving exponents. 1 Determine whether or not each of the following is true. a 24 × 25 = 220 d 2n × 22 = 22n 2 Express each of the following a 8 b The above Activity leads you to revise the rules of exponents Example 1 Solve each of the following equations. a 3 x = Solution: a 3 x = 2 2 3 x = x = 9 Therefore x = 9. b To solve x3 = 8 the nth root of b x3 = 8 3 8 2 x = = INTRODUCTION you have learnt about algebraic equations and their classification. about linear equations in one variable and the methods to solve them. In the present unit, we discuss further about equations involving exponents, radicals, and shall also learn about systems of linear equations in two variables, ratic equations in single variable, and the methods to solve them. EQUATIONS INVOLVING EXPONENTS AND RADICALS Equations are equality of expressions. There are different types of equations that depend on the variable(s) considered. When the variable in use has an exponent is said to be an equation involving exponents. ACTIVITY 2.1 er or not each of the following is true. b (32 )3 = 36 c ( ) 1 2 2 5 5 = e 2x = 8 is equivalent to x = 3. following numbers in power form. 27 c 625 d 343 leads you to revise the rules of exponents that you discussed in Solve each of the following equations. b 3 8 x = c 2 16 x = Squaring both sides = 8, recall that for any real number a and b, if an b. 8 2 = = about algebraic equations and their classification. You about linear equations in one variable and the methods to solve them. In the present unit, we discuss further about equations involving exponents, radicals, and shall also learn about systems of linear equations in two variables, EQUATIONS INVOLVING EXPONENTS Equations are equality of expressions. There are different types of equations that depend exponent other than 1, it that you discussed in Unit 1. n = b, then a is
  • 71. c To solve 2 16 x = In 2x = 24 , the bases are equal and Therefore x = 4 is the solution. Solve each of the following equations a 2 2 8 2 x x+ = The following rule is very useful in solving such equations. Rule: For a 0, ax = a Example 2 Solve 2 1 2 3 3 x x + − Solution: By using the rule, since 3 0, 2x + 1 = x – 2. From this we can see that the solution is Example 3 Solve each of the following equations a 2 1 8 2 x x+ = Solution: a 2 1 8 2 x x+ = ( ) 3 2 1 2 2 x x+ = 3 2 1 2 2 x x+ = 3x = 2x + 1 Therefore x = 1. b 3 3 9 27 x x − = ( ) ( ) 3 3 2 3 3 3 x x − = ( ) ( ) 2 3 3 3 3 3 x x − = 2 6 9 3 3 x x − = 2 6 9 x x − = 7 6 x = − Therefore 6 . 7 x = − c 3 2 5 3 3 x x+ = ( ) 1 2 5 3 3 3 x x+ = Unit 2 Solutions of Equations 2 16 = , first express 4 2 16 as 2 2 x x = = . , the bases are equal and hence the exponents must be equal. = 4 is the solution. ACTIVITY 2.2 following equations. b 1 4 2 x x + = c 2 5 25 x = The following rule is very useful in solving such equations. ay , if and only if x = y. 2 1 2 3 3 x x + − = By using the rule, since 3 0, 2 1 2 3 3 x x + − = , if and only if the exponents From this we can see that the solution is x = –3. Solve each of the following equations. b 3 3 9 27 x x − = c 3 2 5 3 3 x x+ = Expressing 8 as a power of 2 Applying laws of exponents x x 3 3 x x Expressing 9 and 27 as powers of 3 ) x x Applying laws of exponents Applying laws of exponents Solutions of Equations 65 the exponents must be equal. if and only if the exponents
  • 72. Mathematics Grade 9 66 2 5 3 3 3 x x+ = 2 5 3 x x = + ( 3 2 5 = + x x x = 6x + 15 −5x = 15 Therefore x = – 3. 1 Solve each of the following equations a 3 27 x = d 5 2 1 81 243 x+ = g 3 (3 1) 64 x + = 2 Solve ( ) ( 2 2 2 3 3 1 . x x + = − 3 Solve each of the following equations a 2 1 2 1 9 27 81 x x x − + = c 3 4 3 4 1 16 2 64 x x x + − + = 2.2 SYSTEMS OF LINEAR EQUATIONS IN TWO VARIABLES Recall that, for real numbers called a linear equation. The numbers 1 Solve each of the following linear equations a x – 2 = 7 c 2x = 4 2 How many solutions do you 2 5 ) 3 2 5 = + + 15 Exercise 2.1 Solve each of the following equations. b 1 16 4 x   =     c       e 2 2 1 9 27 x x+ = f 16 2 h 3 2 1 81 3 x x − = ) 2 2 2 3 3 1 . x x + = − following equations. 2 1 2 1 x x x − + b 2 2 2 3 2 1 9 243 81 x x x + + − −   =     3 4 3 4 1 16 2 64 x x x + − + SYSTEMS OF LINEAR EQUATIONS IN TWO VARIABLES or real numbers a and b, any equation of the form ax + b = 0 he numbers a and b are called coefficients of the equation. ACTIVITY 2.3 Solve each of the following linear equations. b x + 7 = 3 d 2x – 5 = 7 e 3x + 5 = 14 How many solutions do you get for each equation? 3 1 1 32 16 x−   =     4 3 16 2 x x + = SYSTEMS OF LINEAR EQUATIONS IN = 0, where a ≠ 0 is of the equation. + 5 = 14
  • 73. Observe that each equation has exactly one solution. In general, any linear equation one variable has one solution. G r o u p W o r k 2 . 1 G r o u p W o r k 2 . 1 G r o u p W o r k 2 . 1 G r o u p W o r k 2 . 1 Form a group and do the following 1 Solve each of the following equations a ( 7 3 2 3 2 x x − = + c ( 2 4 2 5 x x + = + 2 How many solutions do you get 3 What can you conclude about number of solutions? From the Group Work, observe that solutions or no solution. Linear equations in two variables We discussed how we solve ax + b = 0. What do you think the solution is 1 Which of the following are linear equations in two variables? a 2x – y = 5 d 2x – y2 = 7 2 How many solutions 3 A house was rented for per m3 . a Write an equation for the total cost b If the total cost an equation. Definition 2.1 Any equation that can be reduced to the form and a ≠ 0, is called a linear equation Unit 2 Solutions of Equations Observe that each equation has exactly one solution. In general, any linear equation has one solution. G r o u p W o r k 2 . 1 G r o u p W o r k 2 . 1 G r o u p W o r k 2 . 1 G r o u p W o r k 2 . 1 Form a group and do the following. Solve each of the following equations. ) 7 3 2 3 2 − = + b ( ) ( ) 3 2 4 2 3 6 x x − + = − − ) 2 4 2 5 + = + How many solutions do you get for each equation? What can you conclude about number of solutions? observe that such equations can have one solution, infinite Linear equations in two variables discussed how we solve equations with one variable that can be reduced to the What do you think the solution is, if the equation is given as ACTIVITY 2.4 Which of the following are linear equations in two variables? b –x + 7 = y c 2x+3 = 4 e 1 1 6 x y + = How many solutions are there for each of the linear equations in two variables for Birr 2,000 per month plus Birr 2 for water consumption rite an equation for the total cost of x-years rent and 200 m3 If the total cost for x-years rent and y m3 of water used is Birr 106 ny equation that can be reduced to the form ax + b = 0, where linear equation in one variable. Solutions of Equations 67 Observe that each equation has exactly one solution. In general, any linear equation in can have one solution, infinite equations with one variable that can be reduced to the form if the equation is given as y = ax + b? of the linear equations in two variables? irr 2 for water consumption 3 of water used. irr 106,000 write where a, b ∈ℝ